Download as pdf or txt
Download as pdf or txt
You are on page 1of 331

Based on NCERT Syllabus

Objective
Science

Objective Science
Based on NCERT Books
Content contribution :
Shivendra Pratap Singh | AU Education Team
© Amar Ujala Publications Ltd.
Published by Amar Ujala Publications Ltd. and printed at C-21, Sector
59, Noida - 201301 (U.P.)
Latest Edition **
ISBN : 978-93-82948-52-0
Due care and diligence has been taken while publishing this book. However, the publisher
does not hold any responsibility for any mistake that may have inadvertently crept in. The
publisher does not accept responsibility for any loss arising out of the use of this book.

All rights reserved. Neither this publication nor any part of it may be reproduced, stored in a
retrieval system, or transmitted in any form or by any means, electronic, mechanical,
photocopying, recording or otherwise, without the prior written permission of the publisher.

All disputes are subject to the exclusive jurisdiction of competent courts and forums in Noida
only.

Part - 1 Biology

Physiology and Diseases 7


Vitamins and Plant Hormones 14
Circulatory System and Nutrition 20
Plants and Animals 26
Science and Technology 31
Modern Technology 37
Recent Medical Advancements 43
Cure and Precautions 49
I
Part - 2 Enviorenmental Biology

National Park and Eco System 55


Degradation and Pollution 61
Planning and Management 67

n
Part - 3 Physics

Mechanics 73
Waves and Currents 79
Electromagnetic 84
Heat and Thermodynamics 89
Wave and Optics 94

Electricity 99
Light and Mechanics 105
General Physics 110
Physics : Technology 115
Sound and Other Mechanics 120
Functional Properties 125
Technological Developments 131
Miscellaneous Questions 136

e
Part - 4 Chemistry

Radioactivity 142
Organic and Inorganic Chemistry 148
Acid, Base and Salt 155
Carbon and Other Substances 160

Laws and Applications 166


Chemistry : Different Dimensions 172
General Chemistry 178
Carbon Compounds and Metals 183
Science and Technology 188

Part - 1
Biology
Physiology and Diseases

Q1. Metastaris is the process by which:

(a) Cancer cells are successfully inhibited to divide any further.


(b) The chromosomes in cells nuclei are attached to the spindle
before moving to the anaphase poles.
(c) Cells divide rapidly under the influence of drugs.
(d) Cancer cells spread through the blood or lymphatic system to
other sites or organs.
Ans. – (d) Cancer cells spread through the
blood or lymphatic system to other sites or
organs by the process called metastaris.
Cancer causing agents are chemical, virus
and continuous physical irritation. Q2. Match List-I with List-II and
select the
correct answer using the codes given
below.
List-I (Achievement in genetics)
(A)Discovery of transduction and conjugation in bacteria.
(B) Establishing the sex-linked inheritance.
(C) Isolation of DNA polymerase from E. Coli.
(D)Establishing the complete genetic code.
List-II (Scientist)
1. Lederberg 2. Morgan
3. Korenberg 4. Hargovind Khurana
Codes:

AB C D
(a) 1 2 3 4
(b) 2 3 4 1
(c) 3 4 1 2
(d) 4 1 2 3
Ans. – (a)
Discovery of transduction
and conjugation in bacteria – Lederberg Establishing the sex-linked
inheritance – Morgan Isolation of DNA polymerase
from E. Coli
Establishing the complete genetic code
– Korenberg

– Khurana

Q3. A small pouch containing silica gel is often found in bottles


of medicine in tablet or powder form because silica gel: (a)
absorbs moisture
(b) kills virus
(c) kills germs and spores
(d) absorbs all who present inside the bottle

Ans. – (a) Silica gel, found in medicinal bottle absorbs moisture and
saves the medicines. It has greater application in medicine
packaging. It is not to kill virus, germs and spores and does not
absorbs all gases thar are present inside the bottle.

Q4. Match List-I with List-II and select the correct answer using
the codes given below.
List-I
(Secreted Hormones)
(A)Growth hormones
(B) Progesterone
(C) Cortisone
(D)Insulin
List-II
(Endocrine Glands)
(1) Adrenal
(2) Pituitary Gland
(3) Pancreas
(4) Gonads
Codes:
AB C D
(a) 1 2 3 4
(b) 2 4 1 3
(c) 3 2 4 1
(d) 4 3 1 2
Ans. – (b)
Growth Hormone – Pituitary Gland Progesterone – Gonads
Cortisone – Adrenal
Insulin – Pancreas Q5. Match List-I with List-II and select the
correct answer using the codes given below.
List-I
(Types of Disease) (A)Fungal Infection (B) Hormonal Disorder (C)
Deficiency Disease (D)Genetic Disease Codes:
List-II
(Related Disease) (1) Diabetes
(2) Ringworm (3) Hemophilia (4) Rickets

AB C D (a) 1 2 3 4 (b) 2 1 4
(c) 3 4 1 2 (d) 4 3 2 1

Ans. – (b)
Fungal Infection Hormonal Disorder Deficiency Disease Genetic
Disease
– Ringworm
– Diabetes
– Rickets
– Hemophilia

Q6. Consider the following statements: Assertion (A): DNA finger


printing has not enough powerful tool to establish paternity and
identity of criminals in rape and other assault cases.
Reasons (R): Trace evidences such a hairs, saliva and dried semen
are adequate for DNA analysis.
Codes:
(a) Both A and R are true and R is the correct explanation of A.
(b) Both A and R are true but R is not a correct explanation of A.
(c) A is true but R is false.
(d) A is false but R is true.
Ans. – (d) Assertion is false but Reason is true. DNA finger printing
has become a powerful tool due to its ability in tracing evidences of
crime and to establish the paternity. These evidences can be
collected from hairs, semen and saliva.

Q7. Consider the following statements and select the correct


answer from the code given below:
Assertion (A): In human being, the females not play a major role in
determining the sex of the offspring.
Reason (R): Woman have two X chromosome.
Codes:
(a) Both A and R are true and R is the correct explanation of A.

(b) Both A and R are true but R is not a correct explanation of A.


(c) A is true, R is false.
(d) A is false, R is true.
Ans. – (a) Both Assertion and Reason are true and R is the correct
explanation. In human being, male play the major role in determining
the sex of the offspring because male has XY chromosome while the
female has XX chromosome. XY combination of chromosome
produces male child while XX combination produces female child.
Q8. Within biological communities, some species are important
in determining the ability of a large number of other species to
persist in the community. Such species are called:
(a) Threatened Species
(b) Keyrtone Species
(c) Allopatric Species
(d) Sympatric Species
Ans. – (b) Within biological communities, some species may be
important in determining the ability of large number of other species
to persist in the community. These crucial species are known as key
stone species. Allopatric species are species having exclusive areas
of geographic distribution. Sympatric species are species having
overlapping area of geographic distribution.
Q9. Match List-I with List-II and select the correct answer using
the codes given below.
List-I
List-II
(Drug/Chemicals) (Their Uses)
(A)Pyrethrin (1) Mosquito control
(B) Nitroglycerine (2) Dilation of pupil
(C) Atropine (3) Heart Trouble
(D)Ether (4) Local Anaesthesia
Codes:
AB C D
(a) 1 2 3 4
(b) 1 3 2 4
(c) 2 3 1 4
(d) 4 2 3 1

Ans. – (b)
Pyrethrin
Nitroglycerine Atropine
Ether
– Mosquito control
– Heart Trouble
– Dilation of pupil
– Local Anaesthesia

Q10. Consider the following and match List-I with List-II and
select the correct answer using the codes given below. List-I
(A)Pheromones (B) Insulin

(C) Adrenaline (D)Estrogen

List-II
(1) Females
(2) Anger, fear,

danger
(3) Glucose
(4) Attracting

partners through sense of smell


Codes: AB C D
(a) 1 2
(b) 2 3
(c) 4 3
(d) 3 2
Ans. – (c)
Pheromones

Insulin
Adrenaline Estrogen 3 4 4 1 2 1 1 4

– Attracting partners through sense of smell


– Glucose
– Anger, fear, danger
– Females

Q11. Consider the following statements and select the correct


answer from the code given below:
Assertion (A): Insect resistant transgenic cotton has been produced
by inserting B+ gene.
Reason (R): The B+ gene is derived from a bacterium.
(a) Both (A) and (R) are true and (R) is the

correct explanation of (A).


(b) Both (A) and (R) are true but (R) is not
a correct explanation of (A).
(c) A is true but R is false.
(d) A is false but R is true.
Ans. – (b) B+ cotton is a transgenic crop. Transgenic plants are those
plants, which have foreign gene incorporated in their DNA. This
insect resistant gene is derived from a bacterium, bacillus
thurigiensis. Q12. Match List-I with List-II and select the correct
answer using the codes given below.
List-I
(A)Protozoan (B) Bacteria (C) Fungi
(D)Virus
Codes:
List-II
(1) Tuberculosis (2) Malaria
(3) Poliomyelitis (4) Ringworm

AB C D (a) 2
(b) 1
(c) 3
(d) 4
Ans. – (a) Protozoan Bacteria Fungi
Virus
143234214321

– Malaria
– Tuberculosis
– Ringworm
– Poliomyelitis

Q13. Vitamins are required for the metabolic activities of our


body. Find reasons: (1) It is classified for their biological and

chemical activities, not for structural activities.


(2) It is mainly stored in small intestine.
(3) It is not synthesized in sufficient quantities by body.
(a) 1 and 3 (b) 2 and 3
(c) 1 and 2 (d) All of above
Ans. – (a) Vitamins is essential for human
body for metabolic activities. It is not sufficiently synthesized by our
body. We
require vitamins from vegetables, fruits
etc.
Its classification is based on biological and chemical activities not for
structural activities. It is stored in liver.
Q14. The stones formed in human kidney consist mostly of:
(a) Sodium sulphate
(b) Sodium acetate
(c) Magnesium sulphate
(d) None of these
Ans. – (d) Uric acid is relatively less soluble in water and may
crystallize and gets deposited in the joints causing gouts. Excessive
uric acid and certain salt like calcium oxalate become the source of
kidney stones, when the kidneys are not functioning properly.
Q15. The complete conversion of glucose, in the presence of
oxygen, into carbon dioxide and water with release of energy is
called:
(a) Hydrolysis
(b) Glycolysis
(c) Anaerobic respiration
(d) None of these
Ans. – (d) The complete conversion of glucose in the presence of
oxygen into carbon dioxide and water with release of energy is called
Anaerobic respiration. Oxygen is not utilized during glycolysis and
anaerobic respiration.
Q16. Match List-I with List-II and select the correct answer using
the codes given below the lists.
List-I
(A)Theory of Mutation
(B) Operon concept
(C) Theory of evolution
(D)One gene one enzyme hypothesis List-II
1. De Vries
2. Beadle and Tatum
3. Darwin
4. Jacob and Monod Code:
AB C D
(a) 1 2 3 4
(b) 1 4 3 2
(c) 3 4 2 1
(d) 4 3 2 1
Ans. – (b)
Theory of Mutation – De Vries Operon Concept – Jacob and Mono
Theory of Evaluation – Darwin One gene one enzyme
hypothesis – Beadle and Tatum Q17. Match List-I with List-II and
select the correct answer using the codes given below the lists:
List-I List-II
(A)Fruit (1) Ovary
(B) Seeds (2) Ovule
(C) Wood (3) Stem
(D)Starch (4) Leaf
Code:
AB C D
(a) 1 2 3 4
(b) 2 3 4 1
(c) 3 2 1 4
(d) 4 2 3 1
Ans. – (b) Fruit is itself a mature ovary formed after pollination and
fertilization. Seeds are developed from the fertilized ovule. Stem of a
tree is formed of wood starch is the main reserve food in plants which
is produced by the leaves via photosynthesis.
Q18. Match List-I with List-II and select the correct answer using
the codes given below.
List-I
List-II (A)Malaria (1) Brain (B) Encephalitis (C) Filaria
(D)Leukaemia (2) Blood Cells (3) Muscles
(4) Lymph node (5) Bone Marrow Code: AB C D (a) 1 2 (b) 2 1 (c) 3 2
(d) 5 1 Ans. – (b)
Malaria
Encephalitis
Filaria
Leukaemia
35454124

– Blood Cell
– Brain
– Lymph node
– Bone Marrow

Q19. Which one of the following organism is likely to show the


highest concentration of DDT, once it has been introduced into
the ecosystem: (a) Toad
(c) Cattle
(b) Grasshopper (d) None of these

Ans. – (a) Biomagnification is the phenomenon of increasing


concentration of a compound in the tissue of organism, as the
compound passes up a food chain, usually as a result of food intake.
In this case, the concentration of compound will increase with
increasing the trophic level. DDT is a non biogradable, biomagnifying
pollutant which increases in constraition from producers and top
consumer. In the given question, only snake represents the teriary
consumer, so the concentration of DDT will be highest in snake.

Q20. Match the following and choose correct answer by using


the codes given below. List-I
(A)Syphilis
(B) Meningitis
(C) Food Poisoning
(D)Tuberculosis

List-II
(1) Mycobacterium tuberculosis (2) Clostridium botulinum (3)
Neisseria meningitis
(4) Treponema palladium
Code:

AB C D
(a) 1 2 3 4
(b) 4 3 2 1
(c) 2 4 3 1
(d) 3 1 4 2
Ans. – (b)
Syphilis – Treponema palladium Meningitis – Neisseria meningitis
Food Poisoning – Clostridium botulinum Tuberculosis –
Mycobacterium Tuberculosis
Q21. Match the following and choose correct answer by using
the codes given below. List-I (Name of Diseases)
(A)Pneumonia
(B) Tetanus
(C) Diphtheria
(D)Whooping cough
List-II (Related Bacteria)
(1) Clostridium tetany
(2) Diplococcus pneumonia
(3) Bacillus pertussis
(4) Corynebacterium Diphtheria
Codes:
AB C D
(a) 1
(b) 2
(c) 3
(d) 4
Ans. – (b) Pneumonia

Tetanus
234
143
421
312

– Diplococcus
pneumonia
– Clostridium tetany Diphtheria – Corynebacterium
Diphtheria
Whooping cough – Bacillus pertussis
Q22. Match the following and choose correct answer by using
the codes given below. List-I(Suspected Cancer Carcinogen)
(A)Asbestos
(B) Vinyl Chloride
(C) Aromatic amines
(D)Cadmium
List-II (Affected Organ)
(1) Urethra
(2) Kidney and Lungs
(3) Lungs
(4) Brain and Liver
Codes:
AB C D
(a) 1 2 3 4
(b) 2 3 4 1
(c) 3 4 1 2
(d) 4 1 2 3
Ans. – (c)
Asbestos – Lungs
Vinyl Chloride – Brain and Liver
Aromatic amines – Urethra
Cadmium – Kidney and Lungs
Q23. Which of the following not correct:
(a) Turner’s Syndrome, – Genetic
Down Syndrome, Disease
Hemophilia
(b) Bird Flue, Influenza, – Infectious
H1N1 Disease
(c) Atherosclerosis, – Degenerative
Heart Failure, Diseases
Diabetes
(d) Gonorrhoea, Syphilis, – Sexually
Herpes, Epilepsy, transmitted
AIDS Disease (SIDs)
Ans. – (d) Gonorrhoea, Syphilis, Herpes
and AIDS are sexually transmitted disease but Epilepsy is not a
sexual disease. Epilepsy is type of mental illness. Turner’s syndrome,
Down syndrome and Hemophilia are genetic disease. Bird flue,
Influenza and H1N1 are infectious disease. Atherosclerosis, heart
failure and diabetes are degenerative disease.
Q24. Living organism require at least 27 elements of which 15
are metals. Among these, those required in major quantities,
include:
(a) Potassium, sodium, magnesium and calcium
(b) Sodium, magnesium, manganese and copper
(c) Molybdenum, calcium, manganese and sodium
(d) Manganese, potassium, copper and magnesium
Ans. – (a) Potassium, sodium, magnesium
and calcium are required in major quantities. Magnesium helps in
metabolic process. Calcium is used in the miotic spindles during cell
division.
Q25. Which of the following are associated with diabetes
mellitus, a common disease in adults:
(1) Lower insulin level in blood
(2) Higher insulin level in blood
(3) Lower sugar level in blood
(4) Higher sugar level in blood
Codes:
(a) 1 & 2 (b) 3 & 4
(c) 1 & 4 (d) 2 & 3
Ans. – (c) Diabetes mellitus is caused due to deficiency of insulin
hormone, secreted by pancreas. Insulin helps in maintaining a
constant blood sugar level. Diabetes mellitus are associated with
lower insulin level in blood and higher sugar level in blood.

Vitamins and Plant Hormones

Q1. Match the following and choose correct answer by using the
codes given below.

List-I
(A) Sleeping Disease (B) Tuberculosis (C) Polio
(D) Asthma

List-II
(1) Fungus (2) Virus
(3) Bacteria (4) Protozoa

Codes: AB C D
(a) 1234
(b) 2341
(c) 3412
(d) 4321
Ans. – (d)
Sleeping Disease
Tuberculosis
Polio
Asthma
– Protozoa
– Bacteria
– Virus
– Fungus Q2. Which of the following not correct: (a) Sleeping
Disease, Dysentery, Malaria – Protozoa
(b) AIDS, Polio, Rabbis – Virus (c) Titenes, Cholera,
Gonorrhea – Bacteria
(d) Asthma, Athletes Foot,
Baldners – Helminthus
Ans. – (d) Asthma, Athlete foot and baldners are caused by fungus
but not by Helminthus. Diarrhea, Filaria is caused by helminthus.
Sleeping disease, Dysentery and Malaria are caused by Protozoa.
AIDS, Polio and Rabbis are caused by virus and Titenes, Cholera and
Gonorrhea are caused by Bacteria.
Q3. Match List-I with List-II and select the correct answer by
using the codes given below:
List-I (Human Disease) )

(A) Malaria
(B) Pyuria
(C) Dysentery
(D) Sleeping Disease List-II (Effected Part) (1) Gums
(2) Spline & RBC (3) Brain
(4) Intestine
Codes:

AB C D
(a) 1 2 3 4
(b) 2 1 4 3
(c) 3 4 1 2
(d) 4 3 2 1
Ans. – (b)
Malaria – Spline & RBC Pyuria – Gums
Dysentery – Intestine Sleeping Disease – Brain
Q4. Which of the following not correct: (a) Wheat, Corn, Potato,
Banana – Carbohydrate
(b) Milk, Egg, Paneer – Vitamin-A (c) Lemon, Orange, Tomato, Chilli,
Germinated seeds – Vitamin-C (d) Pulse, Liver, Vegetables,
Egg – Vitamin-B1
Ans. – (d) Pulse, Liver, Vegetables, Egg are sources of folic acid.
Wheat, Corn, Potato, Banana are sources of carbohydrate. Milk, Egg,
Paneer are sources of Vitamin-A and Lemon, Orange, Tomato, Chilli
are source of Vitamin-C.
Q5. Match List-I with List-II and select the correct answer by
using the codes given below:
List-I (Minerals)
(A) Sodium (B) Calcium (C) Magnesium (D) Phosphorous List-II
(Main Sources)
(1) Salt, Fish, Meat, Milk
(2) Paneer, Gram, Green Vegetables (3) Vegetables
(4) Milk, Milets, Paneer
Codes:

(A) Somatotropic Hormone (B) Triiodothyronine Hormone (C)


Parathyroid Hormone (D) Mineralocorticoids
List-II (Related Gland)
(1) Thyroid Gland
(2) Pituitary Gland
(3) Adrenal Gland
(4) Parathyroid Gland
Codes:

AB C D
(a) 1 2 3 4
(b) 2 3 4 1
(c) 3 4 1 2
(d) 4 1 2 3
Ans. – (a)
Sodium – Salt, Fish, Meat, Milk Calcium – Paneer, Gram, Green
Vegetables
Magnesium – Vegetables
Phosphorous – Milk, Milets, Paneer Q6. Which one of the following
antimicrobial drugs is suitable for treatment of both tuberculosis
and leprosy:
(a) Rifampicin
(b) Streptomycin
(c) P-aminosalicylic Acid
(d) Terramycin
Ans. – (a)
Leprosy is caused by Mycobacterium leprae, while tuberculosis is
caused by bacteria streptococcus or staphylococcus. Rifampicin is a
common medicine in treatment of tuberculosis and leprosy. B.C.G.
vaccine is used.
Q7. Match List-I with List-II and select the correct answer by
using the codes given below:
List-I (Hormone)

AB C D
(a) 1432
(b) 2341
(c) 2143
(d) 3421
Ans. – (c)
Somatotropic Hormone
– Pituitary Gland Triiodothyronine Hormone
– Thyroid Gland Parathyroid Hormone
– Parathyroid Gland Mineralocorticoids
– Adrenal Gland Q8. Match List-I with List-II and select the correct
answer by using the codes given below.
List-I List-II
(A) Mango (1) Oryza sativa (B) Rice (2) Mangifera indica (C) Wheat
(3) Brassica campestris (D) Mustard (4) Triticum aestivum Codes:
AB C D (a) 1234
(b) 2143
(c) 3412
(d) 4321
Ans. – (b)
Mango – Mangifera indica Rice – Oryza sativa
Wheat – Triticum aestivum Mustard – Brassica campestris Q9. The
pituitary gland by virus of its tropic hormones controls the
secretary activity of other endocrine glands. Which one of the
following endocrine gland can function independent of the
pituitary gland: (a) Parathyroid (b) Adrenals (c) Thyroid (d) None of
these Ans. – (a) Parathyroid gland presents in thyroid gland. This
gland secretes parathormone hormone, which regulates Ca++ and
PO4– ion in body. This gland works independently to pituitary gland.
Q10. Match List-I with List-II and select the correct answer by
using the codes given below:
List-I List-II
(Living organism) (Scientific name) (A) Man (1) Felis domestica (B)
Cow (2) Canis familaris (C) Dog (3) Bos indicus (D) Cat (4) Homo
Sapiens Codes:
AB C D
(a) 1234
(b) 2341
(c) 3412
(d) 4321
Ans. – (d)
Man – Homo Sapiens
Cow – Bos indicus
Dog – Canis familaris
Cat – Felis domestica
Q11. Match List-I with List-II and select the correct answer by
using the codes given below:
List-I List-II
(A) Frog (1) Reptilia (B) Lizard
(C) Peacock (D) Kangaroo (2) Amphibia (3) Mammalia (4) Aves
Codes: AB C D
(a) 1 2 3 4
(b) 2 1 4 3
(c) 3 4 2 1
(d) 4 3 1 2
Ans. – (b)
Frog – Amphibia
Lizard – Reptilia
Peacock – Aves
Kangaroo – Mammalia
Q12. Match List-I with List-II and select the correct answer by
using the codes given below:
List-I
(Plants Hormones) (A) Auxins
(B) Gibberellins (C) Cytokinins
(D) Absisis acid

List-II
(Discovery)
(1) Curasava
(2) Darwin
(3) Carns & Adikote (4) Miller

Codes: AB C D
(a) 1 2 3 4
(b) 2 1 4 3
(c) 3 4 2 1
(d) 4 3 1 2
Ans. – (b)
Auxins – Darvin
Gibberellins – Curasava
Cytokinins – Miller
Absisis acid – Carns & Adikote Q13. Fat present below the skin
surface in our
body, acts as a barrier against:
(a) Entry of harmful microorganism from the environment.
(b) Loss of salts from the body.
(c) Loss of heat from the body.
(d) Loss of essential body fluids. Ans. – (c) Fat present below the
skin surface in our body, is called subcutaneous fat depot. It acts as
insulator of body. It acts against loss of heat from the body. Q14.
Which of the following professionals are more likely to run the
risk of a permanent change in their cell’s DNA: (1) X-ray
technician
(2) Coal mines
(3) Researchers using carbon 14 isotope (4) Dyer and painter
(5) Train driver
Code:
(a) 1, 2, 3 and 4
(b) 1, 3 and 4
(c) 2, 3, 4 and 5
(d) 3, 4 and 5
Ans. – (b) Researchers using radioactive C14 isotope and X-ray
technicians are higher at risk of permanent change in the cell’s DNA.
Besides it dyers and painters are also at risk because of permanent
chemical change in DNA.
Q15. Match List-I with List-II and select the correct answer by
using the codes given below:
List-I (Physiological Processes) (A) Protein Synthesis
(B) Respiration
(C) Photosynthesis
(D) Mineral Uptake
List-II (Cell Organelles)
(1) Mitochondria
(2) Ribosomes
(3) Plasma membrane
(4) Chloroplast
Codes:
AB C D
(a) 1 2 3 4
(b) 2 1 4 3 (c) 3412
(d) 4123
Ans. – (b)
Protein Synthesis – Ribosomes
Respiration – Mitochondria Photosynthesis – Chloroplast Mineral
Uptake – Plasma Membrane Q16. Which of the following would
lead to malnutrition:
(1) Imbalanced nutrition
(2) Overnutrition
(3) Undernutrition
Codes:
(a) 1, 2 & 3 (b) 1 & 2
(c) 2 & 3 (d) 1 & 3
Ans. – (a) Malnutrition, in its broad sense, means bad nutrition. It can
be applied to both undernutrition and overnutrition and also for
imbalanced nutrition. Some examples of malnurition are – marasmus,
kwashiorkar, starvation and undereating. Q17. Consider the
following statements: AIDS is transmitted–
(1) By blood transfusion
(2) By sexual intercourse
(3) Across the placenta
(4) By mosquitoes and other blood sucking insects
(5) By infected needle
Code:
(a) 1, 2, 3, 4 & 5
(b) 2, 3, 4
(c) 1, 2, 3 & 5
(d) 3, 4 & 5
Ans. – (c) AIDS is Acquired Immune Deficiency Syndrome. It is
transmitted by sexual intercourse, blood transfusion, infected needle
and from pregnant mother to her baby across the placenta.
Q18. Which one of the following sets is correctly matched:

(1) Diphtheria, Pneumonia and

Leprosy – Hereditary
(2) AIDS, Syphilis and
Gonorrhoea – Bacterial
(3) Colour, Hemophilia – Sex linked (4) Polio, Japanese B
encephalitis and
Plague – Viral
Option:
(a) Only 1 (b) 2 & 3
(c) Only 4 (d) None of these Ans. – (c) Colour blindness and
Hemophilia are sex linked disease. Colour blindness is an inability to
perceive red and green colours. Hemophilia is an inability of body for
blood coagulation during injury. Q19. Which one of the following is
a membrane that protects the developing embryo from
dericcation:
(a) Yolk sac (b) Allantois (c) Both 1 & b (d) Amnion
Ans. – (d) Amnion is the extra-embryonic membrane formed in
Amniotes (Mammals, birds and reptiles). Amnion contains amniotic
fluid that protects growing embryo from dericcation and shock.
Q20. Most of the desert plants bloom during night time because:
(a) desert plants are sensitive to the
phases of moon.
(b) the desert insects are active during
night time.
(c) their blooming is controlled by low
temperature.
(d) both a and c
Ans. – (b) In desert condition, most of the activity of the plants and
animals happens during night because of very high temperature in
day time. The desert insects make themselves active and pollinate
the flowers at night. To attract the insects, most of the desert plants
bloom during night.
Q21. By weaving threads of physics, chemistry and biology into
a rich tapestry, this remarkable scientist provides a unifying
molecular view of the world. He discovered the wonderful
properties of Vitamin-C, first as a cure of common cold and later
as preventing agent against cancer. One of sciences major
figures of all time referred to above is: (a) Linus Carl Pauling
(b) G.N. Lewis
(c) Funk
(d) Carl Land Steiner
Ans. – (a) Linus Carl Pauling described properties of Vitamin-C which
is found in lemon abundantly. He used this Vitamin in curing common
cold. He also described its properties in cancer treatment.
Q22. Which of the following elements are present in all proteins:
(1) Oxygen (O2)
(2) Nitrogen (N2)
(3) Ammonia (NH3)
(4) Hydrogen
(5) Carbon
(6) Fluorine
Codes:
(a) 1, 2, 3 & 4
(b) 1, 2, 4 & 5
(c) 2, 4, 5 & 6
(d) 1, 4, 5 & 6
Ans. – (b) All the proteins are formed of amino acid that contains
COOH and NH2 as functional group. In this way, all the four elements
– Carbon (C), Oxygen (O), Nitrogen (N) and Hydrogen are present in
all proteins.
Q23. It begins as a single cell and grows into a merciless
disease that claims millions of lives year after year. But
scientists are steadily unlocking its mysteries and the fight
against it, may now have reached a dramatic turning point. New
discoveries promise better therapies and hope in the war
against .... the disease referred into the above quotation is:
(a) AIDS (b) Cancer
(c) World Flue (d) Meningitis
Ans. – (b) Cancer is a disease in which cell divides abnormally
without stopping and results into tumour. The substance, which
caused cancer, is called carcinogenic.
Q24. The nutritional deficiency condition that needs to be given
top priority for remedial action in India today is:
(a) Xerophthalmia
(b) Pellagra
(c) Anemia
(d) Foot & Mouth disease
Ans. – (a) Xerophthalmia is caused by Vitamin-A deficiency. This
generally occurs in poored section of the society because often they
do not have adequate amount of fruits in diet. Anemia is caused by
deficiency of blood.

Q25. Match List-I with List-II and select the correct answer by
using the codes given below:
List-I
(A) Protein (B) Hormone (C) Vitamin (D) Enzyme

List-II
(1) Keratin
(2) Progesterone (3) Carotene (4) Pepsin

Codes: AB C D
(a) 1234
(b) 2341
(c) 3214
(d) 4321
Ans. – (a)
Protein –
Hormone –
Vitamin –
Enzyme – Keratin
Progesterone Carotene
Pepsin
Circulatory System and Nutrition

Q1. Consider the following statements and select the correct


answer from the code given below:
(1) The person with diabetes insipidus

feels thirsty.
(2) A person with diabetes insipidus
suffers from excess secretion of
vasopressin.
Code:
(a) Both 1 and 2 (b) Only 1
(c) Only 2 (d) Neither 1 nor 2 Ans. – (b) Vasopressin or antidiuretic
hormone is secreted by posterior pituitary gland. The deficiency of
vasopressin results in a disorder known as diabetes insipidus. The
main symptoms of diabetes insipidus are increase in thirst and
increase in urination. Diabetes is a hormonal disease.
Q2. Consider the following statements and select the correct
answer from the code given below:
(1) Cholera is a disease caused by
bacteria.
(2) Athlete’s foot is a diseases caused by
virus.
(3) Filaria is a disease caused by
Helminthus.
(4) Sleeping diseases is caused by
Protozoa.
Code:
(a) 1, 2, 3 and 4 (b) 2, 3 and 4 (c) 1, 3 and 4 (d) 3 and 4 Ans. – (b)
Athlete’s foot disease is caused by fungus in condition where skin is
left warm and moist for a long period. It results is itching and burning
sensation. Cholera is a caused by bacteria. Filaria is a disease
caused by Helminthus. Sleeping disease is caused by Protozoa.
Q.3 Consider the following statements and select the correct
answer from the code given below:
(1) Fatty acids should be a part of the
balanced human diet.
(2) The cell of the human body can not
synthesize with fatty acids.
Code:
(a) 1 and 2 are correct
(b) Only 1 is correct
(c) Only 2 is correct
(d) Neither 1 nor 2
Ans. – (b) A balanced diet is one which contain the correct proportion
and quality of virus nutrients such as fats, carbohydrates, proteins
etc. to maintain health. The body cells of all mammals including man,
can synthesize the various fatty acids except linolenic acid and
linolenic acid which can be obtained only from food. Q4. Consider
the following statements and select the correct answer from the
code given below:
(1) Lan Wilmut created the first cloned
sheep.
(2) Adam Osborne produced the first
portable computer.
(3) Polio vaccine was created by Levis
Pasteaure. Code:
(a) 1, 2 and 3 (c) 2 and 3 only (b) 1 and 2 only (d) 1 and 3 only Ans. –
(b) The first successful computer was built by an American called
Harmon Hollerith, 1988. The first personal computer was developed
in 1975 by Altair and Apple company developed the portable
computer in 1970 which was used by large number of people. Dolly,
the sheep, the worlds first cloned mammals was produced by Ian
Wilmut of Roslin Institutes in 1996. Polio vaccine was created by
Jonas E. Salk.
Q5. Consider the following statements and select the correct
answer from the code given below:
Assertion (A): Scientists can cut apart and paste together DNA
molecules at will, regardless of the source of the molecules. Reason
(R): DNA fragments can be manipulated using restriction
endonucleases and DNA ligases.
Code:
(a) Both A and R are true and R is the
correct explanation of A.
(b) Both A and R are true but R is not
correct explanation of A.
(c) A is true but R is false.
(d) A is false but R is true.
Ans. – (a) Both A and R are true and R is the correct explanation of
A. Restriction endonucleases and DNA ligases are used in
recombinant DNA technology and DNA cloning.
Q6. What is Cyberknife treatment of cancerous tumours:
(a) It delivers an extremely precise dose
of radiation.
(b) It is a robotic guided system. (c) It can map the spread of tumour
in the
body.
(d) It has higher accuracy.
Ans. – (c) Cyberknife is a non-invasive, sub millimeter accuracy
cancer treatment that can treat most cancers without the need for
surgery or nasty drugs and has no side effect. It has high resolution
CT Scan and PET Scan of your long so that the computer can use
the information to map the location of the tumour correctly which is
essential to radiosurgery.
Q7. Widespread resistance of Malarial parasite to drugs like
chloroquine has prompted attempts to develop a Malarial
vaccine to combat Malaria. Why is it difficult to develop an
effective Malaria vaccine.
(a) Man is only intermediate host and
not the definitive host.
(b) Man does not develop immunity to
Malaria during natural infection. (c) Vaccines can be developed only
against bacteria.
(d) Malaria is caused by different species
of plasmodium.
Ans. – (d) Malaria has caused by several species of plasmodium.
Malaria parasite uses many tools to infect humans. The parasite
genome is very plastic. They are more than 27,000 variants if one
variant of malaria is treated successfully then another variant rises
from the background. Q8. Consider the following condition of a
sick human body:
(1) Swollen lymph nodes
(2) Sweating at night
(3) Loss of memory
(4) Loss of Weight
(5) Uncontrolled growth of cells Which of these are symptoms of
AIDS? (a) 1, 2, 3, 4 and 5
(b) 2, 3, 4 and 5
(c) 1, 2, 3 and 4
(d) 1, 3, 4 and 5
Ans. – (c) AIDS is the disease of immune system in which immune
system weakness and body becomes prone to different types of
infections by pathogens. AIDS has symptoms like swollen lymph
nodes, sweating at night, loss of memory and loss of weight.
Uncontrolled growth of cells is symptom of cancer.
Q9. Consider the following statements and select the correct
answer from the code given below:
Assertion (A): Unsaturated fats are more reactive compared with the
saturated fats. Reason (R): Unsaturated fats have only single bonds
in their structure.

Code:
(a) Both A and R are individually true and R is the correct explanation
of A.

(b) Both A and R are true but R is not correct explanation of A.


(c) A is true but R is false.
(d) A is false but R is true.
Ans. – (c) Assertion is true but Reason is
not correct. Compound having double
bond in their structure are more unstable
compound in comparison to single bond
holder compounds. Unsaturated fats those
have double bonds in their structures are
more reactive than saturated fats.
Q10. With reference to the blood in a normal
person, which one of the following statements is incorrect?
(a) WBC provides immune system.
(b) The blood has more platelets than WBC.
(c) The number of leukocytes in the blood is often an indicator of
disease.
(d) Compared to arteries, veins are less numerous and hold less of
the body’s blood at any given time.
Ans. – (d) The number of blood platelets
per cubic mm in human blood is 3 lacs
while WBCs are 5000 cubic mm of blood
veins are as a complex as the arteries. Veins
and arteries both are types of blood vessels.
Arteries carry blood from heart to different
organs to heart At any given time in a
healthy human, the blood amount is same
in both, as the circulation of blood never
stops.
Q11. With reference to the latest developments in stem cell
research, consider the
following statements:
(1) The stem cells can regenerate themselves in vitro virtually forever.
(2) Indian research centres also created a few cell lines which can be
developed into many types of tissue.
Choose the correct answer.
Code:
(a) Only 1
(b) Only 2
(c) Both 1 and 2 are correct
(d) Both are wrong
Ans. – (c) Stem cells are undifferentiated cells either embryonic or
adult, which divided to produce one stem cell and another which can
pass along a specific differentiation pathway in response to local
singles. Human embryonic stem cells are derived from 5 days old
human embryos, a process which destroys them.
Q12. Consider the following statements and select the correct
answer from the code given below:
Assertion (A): Human diet should compulsorily contains glycine,
serine and tyrosine.
Reason (R): Essential amino acids can not be synthesized in the
human body. Code:
(a) Both A and R are true and R is the correct explanation of A.
(b) Both A and R are true but R is not correct explanation of A.
(c) A is true but R is false.
(d) A is false but R is true.
Ans. – (d) Assertion is false but reason is
true. Essential amino acids are those which
are taken from food and not synthesized in
the body whereas non-essential amino
acids not be supplied in the diet and synthesized in the body. Glycine,
serine and
tyrosine are non-essential amino acids. Q13. Match List-I with List-II
and select the
correct answer by using the codes given
below.
List-I (Scientists)
(A) Temin and Baltimore
(B) Mullis
(C) Arber and Smith
(D) Feldman

List-II (Achievement)

(1) Developed transgenic plants with agrobacterium


(2) Discovered endonucleases

(3) Discovered polymerase chain reaction (4) Discovered reverse


transcriptase Codes:

AB C D (a) 1 2 3 4
(b) 2 3 4 1
(c) 3 4 1 2
(d) 4 3 2 1
Ans. – (d)
Temin and Baltimore – Discovered

Mullis
Arber and Smith
Feldman reverse
transcriptase

– Discovered
polymerase chain reaction
– Discovered

endonucleases
– Developed
transgenic plants
with
agrobacterium Q14. Consider the following statements and
choose the correct answer using the codes given below:
(1) Guinea-worm is an annelid.
(2) Filaria is caused by a nematode. (3) Roundworm has separate
sexes. (4) Tapeworm is not a hermaphrodite. Code:
(a) 1, 2, 3 and 4
(b) 2, 3 and 4
(c) Only 1 and 2
(d) Only 2 and 3
Ans. – (d) All tapeworms are hermaphrodite. Roundworm is
dioecious or unisexual i.e., having separate sexes. Filariasis is
caused by blockage of lymph channels by Wuchereria bancrofti, a
small parasitic nematode worm. Guineaworm is a nematode.

Q15. Match List-I with List-II and select the correct answer by
using the codes given below.
List-I
(A) Shoulder blade (B) Knee-cap
(C) Breast-bone (D) Collar-bone

List-II
(1) Patella (2) Scapulla (3) Clavicle (4) Sternum

Codes: AB C D
(a) 1234
(b) 2143
(c) 3421
(d) 4312
Ans. – (b)
Shoulder Blade
Knee-cap
Breast-bone
Collar-bone
– Scapulla
– Patella
– Sternum
– Clavicle Q16. Which of the following cell organelles play most
significant role in protein synthesis:
(a) Endoplasmic reticulum and ribosome. (b) Golgi apparatus and
lysosome. (c) Mytochondria, lysosome and centrosome.
(d) Centrosome, Golgi apparatus and lysosome.
Ans. – (a) Endoplasmic reticulum is an irregular network of double
membrane and ribosome are small granules either scattered in
cytoplasm or attached to the endoplasmic reticulum. Endoplasmic
reticulum and ribosome are play most significant role in protein
synthesis.
Q17. With reference to the work of human kidney consider the
following statements:
(1) From Bowman’s capsule, the filtered liquid passes through tiny
tubes where much of the glucose is reabsorbed and sent back to the
blood in the renal vein. (2) After the waste is removed in the kidney
the clean blood is sent back through renel artery.
Which of these statements is/are correct?
(a) Only 1 is correct
(b) Only 2 is correct
(c) Both 1 and 2 is correct
(d) Neither 1 nor 2
Ans. – (a) The nephron, with its pressure filter and trouble, is
intimately associated with the blood circulation. Blood from the aorta
enters each kidney through a large renal artery, which divides into a
branching system of smaller arteries.
Q18. With reference to normal human beings, consider the
following statements: (1) Unsaturated fats have only single bonds
in their structure.
(2) In response to the presence of HCl, secretin is produced from the
duodenum.
(3) Enterogastrone is produced in the large intestine in response to
the presence of fatty acids.
Which of these statements is/are correct?
(a) 1, 2 and 3 (b) Only 2
(c) Both 1 and 2 (d) Both 2 and 3 Ans. – (b) The medium of digestion
in stomach is acidic due to presence of HCl. When the food enters in
duodenum, secretion is produced by small intestine that stimulates
pancreas for pancreatic juice secretion. Unsaturated fats those have
double bonds in their structures are a reaction than saturated fats.
Q19. Assertion (A): Person suffering from hemophilia fails to
produce blood clotting factor VIII.
Reason (R): Prothrombin producing platelets in such persons are
found in very low concentration.

Codes:

(a) Both A and R are true and R is the correct explanation of A.


(b) Both A and R are true but R is not the correct explanation of A.
(c) A is true, but R is false.

(d) A is false, but R is true.


Ans. – (b) It is a hereditary disease. In which blood fails to clot due to
absence of factor-VIII.
It is of three types – (i) Heamo–A (ii) Heamo-B (iii) Christmas
disease. Prothrombin producing platelets in such person are not
found or occur in very low concentration.

Q20. Consider the following statements about Probiotic food:


(1) Probiotic food contains live bacteria

which are considered beneficial to humans.

(2) Probiotic food helps in maintaining gut flora.


Which of the statements given above is/are
correct?
(a) 1 only
(b) 2 only
(c) Both 1 and 2
(d) Neither 1 nor 2
Ans. – (c) Probiotic food is food that contains live bacteria which is
considered beneficial and not harmful to humans. Billions
of bacteria inhabit the human digestion
system. These bacteria are referred to as
the gut flora. The gut flora is needed to
break down food remains that have not
been earlier in the digestive system and to
discourage harmful bacteria and yeasts
from invading the body.
Q21. Assertion (A): All the proteins in our food
are digested in small intestine only.
Reason (R): The protein-digesting enzyme
from pancreas are released into small intestine.
(a) A and R are true and R is correct explanation of A.
(b) A and R are true but R is not correct explanation of A.
(c) A is true but R is false.
(d) A is false but R is true.
Ans. – (d) Protein digestion occurs in
stomach as well as in small intestine but
most protein digestion occurs in the upper
small intestine, in the duodenum and
jerunum under the influence of proteolytic
enzymes form pancreatic secretion immediately on entering the small
intestine from
the stomach.
Q22. Proteins are called building block of the
body whose primary function include:
(1) Defence as it produces
(2) Hormonal control
(3) Action on metabolic activities
(4) Repairing of damage
Choose correct statements.
(a) 1, 3 and 4
(b) 2 and 3
(c) 1 and 4
(d) 1, 2, 3 and 4
Ans. – (a) These are the basic function of
protein–
(1) It gives structural support to body.
(2) It forms antibodies.
(3) It repairs the damaged part of the body.
(4) It also helps in hormonal secretions, but this secretion is not
controlled by the protein.
Q23. Assertion (A): DNA finger printing has
become a powerful tool to establish paternity and identification of
criminals in rape
and assault cases.
Reason (R): Trace evidences such as hairs,
saliva and dried semen are adequate for
DNA analysis.
Codes:

(a) Both A and R are true and R is the correct explanation of A.


(b) Both A and R are true but R is not a correct explanation of A.
(c) A is true but R is false.
(d) A is false but R is true.
Ans. – (a) DNA finger printing has
become a powerful tool due to its ability in
tracing evidences of crime and to establish
the paternity. These evidences can be collected from hairs, semen
and saliva. Q24. Which one of the following will helps in
controlling the mosquitoes in ponds:
(a) Dogish (b) Gambusia fish
(c) Langur (d) Orangutan
Ans. – (b) Gambusia affins, approximately named ‘Mosquito Fish’ is
a small, guppy
like fish ranging in size from approximately 3/4 to 1 1/2 inches long.
As its name
implies, it is a natural predator of mosquito larva. Gambusia affins is a
voracious
predator.
Q25. Some species of plants are insectivorous:
(a) They grow in shady and dark places due to this photosynthesis
process does not occur sufficiently.
(b) They are adapted to grow in nitrogen deficient soils and thus
depend on insects for sufficient nitrogenous nutrition.
(c) They cannot synthesize certain vitamins themselves and depend
on the insects digested by them.
Choose the correct option.
(1) a, b and c (2) a and b
(3) Only b (d) None of these
Ans. – (3) An insectivorous plant has carnivorous character, captures
prey items,
such as insect, spiders protozoa as a nitrogen source. Many
insectivorous plants live
in fresh water, where nitrogen is not present in enough amount
because the pH of
the water is extremely acid.

Plants and Animals

Q1. Which of the following flowering plants completes its life


cycle every two year: (a) Dahlia (b) Sunflower
(c) Carrots (d) Mustard
Ans. – (c) On the basis of duration of their life cycle, flowering plants
are classified as annual (which life commonly 1 year like, Mustard,
Sunflower) bienenial (2 year) like Carrot, Cabbage) and perenial
(which has life three year or several year as like ginger dahlia.

Q2. Which of the following is called the ‘police force’ of human


body:
(a) Thrombocytes (c) Erythrocytes (b) Fibrinogen (d) Leucocytes

Ans. – (d) The nucleated white blood cells numbering about 9000 per
cubic millimeter in shape and contain no pigment. They are known as
‘police force’ of the human body. They protect from disease producing
organism. So in the given option (d) is ‘police force’ of the human
body.
Q3. Consider the following statement and select the correct
code. It is related to adipose tissues:
(1) The red bone marrow is also

explained as an example of adipore. (2) It is a connective tissue.


(3) It stores most of the fat in animal

bodies.
(a) 1 and 2 are correct
(b) Only 3 is correct
(c) Only 2 is correct
(d) 2 and 3 are correct
Ans. – (d) Adipose tissue is connective tissue modified for storage of
fat. The cells are commonly large and are rounded or oval in shape
with prominent central vacuoles that push the cytoplasm and nuclei to
the periphery of the cell.

Q4. Fibers of commercial importance derived from coconut and


jute fall under which of the following types of tissue: (a) Phloem
(b) Parenchyma (c) Collenchyma (d) None of these Ans. – (d)
Sclerae nechyma consist of long, narrow, fibre like cell, pointed at
both ends and with thick lined wall that gradually lose protoplasm and
so are dead when fully formed. Its cells are also used in patches and
provide mechanical strength.

Q5. Under which of the following categories does microscopic


organism Euglena fall: (a) Prostista (b) Plant
(c) Animal (d) All of the above Ans. – (a) Euglena is found in ponds
and is very prominent in the summer months, giving a characteristic
green fringe to the water. It has chloroplast with help of which it
manufactures food, very much like autotrophic plants. It moves about
free in animals.

Q6. Given below are the examples of true and false fruits:
(1) Mango (2) Apple
(3) Pears (4) Strawberry (5) Pod of pea plants
Which among the above are true fruits. (a) Mango and pod of pea
plants. (b) Mango, apple, pear.
(c) Mango, pear and pod of plants. (d) Apple, strawberry and pear.
Ans. – (a) A true fruit is transformed and matured ovary. No other
part of flower take part in its formation. A false fruit if any part of the
flower other than the ovary has contributed toward its formation.

Q7. Match the following with List-I and List-II:


(A) Vitamin ‘D’ (1) Hennyman (B) Homeopathy (2) Hopkins (C) Sex
Hormone (3) Stenotch (D) DNA (4) James Watson

and Crick Codes:

AB C D (a) 1 2
(b) 2 1
(c) 4 2
(d) 3 2
Ans. – (b)
Vitamin ‘D’ – Homeopathy – Sex Hormone – DNA – 3 4 3 4 3 1 4 1

Hopkins
Hennyman
Stenotch
James Watson and Crick

Q8. Match the following List-I with List-II and select correct
code:
Disease Virus
(A) AIDS (1) HIV (B) Dengue (2) Arabian Virus (C) Polio (3) Polio (D)
Influenza (4) ABC Codes:

AB C D
(a) 1 2 3 4
(b) 2 3 1 4
(c) 4 3 1 2
(d) 2 4 3 1
Ans. – (a)
AIDS – HIV
Dengue – Arabian virus Polio – Polio
Influenza – ABC
Q9. Consider the following statements and select correct code:
(1) In the leaves of Dictyocaulus, both sides of the leaves are almost
equally exposed to light, the stomata are evenly distributed on both
sides. (2) In the leaves of monocyteledon, there are distinct upper
and lower sides the stomata are confined to lower side only.
Code:
(a) 1 and 2 are incorrect
(b) Only 1 is correct
(c) Both 1 and 2 are correct
(d) None of these
Ans. – (a) It is in the leaves of dictohydeons that there distinct upper
and lower side. Where stomatas are confined to lower side only.
Q10. Match the following with their scientific name and select
correct code:
(A) Man (1) Rana tigrina
(B) Cow (2) Homo spania
(C) Cat (3) Felis domestica (D) Frog (4) Bos indicus
Codes:
AB C D
(a) 2431
(b) 1234
(c) 3241
(d) 4231
Ans. – (a)
Man – Homo spania
Frog – Rana tigrina
Cow – Bos indicus
Cat – Felis domestica Q11. Match the following and select correct
code:
(A) Vitamin
(B) Vitamin ‘A’ (C) Vitamin ‘B’ (D) Vitamin ‘C’ (1) Macular (2) Macular
(3) Funk
(4) Holcutt Codes: AB C D (a) 1234 (b) 3214 (c) 4213 (d) 1432 Ans.
– (b) Vitamin
Vitamin ‘A’ Vitamin ‘B’ Vitamin ‘C’
– Funk
– Maculur
– Maculur
– Holcutt Q12. Given below are some modification of roots for the
purpose of storage food. Match the following with their suitable
example and select correct code: Example
(A) Beet root
(B) Mango Ginger (C) Dahlia
(D) Carrot

Modification (1) Conical


(2) Nudolose (3) Fasciculated (4) Napiform

Codes: AB C D
(a) 1234
(b) 4321
(c) 4231
(d) 4123
Ans. – (c)
Beet root – Napiform
Mango Ginger – Nudolore
Dahlia – Fesciculated
Carrot – Conical Q13. Which of the following types of plants
thrive on rocks:
(a) Ephemeral
(c) Xerophytes (b) Lithophytes (d) Mesophytes Ans. – (b) Some
semiparasistic plants like misfeltoe obtain raw material from their host
plants to manufacture their food. These raw material are obtained
through special adventitious roots called haustoria or sucking roots
that penetrate into tissue of the host plants.
Q14. Match the following with List 1 to List 2 and select correct
code:
List 1

(A) Inflorescence
(B) Bracts
(C) Hypogynies
(D) Phyllotaxy
List 2
(1) Modified leaves
(2) Cluster of flower born on single axis (3) Mode of arrangement of
leaves on a

steam
(4) Position of floral leaves on thalamus Codes:

AB C D
(a) 1 2 3 4
(b) 2 1 3 4
(c) 4 3 1 2
(d) 1 2 4 3
Ans. – (b)
A– 2
B– 1
C– 3
D– 4
Q15. Which of the following statements is incorrect:
(a) Wax is a complex lipid.
(b) Fat yield about twice as many calories as an equal weight of
carbohydrate and protein.
(c) A molecule of fat consists of one molecule of glycerol and three
mole cules of fatty acid.
(d) The plasma membrane and mitochon dria are chief components
of fats. Ans. – (a)
The lipid are classified simple and complex. Fat like coconut oil,
waxes and butter are common simple lipid. Complex lipid included
phosphoras lipid, steroids, glyco lipid, fat soluble vitamin A, D, E and
K. Q16. Consider the following statements and select correct
code:

(1) Lysosome contain digestive enzymes which may cause inter


cellular digestion of certain substances.

(2) During development and during pathological condition lysosomes


are also known to remove unwanted cell and tissues.

(3) Lysosomes is an organelle of plant and animal cell.


Code:
(a) 1 and 3 are correct
(b) 1 and 2 are correct
(c) Only 3 is correct
(d) 1, 2 and 3 are correct
Ans. – (d) Lysosomes may be involved in
destruction of cells and tissue during development e.g. loss of tadpole
tails. In certain
pathological condition they are also known
as remove and digest unwanted tissue and
cell. Lysosomes also called suicide bag
cell.
Q17. Which of the following organelles of the
cell is the site of the Krebs cycle:
(a) Mitochondria
(b) Chloroplast
(c) Ergastic substances
(d) None of these
Ans. – (a) Kreb’s cycle explains the conversation in presence of
oxygen, of substances form by break down of sugar, protein and fat,
to carbondioxide, water and
energy rich compounds. Mitochondria are
rich in fat, protein and enzyme are responsible for aerobic respiration.
It is the site of
Kreb’s cycle.
Q18. Consider the following statement and
select correct code:
(1) Some bacteria have a special type of chlorophyll that can even
absorb the longer wavelength in frared radiation.
(2) Chlorophyll is and iron containing protein.
(3) Chlorophyll occur in two form.

Code:
(a) 2 and 3 are correct
(b) 1 and 2 are correct
(c) 1 and 3 are correct
(d) 1, 2 and 3 are correct
Ans. – (c) Chlorophyll is a green colour pigment universally found in
trees and plants responsible for take/capturing solar energy of
photosynthesis. Its contain magnesium protein not iron and it occur in
two part chlorophyll-a and chlorophyll-b.

Q19. Which among following nitrogenous bases does RNA


contain:
(a) Guanine, Thymine
(b) Adenine, Thymine
(c) Cytosine, Uracil
(d) None of these
Ans. – (c) RNA contains 4 types of nitrogenous bases – cytosine,
uracil, Adenine, Guanine. So in the given option (c) cytosine, uracil
contains the RNA.

Q20. Which of the following are the two organelles of the cell
other than the nucleus that also have DNA in them: (a)
Mitochondria and Golgi bodies (b) Mitochondria and chloroplast (c)
Endoplasmic Reticulum and

Lysosome
(d) None of these
Ans. – (b) In Eukayotes, deoxy ribo nucleac acid (DNA) is packed in
chromosome found in the nucleus, whereas in prokaryotes, where
there is no nucleus, it exists as diffuse cell. Mitochondria and
chloroplast are two organelles have also DNA which is known as
extra chromosomal DNA.

Q21. Which of the following statements related to blood is not


correct:
(a) The red blood cells form the vast

majority of the blood.


(b) The matrix on which a blood clots is
produced from fibrinogen.
(c) Blood is a connective tissue
consisting 55 percent plasma and 45%
blood cell.
(d) Thrombocytes, the largest of the
blood cell is responsible for the
clotting of the blood when vessels are
damaged.
Ans. – (d) Thrombocytes is the smallest of blood cell which is
responsible for the clotting of the blood when vessel are damaged.
Fibrinogen is essential for the normal coagulation of blood. The
matrix which is a blood clot form is produced from it. Q22. Consider
the following statements and select correct code:
(1) Monocotyledonous plants have
a fibrous room system.
(2) In a young steam the epidermis has
numerous opening called stomata. Code:
(a) 1 and 2 are not correct
(b) Only 2 is correct
(c) Only 1 is correct
(d) 1 and 2 both are correct
Ans. – (d) In a young stem the epidermis also have numerous
opening called stomata which are similar to those present on the
surface of a leaf. However in an older steam the stomata are
replaced by structure called lentical.

Q23. Consider the following statements and select correct


answer:
(1) Commonly, the steam of dicotyledon

are branches while those of monocotyledon are unbranched. (2)


Dicotyledonous are plants that have

leaves with parallel venation while monocotyledons have leaves with


reticulate venation.

Code:
(a) 1 and 2 are correct
(b) Only 2 is correct
(c) Only 1 is correct
(d) Both are incorrect
Ans. – (c) Dicotyledonous plants have leaves with reticulate venation
monocotyledons have leaves with the parallel venation so only
statement 1 is correct.
Q24. Which of the following is a herb : (a) China rose (b) Jasmine
(c) Sunflower (d) Lotus Ans. – (c) Herbs are plants with soft steam,
which grow to a height of two to four feet and complete a life cycle
within a year, producing, flower, seeds and fruits e.g. mustard and
sunflower. So in the given option sunflower is an example of herb.

Science and Technology

Q1. Consider the following statements:

(1) Calcitonin regulates the metabolism of calcium.


(2) Oxytocin stimulates contraction of uterine muscles during birth.
(3) ADH stimulates absorption of water and increase the urine
production.
(4) Grave’s disease is caused by malfunctioning of Adrenal gland.
Which of the following is/are correct:
(a) 1 and 2 (b) 2 and 3
(c) 1 and 3 (d) All of the above
Ans. – (a) Calcitonin is a hormone known to
participate in calcium and phosphorus metabolism. In mammals the
major source of calcitonin is from the parafollicular or C cells in
thyroid gland. Main function of oxytocin in
the body are contraction of womb (uterus)
during child birth and location.
Grave disease is causes by excess secretion
of Thyroid hormone.
ADH – It increases the reabsorption of
water in the distal convoluted tubule, collecting tubules and collecting
ducts of
nephrons of the kidneys.
Q2. With reference of bacteria which of the
following statements is correct about
sporulation:
(a) Sporulation is a process of
decay cause by bacteria.
(b) It is spoilage of food due to bacteria.
(c) It means tiding over unfavourable condition in their growth
medium.
(d) It is as sexual reproduction among bacteria through spores.
Ans. – (c) Sporulation process in bacteria
is a means of tiding over unfavourable condition in their growth
medium when nutrients substances in the medium are exhausted and
when condition become
unfavourable the protoplasm of the bacteria cell contract and
condenses into the spherical which later surrendered itself with thick
protective wall. This is called an endospore.
Q3. Consider the following statements regarding stem cells:
(1) Stem cells are unspecialised. (2) It is capable of dividing and
renewing. (3) It can give rise to specialised cells. Select the correct
answer from the codes given below:
(a) 1, 2 and 3 (b) 1 and 2
(c) 2 and 3 (d) 1 and 3
Ans. – (a) One of the fundamentals properties of a stem cell is that it
does not have any tissue specific structure that allow it to perform
specialised functions. However, unspecialised stem cells can give
rise to specialised cells, including heart muscle cells, blood cells or
nerve cells when unspecialised stem cells give rise to specialised
cell, this is called differentiation. Q4. Consider the following
statements and select the correct code:
(1) Viruses can be crystallized.
(2) A virus can multiply only a living cell of plant or an animal.
(3) Outside the host, viruses are surrounded by lipid coat.
(a) 1 and 2 are correct
(b) Only 2 and 3 are correct
(c) 1, 2 and 3 are correct
(d) All are incorrect
Ans. – (a) Outside the host, viruses are surrounded by lipid coat is
not correct, it exist an as inactive particle consisting a core of RNA or
DNA surrounded by protein coat. Viruses are extremely small (25 to
300 nanometer length) obligate which can multiply only in a living cell
and an animal.

Q5. Given below are many types of bacteria according to their


shape. Match them with the disease that they cause and select
correct code. (A) Tetanus
(B) Pneumonia (C) Cholera
(D) Syphilis
(1) Commas
(2) Spirilla
(3) Bacilli
(4) Diplococcus

Codes: AB C D
(a) 1234
(b) 3412
(c) 4231
(d) 4312
Ans. – (b)
Tetanus
Pneumonia
Cholera
Syphilis
– Bacilli
– Diplococcus
– Commas
– Spirilla
Q6. Match the following List-1 and List-2: List-1
(Element)
(A) Hg (Mercury) (B) Pb (Lead)
(C) Cd (Cadmium) (D) HF (Fluoride)

List-2
(Disease) 1. Knock-knee

2. Anemia
3. Minamata
4. Itai-itai
Codes: AB C D (a) 3412 (b) 3241 (c) 1234 (d) 1243 Ans. – (b)
Mercury – Cadmium – Lead – Fluoride – Minamata Itai-Itai
Anemia
Knock-knee Q7. Which of the following related to bacteria is not
correct?
(a) Bacteria do not have an unclear membranes.
(b) Bacteria have a single chromosome. (c) Bacteria are a cellular
organism. (d) Bacteria have a cellulose wall. Ans. – (d) Bacteria do
not have a cellulose was unlike plants cells. Their wall are made of
proteinaceous and fatty substances and have single chromosome is
not enclosed in a nuclear membranes. Granules of glycogen, fat and
other food reserves may be present.
Q8. Match the following List-1 and List-2: List-1
(Biological Entity)
(A) Sterile hybrid breed of male donkey and female horse
(B) Reciprocal cross hybrid of male horse and female donkey
(C) Young male fowl
(D) Mature male fowl
List-2
1. Rooster 2. Cockerel
3. Mule 4. Hinny
Codes:
AB C D
(a) 3 4 2 1
(b) 4 3 1 2
(c) 2 1 3 4
(d) 1 3 4 2
Ans. – (a)
Sterile hybrid breed of male donkey and female horse – Mule
Reciprocal cross hybrid of male horse and female donkey – Hinny
Young male fowl – Cockerel Mature male fowl – Rooster
Q9. Which of the following is not correct: (a) Some bacteria and
fungi are autotrophic while most tetrotrophic in nature.
(b) The total weight of bacteria has been estimated to be as much as
20 times the total weight of all living organism put together.
(c) Bacteria and fungi belongs to the phylum.
(d) Fungi and bacteria are flowerless plants.
Ans. – (a) A few species of bacteria are autotrophic and most
bacteria are heterotrophic. They manufacture their own food either by
using the energy they obtain from the oxidation of inorganic
compound or by using the energy absorbed from sunlight. All fungi
are heterotrophic in nature. Q10. Which one of the following
statements is not correct:
(a) Auxins are produced at the region of elongation.
(b) Auxins are most important plants hormones.
(c) Auxins are also important in regulating the fall of leaves and fruits.
(d) Indoleacetic acid is a principal Auxin. Ans. – (a) Auxins are
produced by growing tips of root and stem. From where they migrate
to the regions of elongation where they are needed for the process of
clongation.
Q11. Some artificial substances are used in place of sugars to
impart sweet taste. Saccharine is one of them:
(1) It is 550 times sweeter than cane sugar.
(2) It is obtained by oxidation of benzene (C6H6)
(3) It is an amine group product. (4) It has zero calorific value.
Codes:
(a) 1 and 2 (b) 1 and 4
(c) 2 and 3 (d) All of the above Ans. – (b) Saccharine is an artificial
sweetener. It is about 550 times sweeter than cane sugar. It has no
calorific value. It is obtained by oxidation of Toluene and is chemically
(C6H4SO2CONH).
It is amide not amene.
Q12. Which one of the following statements explain
parthenocropy:
(a) It is a fruit, which has multiple seeds. (b) It is a fruit which contains
single seeds.
(c) It is a development of fruit without fertilization.
(d) It is a fruit, which result from multiple fertilizations.
Ans. – (c) The seedless variety of fruits such as grapes, banana,
pineapple are partheno carpaus fruit, which has gained world
popularity. Plant breeders are trying to induce parthenocrapy in more
and more variety of fruits.
Q13. Assertion (A): Super ovulation and embryo transplantation are
used for cattle improvement.
Reason (R): Gonadotrophin injection release more than one ova in
high yielding cows. These are fertilized by artificial insemination with
sperms from a pedigree bull. Early embryos are transplanted into
surrogate mothers for development. Codes:
(a) Both A and R are true, R is correct explanation of A.
(b) Both A and R are true, R is not correct explanation of A.
(c) A is true and R is false.
(d) A is false but R is true.
Ans. – (a)
Super Ovulation – It means 6-8 eggs are produced per cycle instead
of a single egg. Embryo transplant – A developing embryo is
removed from pregnant animal at any early stage before the
embryo’s cells have had time to become specialised. Artificial
insemination – In this process semen collected from a supermale
parent is injected into the reproductive tract of the selected female
parent by the breeder. Q14. Phollodes in Australian Acacia is a
modification of which of the following: (b) Whole leaf (d) Stipules
(a) Auxillary bud (c) Petiole
Ans. – (c) Phyllodes in Australian Acacia is a modification of petiole.
In Australian Acacia normal compound leaves are found in seedling
and young plants. They soon fall of and petiole developed in flatted
structure. These look live leaves and are called phlodenes. They
carry on the normal function of the leaves.
Q15. What is the difference between CAT SCAN and MRI?
(1) CAT Scan X-rays, MRI uses no X-ray and thus no ionizing
radiation. (2) CAT Scan gives best results for soft tissues while MRI
gives best result for bones.
(a) 1 and 2 (b) Only 1
(c) Only 2 (d) None of these Ans. – (b) MRI does not do a very good
job with bones. MRI is also not advised to the people, who have an
artificial pacemaker implanted as the magnetic/radiowaves may
interfere. CT – It is a medical imaging method employing tomography.
Digital geometry processing is used to generate a 3D image of the
inside of an object from a large series of 2D X-ray image taken
around a single axis of rotation.
(i) CT Scan carries with it some risks due to use of ionizing radiation
from X-rays, MRI uses no ionizing radiation.
Q16. Given below are modifications of the leaf. Match them with
their examples and select the correct code:
(A) Nepenthes Khasiana (1) Bladder (B) Australian Acacia (C)
Utricularia
(2) Pitcher (3) Philode (D) Wild pea (4) Leaf tendril Codes:
AB C D
(a) 1 2 3 4
(b) 4 3 2 1
(c) 2 3 1 4
(d) 3 2 1 4
Ans. – (c)
Nepenthes Khasiana – Pitcher Australian Acacia – Philode Utricularia
– Bladder Wild Pea – Leaf tendril Q17. In addition to protein and
carbohydrates, other nutritional substances found in milk may
be:
(1) Cobalt (3) Iron
(2) Potassium (4) Calcium Select the correct answer.
(a) 1, 2 and 3 (b) 1, 2 and 4 (c) 2, 3 and 4 (d) 2, 3 and 4 Ans. – (c)
Milk is one of the most nutritious component of human diet. It
contains proteins, carbohydrates, sugar (lactose), fats, sodium,
potassium, calcium, iron, vitamins (A, B12, C and D), magnesium,
phosphorus, thiamin, riboflavin and niacin. It makes bones stronger.
Mother milk contain different types of nutrients. It is very necessary
for baby child.
Q18. Consider the following statements and select correct code:
(1) Eyes of potato marks its point of attachment to the plant.
(2) Turmeric plants and ginger are herbs but unlike herbs they live
through the years.
Code:
(a) Only 2 is correct
(b) Only 1 is correct
(c) Both 1 and 2 are incorrect
(d) Both 1 and 2 are correct
Ans. – (a) Turmeric and Ginger are common example of rhizomes.
During favourable condition aerial short developed from terminal bud
and established new plants, which in turn produce new rhizomes.
They live through the year. Eyes of potato are dormant bud.
Q19. Read the following passage:
An insect whose mouthparts are biting and chewing type in the larvae
condition, while they are siphoning type in the adult condition and this
insect gives an economically important substance during yet another
stage of its development. The insect is: (a) Anopheles (b) Bombyx (c)
Apis (d) Laccifer Ans. – (b) The caterpillar larvae of silkmoth
(Bombyx mori) are voracious feeder, so they have the continuous
supply of food. Each caterpillar larvae has a mandibulate type of
mouthparts adapted to feed easily on mulberry leaves, which adult
has siphoning type of mouthparts commercial silk is obtained from
the Bombyx mori. Q20. Which one of the following statements is
correct about phylloclade?
(a) It is a modification of leaf.
(b) It is an aerial modification of stem. (c) It is a subaerial modification
of stem. (d) It is a modification of root.
Ans. – (b) It is an aerial modification of stem. Sometime, as in many
cactuses, the stem is flattened and green and carries out the function
of the leaf. Such a stem, adapted for manufacturing of food called
phyloclade. Phylloclade are adapted for life in dry and arid regions.
Q21. Which of the following is true about thorn?
(a) It is a modification of leaf.
(b) It is a modification of root.
(c) It is an aerial modification of stem. (d) It is an sub aerial
modification of stem.
Ans. – (c) It is an aerial modification of stem is a true statement
about thorn. The thorn is hard, straight, pointed and sharp structure. It
is a modified branch as it forms axis of leaf as in Duranta.
Q22. Consider the following statements regarding Algabal
bloom:
(1) It occurs in fresh water.
(2) It occurs in marine environment. (3) It is green in colour.
(4) Its concentration may reach millions of cells per milliliter.
Select the correct answers:
(a) 1 and 2
(b) Only 4
(c) All of the above
(d) 1, 2 and 3
Ans. – (c)
Algae bloom is a rapid increase or accumulation in the population of
Algae in aquatic system. It occurs in fresh water as well as marine
water. Its colour is green and its concentration may reach millions of
cells per milliliter.
Its colour also depends on algae species. Q23. Match the following
spices to the parts of plants and select correct code: Part of
Plant Spices
(A) Flower Bud (1) Colve (B) Stigma (2) Safforn (C) Gum (3) Turmeric
(D) Underground steam (4) Asafoitida Codes:
AB C D (a) 1234 (b) 1243 (c) 4321 (d) 4123 Ans. – (b)
Flower bud – Colve Stigma – Safforn Gum – Asafoitida Underground
steam – Turmeric Q24. Consider the following statements
regarding Gene Therapy:
(1) Introducing a new gene into the body. (2) Delivering the genes to
the site using inorganic nano particles.
(3) Use of stem cells.
Select the correct answer using codes: (a) Only 2 (b) Only 3
(c) 1 and 3 (d) All of the above Ans. – (d)
Gene therapy using several different approaches. These includes:
(1) Introducing a new gene into the body. (2) Delivering the genes to
the site using inorganic nano particles, through stem cells, unharm
virus etc.
Q25. Consider the following statement regarding mosquitoes:

(1) Mosquitoes themselves are immune to these viruses.


(2) Most of the world’s population lives in environment amenable to
mosquito growth and survival.
(3) No vaccine has been successfully developed against mosquito
borne diseases.
Which of the statements is/are correct?
(a) Only 1 (b) 2 and 3
(c) Only 3 (d) All of these
Ans. – (d) Mosquitoes themselves are
immune to the viruses they transmit.
Mosquitoes prefer warm and humid climate which ensures their
survival and
growth vaccination against mosquito borne
disease such as Malaria and Dengue have
not been successful, those against other
mosquitoes borne disease such as yellow
fever, Encephalitis have been successful.
Modern Technology

Q1. Consider the following statements regarding milk is correct:


(a) It is the best source of calcium and its

water content is as high as 85 percent. (b) All forms of milk lack


vitamin C. (c) It is an example of suspension. (d) It lacks iron, which is
essential for the

formation of red blood cells. Choose the correct codes:


(A) a, b and c (B) a and d
(C) Only a (d) b and c

Ans. – (B) Milk obtained from animals, which we normally consume


lacks vitamin C. Milk is a colloidal mixture. Human milk contains
vitamin C in adequate amount. It lacks iron and roughage, which is
essential for the formation of RBC. It is the best source of calcium
and its water content is as high as 85 percent.

Q2. Consider the following statements related to cervical cancer


is correct: (1) It is malignant neoplasm of the cervix

uterior cervical area.

(2) India has one of the largest numbers of cervical cancer patients in
the world.

(3) It is examined by paptest.

(4) Gardsil is the most common drug used to cure cervical cancer.
Choose the correct code:
(a) 1, 2 and 3 (b) Only 4
(c) 2 and 4 (d) 1 and 2
Ans. – (a) The cervix is lower part of
uterus the place where baby grow during
the pregnancy. Cervical cancer is caused
by virus called HPV. The virus spreads
through sexual contact. Most women body
are able to fight HPV infection but some
times virus head to cancer.
Gardasil and cervarix are vaccines for cervical cancer.

Q3. Consider the following statements related to reptile is


incorrect:
(a) The body is covered by a water proof

skin with dry epidermal scales. (b) Reptiles are Oviparous.


(c) The eardrums are absent in reptiles. (d) The heart is two
chambered with two

auricles and a single ventricle. Choose the option.


(A) c and d (B) a and b
(C) Only d (D) Only b
Ans. – (A) An animals that crawls or move on its belly (snake) or on
small short leg as lizard is called reptiles.
Reptile is any amniote tetra pod that can lay eggs on land i.e. neither
a mammal nor a bird. The body of reptiles is covered by a water proof
skin with dry epidermal scales. The heart is three chambered with two
auricles and a single ventricle.

Q4. Consider the following statements and select the correct


codes:
(1) A plant gains weight during

photosynthesis but loses weight during respiration.

(2) Respiration is a continuous process where as photosynthesis


occurs only during day time.

Code:
(a) Both 1 and 2 are correct
(b) Both 1 and 2 are incorrect
(c) Only 1 is correct
(d) Only 2 is correct
Ans. – (a) Photosynthesis is a process by which organism that
contain the pigment chlorophyll convert light energy into chemical
energy which can be stored in molecular bones of organic molecule.
During photosynthesis plant gains weight and in respiration losses
weight.
Respiration is a general process by which organism oxidize organic
molecules and derive energy (ATP) from the molecular bond that are
broken. It is opposite process of photosynthesis.

Q5. Consider the following statements related to photosynthesis


is correct:
(1) In photosynthesis process light energy

is converted into chemical energy. (2) The optimum temperature for


photosynthesis is about 30°C. (3) PGAL is formed during light phase.
(4) The energy needed for dark reaction is
provided by ATP.
Choose the correct code:
(a) 1, 2 and 4 (b) 2 and 3
(c) Only 2 (d) Only 1
Ans. – (a) Photosynthesis is a process by which organism that
contain the pigment chlorophyll convert light energy into chemical
energy. The wavelength of sunlight between (400 nm – 700 nm) are
the wavelength that are absorbed by chlorophyll and that drive
photosynthesis. The optimum temperature for photosynthesis is
about 30°C. PGAL is formed during dark phase. It is very reactive.
Q6. Which among the following parts of plants are involved in
gaseous exchange: (a) Lenticels (b) Stomata
(c) Vacuoles (d) Stomata & Lenticels Ans. – (d) Gas exchange is a
biological process through different gases are transferred in opposite
direction across a specialised respiratory surface. Gas exchange is a
process where by water vapour and oxygen leave and CO2 enter
plant leave. The gases move into and out of the plant through
specialised opening located along the lower surface of the leaves,
these openings called stomata are of optimum size, shape and
distribution.
Q7. Through which of the following phenomena explains why
over manuring with chemical fertilizers can sometime caused
the death of plant:
(a) Plasmolysis (b) Turgidity (c) Endosmosis (d) None of these Ans. –
(a) In a plasmolysed cell, the space between the cell wall and
protoplasm comes to be filled with the same hypertonic solution since
it can freely diffuse through the cell wall, which is permeable. If cells
remain plasmolysed for a long period of time, they eventually die from
dehydration.
Q8. Consider the following statements and select correct code:
(1) Root hair are unicellular.
(2) Absorption of water by plants through
its root system take place by osmosis. (3) Absorption of mineral salt
takes place
mainly by the process of diffusion. Code:
(a) 1 and 3 are correct
(b) 2 and 3 are correct
(c) 1 and 2 are correct
(d) All 1, 2 and 3 are correct
Ans. – (c) In some plants, absorption of certain mineral salts may
take place by the physical process of simple diffusion. The movement
of salt cell into the cell will continue so long as their concentration
inside the cell is lower than their concentration of soil.
Q9. Consider the following statements and select correct code:
(1) The neck of region is absent in fish. (2) The heart of fish is four-
chambered. (3) Eyelids are absent in fish.
Code:
(a) 1, 2 and 3 are correct
(b) Only 1 is correct
(c) 1 and 3 are correct
(d) All of the above are incorrect Ans. – (c)

Fish has not eyelids and the neck region is also absent in fish. The
heart of fish is not four-chambered but the heart of fish is two
chambered with a single auricle and single ventricle.

Q10. Through which one of the following process sea weeds


absorb iodine from sea water:
(a) Diffusion
(b) Active transport
(c) Turgidity
(d) All of the above
Ans. – (b) Active transport is involved in the movement of material
across the cell of membranes. Not only sea weeds but cell of the
thyroid gland absorbs and concentrate certain material from very
week solution. Cell membranes play an active role in physiological
process of active transport.

Q11. What is double toned milk?

(a) It contains 1.5% fat and 10% fatless solid.


(b) It contains 4% fat and 7% fatless solid.
(c) Fat is 2.5% and fatless solid is 9%.
(d) None of these.
Ans. – (a) Double toned milk contains
1.5% fat and 10% fatless solid but in toned
milk amount of fatless solid is equal to
pure milk and fat is in less amount.
Generally cream contains 45% fat and butter contain 15% of water
(max) and 99%
fat.
Q12. Which of the following tissue convert
sugar into insoluble starch and store
these up for future use:
(a) Collenchyma
(b) Sclerencenyma
(c) Parenchyma
(d) None of these

Ans. – (c) Parenchymatous cell that make up the cortex of root


contain leucoplast, which convert sugar into insoluble starch and
store these up for the future use. Collenchyma, sclerencenyma do not
convert sugar in insoluble starch.

Q13. Which of the following is regarding as the more important


factor in the ‘ascent of sap’:
(a) Force of imbibition
(b) Force of capillarity
(c) Root pressure
(d) None of these
Ans. – (d) Force of imbibition, force of capillarity or certain vital force
are said be responsible for ascent of sap, it must be noted that these
are hypothesis and not supported by evidence. Root pressure is
thought to be an important factor in the ‘ascent of the sap’.

Q14. Match the following lists:


Function
(A) Necessary for the formation of

protoplasm
(B) Necessary for the formation of protei (C) Necessary for the
formation of

cell wall
(D) Necessary for the formation of
chlorophyll
Element
(1) Potassium (2) Nitrogen
(3) Calcium (4) Magnesium Codes:
AB C D (a) 1234 (b) 2314 (c) 4213 (d) 2431 Ans. – (a)
Necessary for the formation of protoplasm – Potassium Necessary for
the formation of protein – Nitrogen Necessary for the formation of cell
wall – Calcium Necessary for the formation of chlorophyll –
Magnesium Q15. Match the following lists:
Examples
(A) Volvex
(B) Fucus
(C) Spirogyra
(D) Chlamydomonas
Types
(1) Colonial
(2) Filamentous
(3) Single celled
(4) Having leaf and stem like
resemblances
Codes:
AB C D
(a) 1423
(b) 4321
(c) 1234
(d) 3241
Ans. – (a)
Volvex – Colonial
Fucus – Having leaf and stem like resemblances Spirogyra –
Filaments
Chlamydomonas – Single celled Q16. Match the following lists:
(A) Phaeophyta (1) Red pigment (B) Cyanophyta (2) Green pigment
(C) Chlorophyta (3) Brown pigment (D) Rhodophyta (4) Blue-green
pigment Codes:
AB C D (a) 3421 (b) 1234 (c) 4231 (d) 4321

Ans. – (a)
Phaeophyta – Brown pigment Cyanophyta – Blue-green pigment
Chlorophyta – Green pigment Rhodophyta – Red pigment

Q17. Match the following lists:


Examples Types
(A) Chital (1) Monotremata (B) Kangaroo (2) Marsupialia (C) Duck-bill
Platypus (3) Placentalia Codes:

AB C
(a) 1 2 3
(b) 3 1 2
(c) 3 2 1
(d) 2 3 1
Ans. – (b)
Chital – Placentalia Kangaroo – Manotremata Duck-bill Platypus –
Marsupialia Q18. Match the following lists:
Virus Disease
(A) Alpha virus (1) Cervical cancer (B) Corona virus (2) SARS (C)
H5N1 (3) Swine flu (D) HPV (4) Bird flu (E) H1N1 (5) Chikungunya
Codes:
AB C D E (a) 5 2 4 1 3 (b) 1 2 3 4 5 (c) 5 4 3 2 1 (d) 4 3 2 1 5 Ans. –
(a)
Alpha virus – Chikungunya Corona virus – SARS
H5N1 – Bird flu HPV – Cervical cancer H1N1 – Swine flu Q19. Match
the following lists: (A) Brain Box
(B) Only movable bone in the Skull

(C) Immovable joint (D) Join brain to the spinal cord


(1) Mandible

(2) Foramen magnum


(3) Sutures

(4) Caranium Codes:

AB C D (a) 1 2 3
(b) 4 1 3
(c) 3 1 4
(d) 4 2 3
Ans. – (b)
Brain Box – Only movable bone in the Skull – Immovable joint – Joins
brain to the
spinal chord – 4 2 2 1

Caranium
Mandible Sutures
Foramen magnum

Q20.Consider the following statements regarding Algae is


correct:
(a) Algae are capable of photosynthesis. (b) Clostridium and azoto
bacter are

examples of blue-green Algae that are able to fix nitrogen.

(c) Algae is a group of primitive plants mostly aquatic.


(d) In Algae, there is no cell
differentiation into organs like root, steam and leaves.
Choose the correct codes:
(A) a, d and c
(B) b and d only
(C) a and d
(D) None of these
Ans. – (A) Azoto bacter and clostridium
are certain nitrogen fixing bacteria, which live free in the soil. They
convert atmosphere nitrogen into nitrogen that is required for growth
of plants.
Nostoc and anabena are species of bluegreen algae that are
commonly found in soil and water, which are capable of fixing of free
nitrogen. Algae are capable of photosynthesis.
Q21. Match the following List-1 and List-2: List-1 (Minerals)
(A) Sulphur
(B) Potassium
(C) Sodium
(D) Phosphorus
List-2 (Relevance)
(1) Constituent of hair and nails.
(2) Essential for intracellular osmosic pressure.
(3) Important constituents of body fluid.
(4) Essential for the formation of bones and teeth.
Codes:
AB C D
(a) 4321
(b) 1234
(c) 1243
(d) 4312
Ans. – (b)
Phosphorus

Sodium
Potassium

Sulphur
– Essential for the formation of bones and teeth

– Important
constituents of body fluid
– Intracellular osmotic pressure
– Constituents of hair and nails
Q22. Consider the following statements regarding amphibians:
(1) The heart of amphibians is three chambered.
(2) Amphibians are Oviparous.

(3) The skin of Amphibians can play a respiratory role.


(4) Fingers and toes with claws or nails are present in Amphibians.
Choose the correct options:

(a) 1 and 2 (b) 2 and 3


(c) Only 4 (d) All of the above Ans. – (c) Amphibians are cold
blooded animal that usually split their time in land and water. Ex.–
Frog, toad etc. Fingers and toes with claws or nails are present in
Amphibians.
Amphibians are an important component of global ecosystem as
indicator of environmental health. All Amphibians begin their life in
water with gills and tails.

Q23. Match the following List-1 and List-2: List-1 List-2


(A) Pure blood (1) Artery (B) Impure blood (2) Veins (C) Deficiency of
blood (3) Anemia (D) Blood clotting (4) Fibrinogen Codes:

AB C D
(a) 1234
(b) 1243
(c) 3412
(d) 3421
Ans. – (a)
Pure blood – Artery
Impure blood – Veins
Deficiency of blood – Anemia Blood clotting – Fibrinogen Q24.
Ralenza is used commonly drug to treat which one of the
following disease: (a) Swine flu (c) SARS
(b) Bird flu (d) Plague

Ans. – (a) SARS – It is called Severe Acute Respiratory Syndrome. It


is caused by corona virus.
Swine Flu – It is caused by H1N1. Ralenza and Tamiflue are drug
used to treat Swine Flu.
Ralenza has been approved for the treatment of uncomplicated acute
illness due to ment of uncomplicated acute illness due to years and
older who have been symptomatic for no more than two days.
Ralenza belongs to new class of compounds called Neuraminidase is
an enzyme.

Q25. Consider the following statements related to Chikungunya


is correct:
(1) Chikungunya virus is transmitted to

humans by Aedes Mosquito. (2) Real time polymerase chain reaction


test (RT-PCR) is a common test for
Chikungunya.
(3) Chikungunya virus is indigenous to
tropical Africa and Asia.
Choose the correct options.
(a) 1 and 2 (b) Only 3
(c) 1, 2 and 3 (d) Only 1
Ans. – (c) Chikungunya fever is a viral disease transmitted to
humans by the bite of infected Aedes Mosquitoes. Chikungunya virus
(Chik-V) is a member of genus α-virus in the family of togaviridae.
Chikungunya virus is indigenous to tropical Africa and Asia. It is
examined by test (RT-PCR)

Recent Medical Advancements

Q1. Consider the following statements and select correct codes:


(1) Cancer is characterised by

uncontrolled growth and spread of abnormal cells.

(2) Cancer cells and normal cells differ in their life spans.
(3) The only known immortal cells in the body are germ cells.
Codes:
(a) 1, 2 and 3 (b) 1 and 3
(c) Only 1 (d) 1 and 2
Ans. – (a) Normal cells become cancerous
when they mutate into abnormal cells
through a process called transformation,
leading to uncontrolled growth, eventually
forming a tumour of neoplasm. The only
known immortal cells in the body are germ
cells (sperm and egg) although they too
eventually die out if an individual never
has children.
Q2. Consider the following statements and
select the correct code:
(1) GM crops have terminator gene.
(2) Bt-Brinjal is the first genetically modified vegetable of the world.
(3) Bacillus thuringiensis (Bt) is a common bacteria occurring in the
soil which has been used as biopesticide sprays.
(4) Bt cotton was introduced in India in 2002.
Code:
(a) 1, 2 and 3 (b) 2, 3 and 4
(c) Only 4 (d) Only 3
Ans. – (b) GM crops do not have terminator gene. It is still in the
concept stage.
Bt produces a protein toxic to lepidopetron
insects pests which are major cause of concern. Bt-genes is isolated
from the bacteria
and are transferred to many crops globally. Bt cotton is introduced in
India in 2002. Q3. Which one of the following greenhouse gases
is emitted from swamps and tundra:
(a) CO2 (b) CH4
(c) N2O (d) HFC-23
Ans. – (Methane)
Methane (CH4) is called marshy gas. It is hydrocarbon. It is a
saturated compound. It is the first Alkane (CnH2n+2).
The sources of methane (CH4) are:
(1) Enteric fermentation in cattle and insects.
(2) Biomass burning.
(3) Coal mines and natural gas leaks.
(4) Paddies.
(5) Swamps.
Q4. Match the following and select correct
code:
Disease
(A) SARS
(B) AIDS

Test
(1) West Blot Test (2) Pap test

(C) Cervical Cancer (3) Elisha test (D) Swine Flu (4) RIDT Codes:

AB C D (a) 3124
(b) 4321
(c) 1234
(d) 3421
Ans. – (a)
SARS – AIDS – Cervical Cancer – Swine Flu – Elisha test
West blot test Pap test
RIDT
Q5. Which one of the following is a metabolic disease
characterized by high levels of uric acid in the blood:

(a) Mellitus (b) Gout


(c) Ascarisis (d) None of these Ans. – (b) Gout is a metabolic disease
characterized by high level of uric acid in the blood. The onset of the
first acute attack of gout is marked sudden and sever pain in a joint.
Within hours, the affected joint is hot, red, swollen and externally
tender.

Q6. Match the following and select correct code:


Development Process
(A) Ripening of fruit
(B) Cell division
(C) Inhibits all kind of developmental

activities
(D) Growth in length
Plant Hormone
(1) Abscissic Acid
(2) Ethylene
(3) Gibberellins
(4) Cytokinins
Codes:

(b) A healthy pancreas delivers insulin every 10 to 14 minutes.

(c) Smart insulin pump now available in India, mimics the pancreas
by delivering insulin effectively.

Choose the correct codes:


(1) a and b (2) b and c (3) All of the above (4) None of these Ans. –
(2) In type-1 diabetes the body fails to produce insulin and the patient
is required to inject insulin, whereas type-2 diabetes results from
insulin resistance a condition in which cells fail to use insulin properly.
Smart insulin pump now available in India is of greater help to control
type-1 and type-2 diabetes.

Q8. Match the following and select correct code:


Disease
(A) Bird flu (B) Autism (C) Dengue

Alternate name
(1) Break bone fever (2) Avian influenza (3) Hutchinson Gilford

(D) Progeria (4) Asperger’s Syndrome Codes:

AB C D (a) 2413
(b) 3241
(c) 4231
(d) 1432
Ans. – (a)
Ripening of fruit Cell division
Inhibits all kind of developmental activities
Growth in length
– Ethylene
– Cytokinins
– Abscissic Acid
– Gibberellins Q7. Which of the following statements is/are

correct:
(a) Type 1 diabetes result from insulin resistance, a condition in which
cells fail to use insulin properly.
AB C D
(a) 1 2 3 4
(b) 3 1 4 2
(c) 4 3 2 1
(d) 2 3 1 4
Ans. – (b)
Break bone fever – Dengue Avian influenza – Bird flu Hutchinson
Gilford – Progeria Asperger’s Syndrome – Autism Q9. Consider the
following statements regarding burning ice and select correct
code:
(1) Burning ice is found in high pressure and low temperature.
(2) It is chemically methane hydrate.
(3) Japan would become the first country to conduct an offshore
experiment to extract natural gas from deposits of burning ice.
(a) 1 and 2
(b) 3 only
(c) All of the above
(d) None of these
Ans. – (c) Japan would become the first
country to extract natural gas from deposits
of ‘burning ice’. Chemical name of burning ice is methyl hydrate. It is
found in high
pressure and low temperature area such as
ocean floors often near continental fault
lines where the gas crystallizes on contact
with cold sea water.
Q10. With references to communication
application, which one of the following is
technically called ‘white spaces’:
(a) It is the part of spectrum that works without causing any
interference to the primary users.
(b) It is the gap between two spectrums.
(c) It is the spectrum allocated to licensed users.
(d) It is the spectrum that has not been allocated to anyone.
Ans. – (a) The primary users are the
licensed users of spectrum band. They
often do not use the allocated spectrum to
the maximum. White spaces devices could
help find the spectrum’s under utilized
aspects and make the secondary users utilize them without causing
any interference
to the primary users. In microsoft,
researchers are working to harness the
enormous potential of white spaces to
deliver broadband wireless that help people stay better connected.
Q11. Match the following and select correct
code:
Drugs Purpose
(A) Gatifloxacin (1) Pain killer
(B) Tegaserod (2) Decongestant (C) Phenylpropanolamine

(3) Used for constipation

(D) Nimesulide (4) Antibiotic for treating respiratory tract infection

Codes:

AB C D
(a) 4321
(b) 1234
(c) 2314
(d) 4123
Ans. – (a)
Pain killer – Nimesulide
Decongestant – Phenylpropanolamine Used for constipation –
Tegaserod Antibiotic for treating respiratory tract infection –
Gatifloxacin Q12. Consider the following statements regarding
Higgs Boson:
(1) It is a theoretical particle which is believed to have given matter its
mass.
(2) It is also called God’s particle. (3) Large Haldron Collider (LHC)
recently created mini ‘Big Bang’ in the laboratory.
Codes:
(a) 1, 2 and 3 (b) Only 3
(c) 1 and 3 (d) None of these Ans. – (a) Higgs Boson particle was
first proposed by Peter Higgs of University of Edinburgh in 1964. The
LHC should confirm whether it exists or not. Researchers at LHC
recently succeeded in recreating the Big Bang on miniature scale.
Higgs Boson is a theoretical particle which is believed to have given
matter its mass. It is also called God’s particle.
Q13. Consider the following statements related to enriched
uranium:
(1) It is a kind of uranium in which the percent composition of U235 has
been increased through process of isotope separation.
(2) Generation any military nuclear weapons.
(3) It is critical component for both civil nuclear power generation and
military nuclear weapons.
(4) U238 is the only nuclide existing in nature that is fissile with thermal
neutrons.
Choose correct options:
(a) 1 and 2 (b) 2 and 4
(c) 1, 2 and 3 (d) All of the above Ans. – (c) U235 is the only nuclide
existing in nature that is fissile with thermal neutrons.
Enriched uranium is a kind of uranium in which the percent
composition of U235 has been increased through process of isotope
separation. It is critical component for both civil and military purposes.
Q14. Match the following and select correct code:
Type of Drug
(1) Hallucinogen (2) Stimulant
(3) Narcofics
(4) Sedative

Example
(A) Mescaline
(B) Caffeine
(C) Morphine
(D) Benzodiazephine
Codes: AB C D
(a) 1234
(b) 1243
(c) 4321
(d) 2341
Ans. – (b)
Hallucinogen
Stimulant
Narcotics
Sedative
– Mescaline
– Caffeine
– Benzodiazephine
– Morphine

regarding ‘Nuclear Reactor’:


(1) In a Nuclear Reactor, Moderator is used to slow down neutrons
emitted in a nuclear fission reaction.
(2) In a breeder reactor, molten sodium is used as a coolant.
(3) Excess amount of energy is released during nuclear fission.
Choose correct answers:
(a) 1 and 2 (b) 2 and 3
(c) 1, 2 and 3 (d) All of the above Ans. – (c)
In nuclear fission reaction heavy nucleus is
divided into simpler nuclear. In this
process huge amount of energy is released.
It is called Nuclear Energy.
In a breeder reactor, no moderator is used.
Molten sodium is used as coolant. The heat
generated in breeder reactor is several
times more than conventional reactor. Q16. Consider the following
statements and
select correct answer:
(1) Chandrayan-1 data provides support for the idea that solar wind is
made of mostly hydrogen ion.
(2) Solar wind compresses the magnetic field of the earth on its sun
facing side.
Codes:
(a) Both 1 and 2 are true
(b) Both 1 and 2 are untrue
(c) Only 1 is true
(d) Only 2 is true
Ans. – (a) The solar wind compress the
magnetic field of the earth on its sun facing
side and draws it out in the opposite side
like a tail. Solar wind consists elementary
particles such as protons and electrons in
roughly equal together with α-particle.
Chandrayan-1 data provide support for the
idea that solar wind is made of mostly

hydrogen ion interacts with oxygen in Q15. Consider the following


statements

lunar soil and rocks to produce water. Q17. Match the following
lists:
List-1 List-2
(A) Kinetic energy (1) M × a
(B) Potential energy (2) force x displacement
(C) Work (3) 1/2 mv2
(D) Force (4) mgh
Codes:
AB C D
(a) 1 2 3 4
(b) 4 3 1 2
(c) 3 4 2 1
(d) 2 3 4 1
Ans. – (b)
Kinetic energy – 1/2 mv2
Potential energy – mgh
Work – Force × displacement
Force – mass × acceleration Q18. If parent are of blood group AB
and O,
then the possible blood group of children
would be which among the following?
(a) A or B (b) A or O
(c) A, B or AB (d) O, A, B or AB
Ans. – (a) If parent are of blood group AB
and O, then the possible blood group of

children would be A or B. Genotype of blood group AB and O would


be AB or OO respectively. Children with blood group O and AB is not
possible in this case.

Q19. Which of the following is congenital abnormality in human


being due to trisomy of chromosome 21:

(a) Mongolism
(b) Colour Blindness
(c) Turner’s Syndrome
(d) Klinefelter Syndrome
Ans. – (a) Down Syndrome or Mongolism is congenital abnormality in
human being due to trisomy of chromosome 21 (Chromosome 21
occur in triplicate rather than in duplicate). The affected individual
therefore carry 47 chromosome and greatly retarded physically and
mentally.

Q20. Match the following and select correct code:


Chromosome Abnormalities
(1) XO (A) Turner Syndrome (2) XXX (B) Super female
(3) XXY (C) Super male
(4) XYY (D) Klinefelter’s Syndrome Codes:

AB C D
(a) 3421
(b) 1234
(c) 4132
(d) 4321
Ans. – (a)
Turner Syndrome – XXY Super female – XYY Super male – XXX
Klinefelter’s Syndrome – XO
Q21. Consider the following statements and select the correct
option:
(1) When a charge particle enters
perpendicular to direction of magnetic field then it moves on the
circular path.

(2) Lorentz force acts when charge particle moves in magnetic field.
(3) Magnetic field lines form closed curve.
(4) Electric field inside the conductor is zero.
Choose correct option:
(a) 1 and 2 (b) Only 4
(c) All of the above (d) 1 and 3
Ans. – (c)
Lorentz force acts when charge particle
moves in magnetic field
−−> −−>

Fmag = q(V × B) = qvBsinθ


Radius of circular path which is followed by charge

r = MV
qB
Magnetic field lines form close loop

Q22. Given below are the names of some human diseases.


Which disease only represents bacteria disease?
(1) Dengue (2) Cholera (3) Plague (4) Chicken pox (5) Yellow fever
(6) Measles

Code:
(a) 1, 2, 3, 4 and 5 (b) 4, 3, 5 and 6 (c) 2 and 3 (d) 5 and 6 Ans. – (c)
Viral disease which cause human sickness are smallpox, influenza,
yellow fever, measles, chicken pox, Hepatitis A, viral pneumonia,
AIDS, common cold, dengue etc. cholera and plague are bacterial
disease.

Q23. Match the following and select correct code:


Band Communication Frequencies

(1) K-Band (A) 18-27 GHZ (2) KV-Band (B) 1-2 GHZ (3) C-Band (C)
4-8 GHZ (4) L-Band (D) 12-18 GHZ Codes:
AB C D (a) 4312
(b) 1243
(c) 1432
(d) 1234
Ans. – (c)

K-Band – 18-27 GHZ KV-Band – 12-18 GHZ C-Band – 4-8 GHZ


L-Band – 1-2 GHZ

Q24. Which of the following is female sex hormone:


(a) Androgen (b) Oestrogen
(c) Adrinalin (d) Vasopressin Ans. – (b) Oestrogen is the female sex
hormone. Apart from producing Ova, the ovaries also produce female
sex hormone known as Oestrogen which are responsible for the
development of the secondary sexual character at the time of
puberty.

Q25. Consider the following statements and select the correct


option:
(1) For generic manufactures “the

product is the product”, while manufactures of innovator biologics


claim “the process is the product”.

(2) Biosimilar medicines are replica of biologic drugs.


Code:
(a) Both 1 and 2 are correct
(b) Both 1 and 2 are incorrect
(c) Only 1 is correct
(d) Only 2 is correct
Ans. – (c) Biosimilars are approved new
versions of innovator biologic drugs that
are developed after the innovator’s patient
expires. Unlike generies, however, biosimilar are not exact replicas of
biologic drugs
with the original biologic purification
process not available to biosimilar makers,
biosimilar can never be made in exactly the
same manner as the original biological
drug.

Cure and Precautions

Q1. Consider the following statements:

(1) Ribosomes are involved in protein synthesis.


(2) Mitochondria help in oxidative phosphorylation and generation of
ATP.
(3) ER helps in the transport of substance, synthesis of proteins.
Which of the statement given above are
correct about bioleading?
(a) 1 and 3 (b) 2 and 3
(c) 1, 2 and 3 (d) 1 and 2
Ans. – (c) The ER bearing ribosomes on
their surface, is actively involved in protein
synthesis, secretion and transport of substances.
Ribosomes are the site of protein synthesis.
Mitochondria are the sites of aerobic respiration. They produce
cellular energy in the
form of ATP so it is called power house of
the cell.
Q2. Consider the following statements
regarding Human Blood:
(1) Blood is a liquid connective tissue.
(2) Plasma and blood corpuscles are two main constituents of blood.
(3) Blood corpuscles is divided into three parts like RBC, WBC and
blood platelets.
(4) Life span of RBC is (20-120) days.
Choose the correct answer:
(a) 1 and 2
(b) 2 and 3
(c) All of the above
(d) Only 4
Ans. – (c) Blood is a liquid connective tissue. It is a basic solution
whose PH value
is 7.4. Two elements are found in human
blood (i) Plasma (ii) Blood Corpuscles.
Life span of RBC is (20-120) days. Its death occurs in the liver, so
liver is called graveyard of RBC.
RBC
Blood Corpuscles Blood corpuscles WBC
Blood Platelets Q3. Phytohormone is a naturally produced
organic substance in:
(a) Plants to control growth and physiological functions at the site of
its production.
(b) Higher plants which controls growth and development in general.
(c) Higher plants controlling growth and development at a site quite
removed from the site of its production and are effective when
present in large quantities.
Ans. – (c) Phytohormones is a naturally
produced organic substance in higher
plants controlling physiological and
growth function at a site quite removed
from the site of its production and is effective; when present even in
minute quantity.
They are organic substances and are capable of influencing
physiological activities
leading to promotion inhibition and modification of growth.
Q4. Consider the following statements
regarding Islets of Langerhans:
(1) It is a part of liver.
(2) It is discovered by Langerhans.
(3) Pancreas is the second largest gland in human body.
(4) Pancreas is endocrine and exocrine gland.
Choose the correct answer:
(a) All of the above (b) 2, 3 and 4
(c) 1 and 2 (d) 1 and 4 Ans. – (b)

Islets of Langerhans is the part of Pancreas. It is discovered by


Langerhans. It’s β-cell secrets insulin, α-cell secrets glucagon and γ-
cell secrets somatostatin hormone. Pancreas is endocrine and
exocrine gland. It was discovered by Waiting and West in 1921. It is
second largest gland in human body. Diabetes is caused by lack of
insulin.

Q.5 A lake near a village suffered heavy mortality of fishes.


Consider the reason for this:
(1) The area was sprayed with DDT by an

aircraft.
(2) Urea and phosphate fertilizers were
used in the crops in the vicinity. (3) The lake water turned green and
stinky.
(4) Phytoplankton in the lake declined
due to reduction in photosynthesis. Choose the correct options:
(a) 1 and 2 (b) 2 and 3 (c) All of the above (d) 1 and 3 Ans. – (a) A
lake near a village suffered heavy mortality of fishes within a few
days, because lots of urea and phosphate fertilizers were used in the
vicinity of crops and area was sprayed with DDT by an aircraft.
Q6. Match the following List-1 and List-2: List-1
(A) Botulism
(B) Anthrax
(C) Foot & mouth disease
(D) T.B.
List-2
1. Chlostridium
2. Mycobacterium Tuberculosis 3. Bacillus Anthrax11111
4. Virus
Codes:
AB C D (a) 1432 (b) 1 2 3 4
(c) 1 4 2 3
(d) 2 4 1 3
Ans. – (a)
Botulism – Chlostridium Anthrax – Virus
Foot and mouth
disease – Bacillus Anthrax T.B. – Mycobacterium Tuberculosis Q7.
Which of the following is not correct about yeast?
(a) Yeast in nature lives on the surface of
fruits and other similar condition. (b) Yeast cell reproduced by
budding. (c) In baking, yeast is added to uncooked
dough, which rise as result of oxygen
bubbles given off.
(d) None of these
Ans. – (c) In the given option (c) is not correct about yeast. In baking,
yeast is added to uncooked dough to made dough ‘rise’ as the result
of carbondioxides (CO2) bubble given off, before the bread is baked.
Q8. Who among the following propounded the germ theory of
disease:
(a) Typhoid
(b) Alexendar Fleming
(c) Hanse Bathe
(d) Louis Pasteur
Ans. – (d) The germs theory of disease given by Louis Pasteur. He
was the first to demonstrate experimentally that the fermentation is
not caused by chemical decay but by the action of certain micro
organisms. He discovered and identified several disease causing
micro organism and developed vaccines against some of them. Q9.
Match the following lists.
Chemical Name (A) Biotin
(B) Calcefrol (C) Tocopherol

Vitamin
(1) Vitamin H (2) Vitamin D (3) Vitamin E (D) Phylloquinone (4)
Vitamin K Codes:

AB C D
(a) 1 4 3 2
(b) 2 4 3 1
(c) 4 3 2 1
(d) 1 2 3 4
Ans. – (d)
Biotin – Vitamin H
Calcefrol – Vitamin D
Tocopherol – Vitamin E
Phylloquinone – Vitamin K
Q10. Yeast is an example of which of the following:
(a) Algae (b) Bacteria
(c) Fungi (d) All of these Ans. – (c) The common examples of fungi is
Mushroom, Yeast etc. moulds is also a common example of fungi so
in the given question yeast is an example of fungi. Fungi is
heterotrophic in nature not Autotrophic.
Q11. Which of the following types of root modification help
obtaining raw material from host plants:
(a) Stilt roots
(b) Haustoria roots
(c) Epythytic roots
(d) None of these
Ans. – (b) Few semiparasitic plants like mistletoe obtain raw
materials from their host plant to manufacture their food. These raw
materials are obtained through special adventitious roots known as
sucking roots or haustoria that penetrates into the tissues of the host
plant e.g. mistletoe.
Q12. Anthrax is a disease caused by which of the following:
(a) Fungi (b) Virus
(c) Algae (d) Bacteria
Ans. – (d) Anthrax is a disease caused by bacteria. It is a highly
infectious disease of animals, such as cattle, sheep caused by
bacterium. It could also spread to human beings. The disease is
characterized by inflammatory skin lesions and high rate of mortality.
Q13. Which of the following is not disease caused by fungi:
(a) Softret
(b) Wheat rust
(c) Ringworm
(d) Late Blight of Potato
Ans. – (a) Soft drink is a disease of plants, such as potato, tomato,
cabbage, cauliflower, cucumber caused by bacteria. The disease is
characterized by large scale of death cells resulting in the formation
of soft shining masses in edible part of these plants. The disease is
characterized by circular patches and severe itching.
Q14. Match the followings:
(A) Plant tolerating saline
condition (1) Halophytes (B) Plants adapted
to seasonal or perennial
drought
(C) Plants that derive water and nutrients from air.
(D) Deserts plants
that complete their cycle in very short period
(2) Xerophytes

(3) Epiphytes
(4) Ephemeral Codes:

AB C D (a) 1234
(b) 2134
(c) 4231
(d) 3241
Ans. – (a)
A– 1
B– 2
C– 3
D– 4
Q15. These roots grew in marshy places and are also called
breathing roots. They come out vertically outgrowth from the
ground and grow into the air to absorb O2 from atmosphere
through their numerous pores. Which roots has been referred to
in above statement: (a) Haustoria
(b) Epiphytic roots
(c) Assimilatory roots
(d) Pneumatophores
Ans. – (d) The branches of the roots that spread under the ground
develop vertically outgrowth that come above the ground and grow
into the air. These are known as pneumatophores. They are also
called breathing roots because they absorbs O2 from the atmosphere
through their numerous pores e.g. mangrove root.
Q16. Consider the given statement and choose correct code:
(1) Epithetic root develop adventitious roots that freely hang in the air
to absorb oxygen with help of a spongy tissue called Velamen.
(2) There is an organic connection between the epiphytic and the
trees on which they thrive.
Code:
(a) Only 1 is true
(b) Both are untrue
(c) Only 2 is true
(d) Both are true
Ans. – (b) There is no organic connection between epithytes root and
trees on which they thrive. The tree merely provided mechanical
support. Epythytic root absorbs atmospheric moisture with the help of
spongy tissue called velamen.
Q17. Match the following List-1 and List-2: List-1

(A) Green revolution (B) White revolution (C) Blue revolution (D)
Yellow revolution List-2
1. Milk production 2. Food grains
3. Fish production 4. Edible oil
Codes:

AB C D (a) 2 1
(b) 1 2
(c) 2 1
(d) 3 4
Ans. – (a)
Green revolution Yellow revolution Blue revolution White revolution 3
4344312

– Food grains
– Edible oil
– Fish production
– Milk production

Q18. In the given below which is not a disease caused by


bacteria:
(a) Plague (b) Leprosy
(c) Mumps (d) Tuberculosis Ans. – (c) Plague, Leprosy, Tuberculosis
are caused by bacteria but mumps is not caused by bacteria. Mumps
is a disease caused by paramyxo virus. With painful enlargement of
parotid glands, there is difficulty in opening of the mouth.

Q19. In the given below which one of the disease not caused by
virus?
(a) Chicken pox (c) Influenza (b) Measles (d) Typhoid
Ans. – (d) Chicken pox, influenza, measles are caused by virus but
typhoid is not caused by virus. Typhoid is caused by red shape
bacterium with numerous flagella, which help in active movement. So
option (d) is not correct.

Q20. Match the following List-1 and List-2: List-1 List-2 (A)
Protoplasm 1. De Duve (B) Cell theory 2. Altman (C) Mitochondria 3.
Purkense (D) Lysosome 4. Schleiden Codes:

AB C D
(a) 3 4 2 1
(b) 3 4 1 2
(c) 1 2 3 4
(d) 1 3 2 4
Ans. – (a)
Protoplasm – Purkense Cell Theory – Schleiden Mitochondria –
Altman
Lysosome – De Duve Q21. Match the following with list-1 to list-2
and select correct code:
List-1
(A) Modified leaves
(B) Cluster of flower born on a single axis (C) Mode of arrangement
of leaves on stem
(D) Position of floral leaves on the thalamus
List-2
1. Inflorescence
2. Bracts
3. Hypogyny
4. Phyllotaxy
Codes:
AB C D (a) 1 2 4 3
(b) 2 1 4 3
(c) 4 3 2 1
(d) 3 2 4 1
Ans. – (b)
Modified leaves – Bracts Cluster of flower on
a single axis – Mode of arrangement of leaves in stem – Position of
floral leaves on the thalamus – Inflorescence
Phyllotaxy

Hypogyny Q22. Consider the following statements regarding


Genetic disease in Man:

(1) Patients do not identify red and green colour in colour blindness.
(2) Turner’s syndrome is found in women. Due to this reproductive
part of women is in undeveloped condition.
(3) Downs syndrome is a genetic disease in which persons have dull
mind.
(4) Women are the carrier of hemophilia.
Choose the correct codes:
(a) 1 and 2
(b) 2 and 3
(c) All of the above
(d) Only 4
Ans. – (c) In colour blindness, patients do
not identify the red and green colour. It is
mainly found in male because women acts
like a carrier.
In Hemophilia clotting time is increased
due to this blood loss is more when wound
occur.
Downs syndrome is a genetic disease in
which persons have dull mind and irregular
physical structure. In turner’s syndrome
reproductive part of women is in undeveloped condition so they are
sterile. Q23. Match the following list-1 with list-2 and
select correct code:
list-1
(A) Putrefying bacteria
(B) Nitrate bacteria
(C) Nitrite bacteria
(D) Non-symbiotic nitrogen fixing bacteria
list-2

1. Azote bacter
2. Nitro bacter
3. Pseudomonas
4. Bacillus mycoides Codes:
AB C D (a) 1234 (b) 4231 (c) 4321 (d) 2143 Ans. – (b)
Putrefying bacteria Nitrate bacteria Nitrite bacteria
Non-symbiotic nitrogen fixing bacteria
– Bacillus mycoides
– Nitro bacter
– Pseudomonas

– Azote bacter

Q24. Which of the following is the basis for the Airy’s theory of
Isostasy:
(a) Law of Floatation
(b) Law of Compensation
(c) Principle of varying density
(d) Principle of uniform depth
Ans. – (a) Law of floatation is given by Archimedis. It states that “A
floating body displaces its own weight of the fluid in which it floats.
Airy’s where different topographic heights are accommodated by
changes in crustal thickness, in which the crust has a constant
density. Every effect has a cause. Every result has an antecedent.
The law of compensation keeps up the balance, and establishes
peace, concord, equilibrium and justice in nature.

Q25. Genetic use Restriction Technology (GURT) has been in


controversy for some time:
(1) GURT is used in connection

with plants.
(2) It is applied only for second
generation sterile.
(3) It can help in protecting in
biodiversity.
(4) It’s seeds are available in the market
at cheaper price than normal seeds. Choose the correct answer:
(a) 2 and 3 (b) 1 and 3
(c) 1, 2 and 4 (d) All of the above Ans. – (b) GURT is a term, used in
connection with plants and used to modify the behaviour of plants,
through genetic manipulation. It is also called terminator technology.
GURT can be of two types, VGURT (Variety specific GURT) and
tGURT (Trait specific). GURT can help in protecting biodiversity by
preventing the spread of transgenic crops into the wild and destroying
indigenous crops.

Part - 2
Ennvviioorrenmmeennttaall BBiioollooggy
National Park and Eco System

Q1. The Siberian Crane, an endangered migratory bird is a


regular visitor of which of the following National Park or Bird
Sanctuaries:
(a) Nawabganj Bird Sanctuaries (b) Vedanthangal Bird Sanctuaries
(c) Keoladeo National Park
(d) None of these
Ans. – (c) Keoladeo National Par, visiting migratory birds including
water birds. The rare Siberian Cranes visit the place in winter season.
Keoladeo national park in Bharatpur, Rajasthan is a famous avifauna
sanctuary that plays nest to thousands of birds especially during the
winter season.
It is also a declared world heritage site. The sanctuary is one of the
richest birds areas in the world. It is known for nesting of its resident
birds and visiting migratory birds including water birds.

Q2. In an ecotone, the species which become abundant are


called:
(a) Foster species
(b) Endemic species
(c) Edge species
(d) a and b
Ans. – (c) Ecotone is area between two biomass but different
patches of the landscape, such as forest and grassland. The ecotone
contains not only species common to the communities on both sides,
it may also include a number of highly adaptable species an edge
effect along the boundary line, with the area displaying a greater than
usual diversity of species.

Q3. The highest per capita emitter of carbon dioxide (CO2) in the
world is:
(a) Qutar (b) India
(c) China (d) USA
Ans. – (d) United State of America is the highest per capita emitter of
CO2 in the world ranked by metric tonnes of emission from the
burning of fossil fuels and cement manufacture, but not emissions
from land use such as deforestation.

Q4. Aquatic plant introduced from America to check pollution


turned out to be a troublesome weed in Indian water bodies. The
name is of this invasive alien species is:
(a) Eichhomia (b) Pistia
(c) a and b (d) None of these Ans. – (a) Eichhomia Crasspes,
commonly known as common water Hyacinth, is an aquatic plant
native to the Amazon basin and is often considered a highly
problematic invasive species outside its native region. The plant was
introduced by Belgian colonists to Rwanda to beautify their haltings
and then advanced by natural means to Lake Victoria where it was
first sighted in 1988.

Q5. ‘Lion-tailed Macaque’ is the key faunal species of which


Biosphere Reserve: (a) Sundarban (b) Nilgiri
(c) Nokrek (d) None of these
Ans. – (b) ‘Lion-tailed Macaque’ is the key faunal species of Nilgiri
Biosphere Reserve park. Nilgiri Biosphere Reserve Park is in Tamil
Nadu and is a famous hot spot. It was established in 1986. Liontailed
Macaque is an endangered species. Nilgiri Biosphere is the first
Biosphere Reserve in India.

Q6. Consider the following Tiger Reserve of India:


(1) Manas Tiger

Reserve – Assam
(2) Namdaffa Tiger
Reserve – Odisa
(3) Panna Tiger
Reserve – Madhya Pradesh (4) Sariska – Chattisgarh
Which of the above Tiger Reserve are correctly paired with their
respective location:
(a) 1 and 3 (b) 2 and 4
(c) 1, 2 and 3 (d) None of these
Ans. – (a)
Manas Tiger Reserve – Assam
Namdaffa Tiger
Reserve – Arunachal Pradesh Panna Tiger Reserve – Madhya
Pradesh Sariska Tiger Reserve – Rajasthan
Q7. Consider the following statements:
(1) The State of Forests Report (SFR) is
brought out by Forest Research
Institute, Dehradun.
(2) The forest cover assessment of India
is published in this annual report. Which of the above statements
is/are correct in this regard?
(a) Only 2 (b) Only 1
(c) Both 1 and 2 (d) None of these Ans. – (d) The State of Forests
Report (SFR) is brought out by the Forest Survey of India. The forest
cover assessment of India is published in this biennial report. Q8.
Consider the following classification of Biodiversity:
(1) Gamma diversity

(2) Alpha diversity


(3) Beta diversity
– Overall

Biodiversity
– within
Community
diversity
– between
Community
diversity. Which among the above is/are correctly paired?
(a) Only 3 (b) 1 and 2
(c) 1, 2 and 3 (d) None of these Ans. – (c)
*

* Alpha diversity – (within Community diversity) refers to the diverse


organism sharing the same
community/habitat.
Beta diversity – (between Community diversity) refers to the rate of
replacement of species along a gradient of habitats or communities.
Gamma diversity – (overall) refers to the diversity of habitats over the
total landscape or geographical area.

Q9. Consider the following statements regarding ‘Bhitarkanika


Mangroves’: (1) Bhitarkanika Mangroves is in Odisa is

a famous olive-riddley turtle. (2) It has the highest density of salt-


water
crocodiles in the country.
(3) It is a Ramsar site which is located in
the deltaic region of Subarnrekha
river.
Which of the statements given above is/are correct?
(a) Only 1 (b) Only 2
(c) 1 and 2 (d) 1, 2 and 3
Ans. – (c) It is a Ramsar site which is located in the deltaic region of
Brahman and Baitarni rivers. Bhitarkanika in the highest density of
salt-water crocodiles in the country.
Q10. Consider the following statements regarding Keoladeo
National Park: (1) Formerly known as the Bharatpur
Bird Sanctuary.
(2) It is one of the Ramsar wet lands of
International importance.
(3) It is inscribed in the UNESCO world
heritage list.
Which of the statements given above is/are correct?
(a) Only 1 (b) 1 and 2
(c) Only 3 (d) 1, 2 and 3 Ans. – (d) Keoladeo National Park in
Bharatpur, Rajasthan is a famous avifauna sanctuary that plays host
to thousands of birds especially during the winter season. It is also a
declared world heritage site. Q11. Consider the following flexible
mechanism of Kyoto Protocol:
(1) The Emission Trading Mechanism
(ETM) allows parties to the Kyoto
Protocol to buy ‘Kyoto Units’ from
other countries to help meet their
domestic emission reduction targets. (2) Through Joint
Implementation (JI),
countries can meet their domestic
emission reduction targets by buying
greenhouse gas reduction unit from
non Annex-I countries to the Kyoto
Protocol.
Which of the above statements is/are correct?
(a) Only 1 (b) Only 2
(c) 1 and 2 (d) None of these Ans. – (a) Through the Joint
Implementation, any Annex-I country can invest in emission reduction
projects (referred to as “Joint Implementation Projects”) in any other
Annex-I country as an alternative to reducing emission domestically.
Q12. All species of ‘Lemur’ are endemic to which among the
following places of the world:
(a) Madagascar (b) Greenland (c) South Africa (d) None of these
Ans. – (a) Endemism is the ecological state of being unique to a
defined geographic location, such as an island, nation or other
defined zone or habitat type. Q13. Which among the following is
the world’s largest wetland system: (a) Pantanal – South America
(b) Everglades – France
(c) Camorgue – Newziland (d) None of these
Ans. – (a)
Pantanal – South America Everglades – USA
Camargue – France
The worlds largest wetland is the Pantanal, which covers 2,00,000
square kilometers (during the wet season) through Brazil, Paraguay
and Bolivia, although 80% of it

is in Brazil. It is a land of loafed grasslands, savannas and tropical


forests.

Q.14 Which among the following multilateral environment


agreements (MEAs) is not correctly paired with the respective
issue it deals with:
(a) Basel Convention of 1989 –
Regulation of trans boundary movement, transit handling and use of
living modified organisms.

(b) Montreal Protocol of 1987 – Ozone

Depleting substances.
(c) Bonn Convention of 1979 – The
Conservation of Migratory Species. (d) None of these
Ans. – (a) The Basel Convention on the control of trans boundary
movements of hazardous wastes and their disposal, usually known
simply as the Basel Convention, is an international treaty that was
designed to reduce the movements of hazardous waste between
nations, and specifically to

prevent transfer of hazardous waste from developed to less


developed countries.

Q.15 Which of the following two criteria have to be not in order


to qualify as a ‘biodiversity hot spot’ on the world hot spots
map:
(1) The region must contain at least 1.5

percent or 15,000 species of vascular plants as endemic species.

(2) The region has to have lost at least 70 percent of its primary
vegetation.
Choose the correct option.
(a) Only 1 (b) Only 2
(c) 1 and 2 (d) None of these
Ans. – (b) The region must contain at least
5 percent or 1500 species of vascular plants
as endemics and it has to have lost at least
70% of its primary vegetation.
Q16. The ‘Red Data Book’ is the document of
rare and endangered species of:
(1) Animals (2) Plants
(3) Fungi (4) Bacteria
(a) Only 1 (b) 1 and 2
(c) 1, 2 and 3 (d) 1, 2, 3 and 4
Ans. – (c) A Red Data Book contains list
of species whose continued existence is
threatened. Species are classified into different categories of
perceived risk. Each
Red Data Book usually deals with a specific group of animals or
plants and fungi.
They are now being published in many different countries and
provided useful information on the threat status of the species. Q17.
Which of the following condition indicate the impact of global
warming.
Select the correct answer from the code
given below:

(1) Melting of glaciers


(2) Rise in sea level
(3) Charge in weather condition Code:
(a) Only 1 (b) 1 and 2
(c) Only 3 (d) All the above Ans. – (d) An indicate of the impact of
global warming as melting of glaciers, rise in sea level, change in
weather condition and rise in global temperature etc.

Q18. Match List-I with List-II and select the correct answer using
the code given below the lists:
List-I
(Sanctuary/National Park)
(A) Kaziranga
(B) Gir
(C) Sundarban
(D) Periyar
List-II
(Main Protected Wild Animal)
1. Tiger
2. Elephant
3. Lion
4. Rhinoceros
Codes:
AB C D (a) 1
(b) 4
(c) 4
(d) 1
Ans. – (b) Kaziranga Gir
Sundarban Periyar
234312321243

– Rhinoceros
– Lion
– Tiger
– Elephant

Q19. Match List-I with List-II and select the correct answer using
the code given below the lists:
List-I List-II
Tiger Reserve (A) Bandipur (B) Manas
(C) Namdapha (D) Simliport

State
1. Karnatka
2. Assam
3. Arunachal Pradesh
4. Odisha
Codes: AB C D (a) 1 (b) 4 (c) 3 (d) 1 Ans. – (a) Bandipur Manas
Namdapha Simliport 2 3 4 3 2 1 4 1 2 3 2 4

– Karnatka
– Assam
– Arunachal Pradesh
– Odisha

Q20. Consider the following statements: Assertion (A): A biome


is an ecosystem.

Reason (R): It comprises of both living and non-living elements of a


region.
Select the correct answer from the code given below.
Codes:
(a) Both (A) and (R) are true and (R) is

the correct explanation of (A). (b) Both (A) and (R) are true, but (R) is
not the correct explanation of (A).

(c) (A) is true but (R) is false.


(d) (A) is false but (R) is true.
Ans. – (a) A biome is an ecosystem because it comprises of both
living and non-living elements of a region.

Q21. Consider the following statements: Assertion (A): Coral


reefs are called the rain forests of the oceans.
Reason (R): They are highly rich in biodiversity.
Select the correct answer from the code given below.
Code:
(a) Both (A) and (R) are true and (R) is the correct explanation of (A).

(b) Both (A) and (R) are true, but (R) is not the correct explanation of
(A).
(c) (A) is true, but (R) is false.
(d) (A) is false but (R) is true.
Ans. – (a) Coral reefs are called the rain
forests of the oceans because they are highly rich in biodiversity.
Q22. Which one of the following pairs is not
correctly matched:
State
(a) Goa
(b) Meghalaya
(c) Nagaland
(d) Sikkim
Ans. – (d)
Goa –
Meghalaya –
Nagaland –
Sikkim –
Main Language Konkani
Khasi
Angami
Tibetan

Konkani Khasi
Angami Limbu

Q23. The Cartagena is regarding safe use, transfer and handling


of:
(a) Toxic by-products and industrial

effluents.
(b) Living Modified Organism (LMOs) (c) Nuclear waste
(d) None of these
Ans. – (b) The Cartagena Protocol on Biosafety to the Convention on
Biological diversity was adopted on 29 January, 2000 as a
supplementary agreement to the Convention on Biological Diversity
and entered into free on 11 September, 2003. It is an international
treaty governing the movements of Living Modified Organism (LMOs)
resulting from modern biotechnology from one country to another.

Q24. Consider the following statements regarding ‘Kyoto


Protocol’:
(1) It is legally binding for Annex-II

countries.
(2) It emerged from the deliberations of

Cop-3 to the UNFCC.


(3) The signatories have to reduce their
individual green house gas emission
by 5.2% from 1999 level.
Which among the above statements is/are correct.
(a) Only 1 (b) 1 and 2
(c) 2 and 3 (d) None of these Ans. – (b) Kyoto Protocol of Cop-3 to
the UNFCC is a legally binding agreement for Annex-I countries to
reduce their collective green house gas emission by 5.2% from 1990
level.

Q25. Which among the following is the most abundant green


house gas (GHG) in the earth’s atmosphere:
(a) Water vapour
(b) Carbon dioxide (CO2)
(c) Nitrozen (N2)
(d) Tropospheric Ozone
Ans. – (a) Water vapour also accounts for the largest percentage of
the greenhouse effect between 36% and 66% for clear sky conditions
and between 66% and 85% when including clouds.

Degradation and Pollution

Q1. Consider the following statements diseases never become


more severe. regarding ‘Keibul Lamjao National Q3. Pneumoconiosis
affects the workers whoPark’: work mainly in:
(1) The national park is characterized by (a) Tanneriesmany floating
decomposed plant (b) Glass industrymaterials locally called phumds. (c) Coal mining
industry(2) It is an integral part of Loktok lake (d) a and band also the only floating national
park in the world. Ans. – (c) Particulate matter from pro

Which among the above statements iscessing industries such as mine dust,
cotton
correct? dust, iron mill dust, flour mill dust etc. (a) Only 1 cause
pneumoconiosis, byssinosis,

(b) Only 2
emphysalma, siderosis and other pul
monary diseases.(c) 1 and 2 Q4. Which of the following statements
is/are(d) None of these correct:
Ans. – (c) The Keibul Lamjao National(a) Nilgiri – Himachal PradeshPark is
national park in the Bishnupur dis(b) Nokrek – Sikkimtrict of the state of Manipur in India. The

Brow-antlered deer, which was first dis(c) Namdapha – Arunachal


Pradesh covered in Manipur in 1839 and named (d) Sundarban –
Gujrat Cervus eldi eldi in 1844 in honour of Lt.Ans. – (c)Percy Eld – a British
officer, was reported Nilgirian extinct species in 1951. The animal

species concerned here is the endangered Nokrek


‘Brow-antlered deer or Sangai. Namdapha
Q2. Which among the following chronic land Sundarban

– Tamil Nadu
– Meghalaya
– Arunachal Pradesh
– West Bengal
disease commonly known as ‘black lung’ Q5. Match List-I (Indian wild life
species)disease, leads to reduced life expectancy with List-II (Scientific names) and
selectin coal-miners: the correct answer using the codes given

(a) Callier’s Asthma below.


(b) Cancer List-I
(c) Progressive Massive Fibrosis (Indian wild life species) (d)
Pneumoconiosis (A) Asiatic wild ass
Ans. – (d) A chronic occupational lung (B) Barasingha
disease contracted by the prolonged(C) Chinkarabreathing of coal mine dust.
Silica and car(D) Nilgiribon in the coal just cause black lung dis

ease. About one of every 20 miners studied List-II


in the US has x-ray evidence of black lung (Scientific names)
disease, a form of pneumoconiosis. In its1. Boselaphus tragocomelusearly
stage, called simple pneumoconiosis, 2. Cervus duvaucllithe disease does not prevent the
miners, the
3. Equas hemionus 4. Gazella-gazella Codes:

AB C D
(a) 3241
(b) 1234
(c) 3214
(d) 2341
Ans. – (a)
Asiatic wild ass – Equas hemionus Barasingha – Cervus duvauclli
Chinkara – Gazella-gazella Nilgiri – Boselaphus tragocomelus Q6.
Which of the following is/are causes of Thermal Pollution:
(1) Use of water as coolant.
(2) Storm water by power plants.
(3) Silting effect in the rivers.
Select the correct answer using the code given below.
(a) 1 and 2 (b) 2 and 3
(c) Only 3 (d) 1, 2 and 3
Ans. – (a) Thermal pollution is the degradation of water quality by
any process that changes ambient water temperature. A common
cause of thermal pollution is the use of water as a coolant, storm
water by power plants and industrial manufactures, when water used
as a coolant is returned to the natural environment at a higher
temperature.
Q7. Consider the following statements: (1) Salt water has the
potential to pump pollutants from the ocean into coastal freshwater
aquifers, making wells and other water sources contaminated. (2) The
major fraction of oil
contaminating the marine water is because of oil spilling.
Which of the above statements is/are correct?

(a) Only 1 (b) Only 2


(c) 1 and 2 (d) None of these Ans. – (a) Saltwater has the potential to
pump pollutants from the ocean into coastal freshwater aquifers,
making wells and other water source contaminated. Organic
pollutants are much, less soluble in saltwater than in freshwater.

Q8. Consider the following statements regarding artesian


aquifer:
(1) The great Artesian basin is the largest

and deepest artesian basin in the world.

(2) It is a confined aquifer containing groundwater under negative


pressure.
Which of the statements given above
is/are correct?
(a) Only 1 (b) Only 2
(c) 1 and 2 (d) None of these
Ans. – (c) Artesian aquifer is a confined
aquifer containing groundwater under positive pressure. This causes
the water level
in a well to rise to a paint, where hydrostatic equilibrium has been
reached. This
type of well is called an Artesian well.
Water may even reach the ground surface if
the natural pressure is high enough in
which case the well is called a flowing
Artesian well.
Q9. Primarily, the factor that distinguishes
wetland from other land forms or water
bodies is the characteristic vegetation
that is adapted to its unique soil condition. In this regard, which
statements
is/are correct?
(1) The water found in wetlands can be saltwater, fresh water or
barkish.
(2) Main wetland types include at least one meter depths of water.
(3) The largest wetlands in the world include the Amazon river basin
and the west Siberian plan.
Select the correct answer using the codes
given below.

(a) Only 1 (b) Only 2


(c) 1 and 2 (d) Only 3
Ans. – (b) A wetland is a land area that is saturated with water.
Wetland may be salt

water or fresh water. The largest wetland in the world are Amazon
river basin and west Siberian plan. Ramsar convention is an
international treaty for the conservation and sustainable utilization of
wetland.

Q10. Which of the following is/are source of methane:


(1) Landfills
(2) Coal mining
(3) Rice cultivation
Select the incorrect answer using the code given below.
(a) Only 1 (b) Only 2
(c) 1, 2 and 3 (d) None of these Ans. – (d) Source of the methane is
Landfills, coal mining, rice cultivation and apart from gas fields etc.

Q11. Consider the following about Green India Mission (GIM):


(1) Elimination of poverty is also an

aspect of Green India Mission. (2) The target is to be 5 million


hectares

of forest and non-forest land under green cover and improved the
quality of forest cover an another 5 million hectares.

Which of the statements given above is/are correct.


(a) Only 1 (b) Only 2
(c) 1 and 2 (d) None of these Ans. – (c) Green India Mission will be
implemented by gram sabhas with the technical and managerial
support of the forest personnel. Now money will directly go to the
gram sabhas and not through forest departments of state
governments. Its mission is also to enhance the annual CO2
sequestration of 50-60 million tonnes by the year 2020.

Q12. Consider the following statements regarding the wild life


related legislation:
(1) The Wildlife Protection Act, 1972,

provides for 6 schedules, in which the lost one is for the protection of
the plants.

(2) India was a signatory of the CBD and in order to realize the
objective of CBD, in 2002 the Biodiversity Act, 2002 was enacted.

Which of the statements given above is/are correct:


(a) Only 1 (b) 1 and 2
(c) Only 2 (d) None of these Ans. – (b) India is one of the first few
countries to have enacted such legislation. The act is to be
implemented through a three-tiered institutional structure, National
Biodiversity Authority (NBA), State Biodiversity Boards (SBBs) and
Biodiversity Management Committees (BMCs).

Q13. Consider the following places of ancient India:


(1) Daojali Hating (3) Burzaham
(2) Kolthiwa (4) Mahgara

Which of the above are Neolithic sites declared by


Archaeological Survey of India?
(a) 1, 2, 3 and 4
(b) 1 and 4
(c) 2, 3 and 4
(d) None of these
Ans. – (a)
The Daojali Hating – Tripura
Kolthiwa – Uttar Pradesh Mahagara – Madhya Pradesh Burzaham –
Jammu and

Kashmir
Q14. UNESCO’s World Heritage sites is a place of special and
cultural or physical significance with regard to this, consider the
following statements:
(1) The listed world heritage sites has

became legal territory of UNESCO. (2) By sites ranked by the


country, Iran
has greatest number of world heritage
sites with 47 sites.
Which of the statement given above is/are correct?
(a) Only 1 (b) Only 2
(c) 1 and 2 (d) None of these Ans. – (d) The list is maintained by the
international world heritage programme administered by the
UNESCO World Heritage Committee. By sites ranking Iran with 47
sites, followed by Spain (44) and China (43). The listed each world
heritage sites remain part of the legal heritage of the state, wherein
site is located.
Q15. Consider the following statements regarding Kyoto
Protocol:
(1) The main feature of the protocol is
that it established legally an non
binding commitments to reduce
emissions of green house gases for
Annex-I parties.
(2) The targets of Kyoto protocol apply to
the four green house gases carbon
dioxide (CO2), Methane (CH4),
Nitrous oxide (N2O), Sulphur
hexafluoxide (SF6), and two groups of
gases, hydro fluoro carbon (HFCs)
and per fluoro carbons (PFCs). Which of the above statements are
correct?
(a) Only 1 (b) Only 2
(c) 1 and 2 (d) None of these
Ans. – (c) Some of the principal concepts of the Kyoto Protocol are
Binding Commitments for the Annex-I Parties. The main feature of
the protocol is that it established legally binding commitments to
reduce emissions of green house gases for Annex-I parties. The
commitments were based on the Berlin mandate, which was a part of
UNFCCC negotiations leading up to the protocol.
Q16. Which of the following statements are not an objective of
the National Forest Policy:
(1) Environmental stability and
Ecological Balance.
(2) Checking the extension of Sand
Dunes.
(3) Reduction of Soil Erasion.
(4) Conservation of the Natural Heritage. Select the correct answer
using the codes given below.
(a) 1 and 2 (b) 2, 3 and 4 (c) 1, 2 and 3 (d) 1, 2, 3 and 4 Ans. – (d)
Maintenance of environmental stability through preservation and
where necessary, restoration of the ecological balance that has been
adversely disturbed by serious depletion of the forests of the country.
Checking the extension of Sand-dunes in the desert areas of
Rajasthan and along the coastal tracts.
Checking soil erosion and deduction in the catchment areas of rivers,
lakes, reservoirs in the interest of soil and water conservation, for
mitigating floods and droughts and for the retardation of siltation of
reservoirs.
Q17. The Millennium Ecosystem Assessment describes the
following major categories of ecosystem service revisioning,
supporting, regulating, preserving and cultural. Which one of the
following is supporting service:
(a) Production of food and water (b) Maintenance of diversity
(c) Control of climate and flood (d) None of these
Ans. – (d) Millennium Ecosystem Assessment (MEA), a four year
study involving more than 1300 scientists worldwide. This grouped
ecosystem service into four broad categories; provisioning, such as
the production of food and water regulating, such as the control of
climate and disease; supporting, such as nutrient cycle and crop
pollution and cultural, such as spiritual and recreational benefits. Q18.
The increasing amount of carbon dioxide in the air is slowly
raising the temperature of the atmosphere, because it absorbs:
(a) The ultraviolet part of the solar
radiation.
(b) The water vapour of the air and
retains its heat.
(c) All the solar radiations.
(d) The infrared part of the
solar radiation.
Ans. – (a) Carbon dioxide (CO2) absorbs the ultraviolet rays of solar
radiation and causes global warming. Gases which are responsible
for green house effect are water vapour, CH4, CO2, H2 and CFC.
Q19. Which of the following Biosphere Reserves are included in
the UNESCO’s world network of Biosphere Reserves: (1) Gulf of
Mannar
(2) Simlipal Biosphere Reserve
(3) Manas
(4) Panna
Which of the statements given above are correct?
(a) Only 1 (b) Only 2
(c) 1 and 2 (d) 3 and 4
The biosphere reserves included in the UNESCO’s world network of
biosphere reserves are : Nilgiri Biosphere Reserve, Gulf of Mannar,
Biosphere Reserve, Sundarban National Park, Nanda Devi Biosphere
Reserve, Nokrek Biosphere Reserve etc.
Q20.Consider the following statements regarding the
conservation of Tiger by the states:
(1) The states have to establish a Tiger conservation foundation,
based on the good practices emanating from some
tiger reserve.
(2) The states have to prepare annual
report for laying before parliament. (3) The tiger states have to tiger
conservation plan to ensure the
protection of the tiger reserve. Which of the above statements is
correct?
(a) Only 1 (b) Only 2
(c) 1, 2 and 3 (d) 2 and 3
Ans. – (c) The provision for the states regarding the conservation of
the tigers state level steering committees will be set up in the Tiger
states under the Chairmanship of respective Chief Ministers.
Q21. Consider the following statements regarding the National
Biodiversity Act, 2002:
(1) The Biodiversity Act, 2002 was
enacted by the Govt. of India in order
to give effect to the provision of
Convention on Biological Diversity
(CBD).
(2) The act provides for a two tier
institutional framework for the
implementation of the provisions of
the legislation.
(3) The act has provided for National
Biodiversity Authority so as to
regulate the implementation of the
provision of the legislation.
Which of the above statements are correct?
(a) 1 and 3 (b) Only 3
(c) 2 and 3 (d) 1, 2 and 3
Ans. – (a) The State Biodiversity Boards (SBBs) constituted by the
state governments, deal with all matters relating to access by Indian
for commercial purpose. The Indian industry is required to provide
prior intimation to the concerned SBB about the use of biological
resource. The SBB has the power to restrict any such activity, which
violates the objective of conservation, sustainable use and equitable
sharing of benefits.
Q22. Consider the following statements regarding
Environmental Kuznels Curve (EKC):
(1) Environmental Kuznels Curve (EKC)
hypothesis that the relationship
between per capita income and the use
of natural resource and/or the
emission of wastes has an inverted
U-shape.
(2) The preference for environmental
quality is the solo cause for the
inverted U-shape of the curve. Which of the above statements are
correct?
(a) Only 1 (b) Only 2
(c) 1 and 2 (d) None of these Ans. – (a) The term EKC is based on its
similarity to the time-series pattern of income inequality described by
Simon Kuznels in 1955. A 1992 WB Report make the nation an EKC
popular by suggesting that environmental degradation can be solved
by policies that protect that environment and promote economic
development. Q23. Consider the following statements regarding
environmental pollution: (1) Nitrogen Oxide (NO) and Sulphur
dioxide (SO2) are responsible for acid
rain.
(2) Gas heaters, coal mines and heating
devices are the source of carbon
monoxide.
(3) CFC cause Ozone depletion in the
mesosphere.
Select the correct answer.
(a) Only 1 (b) Only 2
(c) 1 and 2 (d) 2 and 3
Ans. – (c) The Nitrogen oxide and sulphur dioxides are responsible
for acid rain. The gas heaters, coal mines, heating devices and paddy
land [wetland are the source of the CH4 (Methane)].
Q24. Global warming has affected humidity at large. Following
are name of technologies, which could be used for carbon
sequestration.
Choose the correct answer from the codes given below.
(1) NOTT-202
(2) Ocean-iron fertilisation
(3) BECCS
(a) Only 1 (b) Only 2
(c) 2 and 3 (d) 1, 2 and 3
Ans. – (d) NOTT-202 is ‘metal-organic’ framework that works like a
sponge, absorbing a number of gases at high pressure but as
pressure is reduced, CO2 is rejoined as other are released. As per
reported in ‘Nature’ magazine, NOTT2022 holds promise for carbon-
capture and storage.
Q25. Consider the following statements about Hydroponics:
(1) Hydroponics is used in commercial
greenhouse vegetable production
around the world.
(2) It is the method of growing plants
without soil.
(3) Almost any terrestrial plant will grow
with hydroponics.
Select the correct answer.
(a) Only 1 (b) 2 and 3
(c) 1 and 2 (d) 1, 2 and 3
Ans. – (d) Hydroponics is explained as growing plants without soil.
This production system may use a wide variety of organic and
inorganic materials. The nutrient solution, rather than the media in
which the plant are growing, always supplies most of the plant.

Planning and Management

Q1. Nagoya Protocol on Access and Benefit Sharing (ABS) is


applied to which of the following aspects of biodiversity
protection:
(1) Genetic resources that are covered by

the CBD.
(2) Traditional Knowledge (TK)
associated with genetic resource that
are covered by the CBD.
Which following statements above are correct?
(a) Only 1 (b) Only 2
(c) 1 and 2 (d) None of these
Ans. – (c) Nagoya Protocol on access and benefit sharing is all fewer
given aspects of biodiversity protection. The benefits arisQ4.ing from the
utilization of the traditional

knowledge as against the benefits arising from their utilization.

Q2. The winter rainfall in North India are largely related to


streams and western disturbances why? Choose the following
correct answer:
(1) The westerly jet streams help the

western disturbances to reach the


North Indian region in winter.

(2) It (jet stream) helps the western


disturbances to travel in a particular direction under the influences of
westerlies.

(a) Only 1 (b) Only 2


(c) 1 and 2 (d) None of these
Ans. – (c) The winter rainfall in North India is closely associated with
the western disturbances. These disturbances originatQ5.over the east
Mediterranean ocean. It

moves towards the North India under the influence of westerlies. Jet
stream, is the relationship between jet stream and western
disturbance.
Q3. Consider the statements below regarding the sub-tropical
deserts in the world. Choose the following correct answer: (a)
These desert are located along the

west coast of the continents.


(b) These are under the influence of
sub-tropical high pressure belts. (c) These are also influenced by the
movement of cold oceanic current
along the coast.
(d) (a), (b) and (c).
Ans. – (d) They are also under the influence of the sub-tropical high
pressure belts of the world. These belts create the anticyclonic
conditions and hence also rainfall. The cold current along the coast is
responsible for the rainfall over the ocean. Niche construction
refers to:
(1) Process, based on feedback
relationship between natural
environment and the organism. (2) Process, in which ecological
biodiversity of an ecosystem has been
increased.
(3) Process, in which only environment
after the ecological profile of the
organism through natural selection. Which following is the correct
answer: (a) Only 1 (b) Only 2
(c) 1 and 3 (d) 2 and 3
Ans. – (a) ‘Ecological Niche’ word was introduced by ‘Grenels’ in
1971. Niche is a micro habitat where same types of species or sub
type species lived. This process is

based on feedback relationship between natural environment and the


organism.

Which of the following are a method for Solid Waste


Management. Select the correct answer from the codes given
below.
(1) Landfills
(2) Incineration
(3) Source Reduction
(a) Only 1 (b) Only 2
(c) 1 and 2 (d) 1, 2 and 3 Ans. – (d)
Solid Waste Management includes: (1) Source reduction
(2) Recycling and composing
(3) Combustion/incineration and

landfills.

Q6. Which of the following are components of Photochemical


Smog:
(1) Ozone
(2) Peroxyacetyl nitrate
(3) Nitrogen oxide
(4) Sulphur dioxide
Choose the correct answer from the codes given below.
(a) 1 and 2 (b) 3 and 4
(c) 1, 2 and 3 (d) 1 only
Ans. – (c) Photochemical smog is an air pollutant, which is created
by the reaction between sunlight and pollution like hydro

carbons, nitrogen dioxide and other volatile organic compounds


created from fossil fuel combustion.

* NO and NO2
*O3
*PAN
* VOCs

Q7. Consider the following statements regarding western


disturbances: (1) These are small low pressure systems

originating in Mediterranean region. (2) These are temperate


cyclones in occluded stage.

Which of the select correct answer? (a) Only 2 (b) 1 and 2


(c) Only 1 (d) None of these Ans. – (b) Western disturbance is the
term used in India, Pakistan, Bangladesh and Nepal to describe an
extra tropical storm originating in the Mediterranean that brings
sudden winter rain and snow to the North-Western parts of the Indian
subcontinents.

Q8. Carbon dioxide dissolves more easily in sea water compared


to pure water because sea water naturally contains: (a) Salt (b)
Minerals (c) Carbonate ions (d) Marine life Ans. – (c) Carbonate ions-
it is dissolved in the saltwater because saltwater contains calcium
carbonate ions as compare to pure water and is dissolved more
easily. Salt is produced by neutralized reaction that is –
Acid + Base = Salt

Q9. The Temperate Grassland or Shrubland Biome is commonly


known as:
(1) Steppe in Central Asia
(2) Prairie in North America
(3) Veld in South Africa
Choose the correct option:
(a) Only 3 (b) 1 and 2
(c) 1, 2 and 3 (d) 2 and 3
Ans. – (c)
Steppe – Central Asia
Prairies – North America
Pampas – South America
Veld – South Africa

Q10. The ranking of countries as the highest per capita emitters


of carbon dioxide in the world, takes into account their annual
CO2 emission from:
(1) Land use such as deforestation (2) Burning of fossils fuels
(3) Manufacture (Cement etc.)
(a) Only 2 (b) Only 3
(c) 2 and 3 (d) 1, 2 and 3 Ans. – (c) The data only considers carbon
dioxide emission from the burning of fossil fuels and cement
manufacture but not emission from land use such as deforestation. In
the list below the countries have been ranked by their metric tonnes
as carbon dioxide emission per capita in 2011.
Q11. The ‘Thickness’ of stratospheric ozone layer is measured
in:
(a) Dobson units (b) Joule units (c) Melson units (d) None of these
Ans. – (a) The ozone layer was discovered in 1913 by the French
physicists Chorles Fabry and Henri Buisson. Its properties were
explored in detail by the British meteorologist G.M.B. Dobson, who
developed a simple spectrophotometer (the Dobson meter) that could
be used to measure stratospheric ozone from the ground.

Q12. Which among the following is key faunal species that is


being conserved and monitored in ‘Dachigam National Park’: (a)
Kashmir stag or Hangul
(b) Golden Oriole
(c) Musk Deer
(d) None of these
Ans. – (a) Dachigam National Park is located 22 kilometers from
Srinagar, Jammu and Kashmir. It covers an area of 141 square
kilometers. The main animal species that Dachigam is most famous
for is the Hangul or Kashmir stag.

Q13. Which among the following awards has been recently


instituted by the Govt. of India for individuals or communities
from rural areas that have shown extraordinary courage and
dedication in protecting wildlife:
(a) Amrita Devi Bishnoi Award
(b) Rajiv Gandhi Green Award
(c) Govind Vallabh Pant Paryavaran

Award
(d) None of these
Ans. – (a) This award is given for significant contribution in the field
of wildlife protection, which is recognized as having shown exemplary
courage or having done exemplary work for the protection of wildlife.

Q14. Consider the following statements regarding Kyoto


Protocol:
(1) The parties to the protocol are to
submit report regarding the emission level of a regular interval of
time.

(2) The Kyoto Protocol was adopted in Kyoto, Japan on December


11, 1997 and entered into force on February 16, 2005.

Which of the above statements is correct.


(a) 1 and 2 (b) Only 1
(c) Only 2 (d) None of these Ans. – (a) Kyoto Protocol was adopted
in 1997. The aim of protocol is to controlling the concentration of
green house gas and focusing on struggle against global warming.
Kanada rejected Kyoto Protocol in 2011. In Marrakesh Accord, the
detailed rules for the implementation of Kyoto Protocol were adopted.

Q15. Consider the following statements: (1) Wildlife Sanctuaries


may be public or private and managed by any level of govt. or private
entity.

(2) National Parks are managed by only Central Government and


associated with the protection of the endangered species only.

Select the above correct statement. (a) Only 1 (b) Only 2


(c) 1 and 2 (d) None of these Ans. – (b) National Parks are managed
by only Central Government and associated with the protection of the
endangered species. Wildlife Sanctuaries cannot be under private
control. Wildlife Sanctuaries only managed by government only.

Q16. Consider the following statements about the geological


structure of the Earth: (1) Lithosphere forms the 3/10 of the total

surface area of the earth.


(2) The inner core of the earth contains
the solid iron and nickel.
(3) The upper mantle is called
Asthenosphere and carry the ocean
and continent on their backs. Which of the above select
statements is/are correct?
(a) Only 1 (b) Only 3
(c) 1 and 2 (d) 1, 2 and 3 Ans. – (d) The Asthenosphere is the molten
material inside on the earth. Its thickness varies from 150 to 200 km.
It is the part of the upper mantle. The Lithosphere is the upper part of
the earth or crust.
Q17. Consider the following statements: (1) Lithopone is used for
dyeing as it
remains unaffected by H2S.
(2) Deficiency of CO in human body
results in veilson’s disease.
(3) Zinc blende is used in phosphorescera
screens.
(4) Lunar caustic is responsible for
artificial rain.
Which of the select statements given above are correct.
(a) Only 3 (b) 1 and 2
(c) 2 and 4 (d) 1, 2 and 3
Ans. – (d) Circulation of the bill for eliciting public opinion, referring
the bill to a joint committee as well as moving amendments to the bill
all steps that can be taken by the house concerned during the second
reading of the bill. However acceptance or rejection of the bill as a
whole is done during the third reading.
Q18. Consider the following statements: (1) Most of the reduction
of carbon
emission can be done through
renewable energy, energy efficiency
and fuel switching.
(2) India has the largest potential for
production of Certificate for Emission
Reduction (CER).
Which of the above statements is correct.

(a) Only 1 (b) 1 and 2


(c) Only 2 (d) None of these Ans. – (a) CDM projects could be
registered in 2001 and 2012, the end of the Kyoto Commitment
Period. The CDM is expected to produce some 1.5 billion tonnes of
CO2 in emission reductions. Most of these reduction are through
renewable energy, energy efficiency and fuel switching. By 2012, the
largest potential for production of CERs are estimated in (52% of total
CERs) and India (16%) CERs production in Latin America and the
Caribbean make up 15% of the potential total, with Brazil as the
largest producer in the region (7%).

Q19. The ‘Red Data Book’ published by the International Union


for Conservation of Nature and Natural Resources (IUCN)
contain list of:
(1) Protected sites for conservation of

nature and natural resources in various countries.


(2) Endangered plant and animal species.
(3) Endemic plant and animal species present in the biodiversity hot
spots.
Which of the select the correct codes:
(a) Only 1 (b) Only 2
(c) 1 and 3 (d) 1, 2 and 3
Ans. – (b) IUCN categorisation of the
species on the basis of threat perspective–
* Extinct
* Dafe Deficient
* Extinct in wild
* Critically Endangered
* Endangered
Q20. The Coastal Regulation Zone, 2011 prohibits which of the
following:
(1) Landing or manufacture oil storage or disposal of hazardous
substance except storage of petroleum product.
(2) Established new industries and expansion except department of
atomic energy etc.
Which of the following statements given
above are correct.
(a) 1 and 2 (b) Only 2
(c) Only 1 (d) None of these
Ans. – (a) Coastal Regulation Zone was
notified to endure livelihood security to the
fisher communities and other local communities living in coastal area.
So, the
above option are prerequisite for that. Q21. Consider the following
statements about
Biochemical Oxygen Demand (BOD):
(1) The quality of organic pollutant in water can be determined by
measuring BOD.
(2) When huge amount of sewage is jumped into river, the BOD will
increase.
Select the correct statements given
above.
(a) 1 and 2 (b) Only 2
(c) Only 1 (d) None of these
Ans. – (b) Biochemical Oxygen Demand
(BOD) stands for Biochemical Oxygen
Demand. If BOD of a river is found high, it
means water is highly polluted.

Q22. Consider the following statements: (1) Interactions between


the two organisms, in which one organism kills and feeds on the
second organisms, is called Parasitism.

(2) Mutualism is the way two organisms of different species


biologically interact in a relationship, in which each individual derives
a fitness benefit.

Which of the above statements is correct.

(a) Only 2 (b) Only 1


(c) 1 and 2 (d) None of these Ans. – (a) Interaction between the two
organisms, in which one organism kills and feed on the second
organisms, is called predion.
Parasitism is the relation between two organisms, where one
receives benefits at the cost of the other.

Q23. Black Carbon has recently emerged as a major contributor


to global climate change. In the context of it, consider the
following:
(1) Primary sources include emissions
from diesel engines and cook stoves.

(2) Black carbon particles strongly absorb sunlight and give soot its
black colour.

Which of the statements given above is correct.


(a) Only 1 (b) Only 2
(c) 1 and 2 (d) None of these Ans. – (c) Black Carbon (BC) has
recently emerged as a major contributor to global climate change,
possibly second only to CO2 as the main driver of change. BC
particles strongly absorb sunlight and give soot its black colour etc.

Q24. Which of the following convention are not associated with


the Ozone Layer Protection:
(1) Montreal Protocol
(2) Vienna Convention
(3) Cortagene
(4) Nagoya Convention
Select the correct answer using the code given below.
(a) Only 1 (b) 1 and 2
(c) 1 and 3 (d) Only 3
Ans. – (d) The Montreal Protocol on substance that deplete the
ozone layer is an internationaly designed protocol to protect the
ozone layer by phasing out the production of numerous substance
believed to be responsible for depletion. The Cortagena Protocol on
Biosety is on international agreement.

Q25. Which of the following is a statements representing the


Asian Brown Cloud correctly:
(a) It is a thick layer of pollutants. (b) It is a shield to restrict the
Gamma

ultraviolet rays to reach the earth surface.


(c) (a) and (b)
(d) None of these
Ans. – (a) The Indian Asian Brown Cloud is a layer of air pollution
that recurrently covers parts of South Asia, namely the Northern
Indian Ocean, India and Pakistan. The cloud appears as a giant
brown stain hanging in the air over much of South Asia and the Indian
Ocean every year between January and March, possibly also during
earlier and later months.

Part - 3
Physics
Mechanics

Q1. Match the following:


List-1
A. Photoelectric effect
B. Magnetic field
C. Electric press
D. Radioactivity
List-2
X. Tesla
Y. Eienstein
Z. Henry Becquarle
P. Henry Sheile
Choose the correct option:
(a) A-Y, B-X, C-P, D-Z
(b) A-X, B-Y, C-P, D-Z
(c) A-Z, B-X, C-Z, D-P
(d) A-X, B-Y, C-Z, D-P
Ans. – (a)
Photoelectric effect – Eienstein Magnetic field – Tesla
Electric Press – Henry Sheile Henry Becquarle – Radioactivity

Q2. Food cans are coated with tin not with zinc because:
(a) Zinc is costlier than tin
(b) Zinc has higher melting point than tin (c) Zinc is more reactive
than tin (d) Zinc is less reactive than tin
Ans. – (c) In metal activity series the position of zinc is above in
comparison to tin i.e. zinc is more reactive.
Tin can process was allegedly developed by Frenchman Pillippe de
Girard. Food cans are still mostly made from tin plate using a three
piece construction.
Although tin is corrosion resistant, acidic food like fruits and
vegetables can cause corrosion of the tin layer causing Nausea,
Vomiting and Diarrhoea.
Q3. While cooking, if the bottom of the vessel is blackened on
the outside, it means that:
(a) The food is not cooked completely. (b) The fuel is not burning
completely. (c) The fuel is wet.
(d) The fuel is burning completely. Ans. – (b) Bottom of the vessel is
blackened due to incomplete combustion of fuel. It happens due to air
holes are blocked and proper oxygen is not supplied.
For cooking food we must use thick pot bottoms because it distribute
heat better because the heat spreads in uniform manner. When the
bottom of the pot is too thin, the heat of the flame immediately
penetrates in that the spot, possibly resulting in burnt or blackened.

Q4. Which one of the following has high calorific value:


(a) Hydrogen (b) Coal
(c) Natural gas (d) Gasoline Ans. – (a) Calorific values of above fuels
is as follows:
Hydrogen – 150
Coal – 33
Natural Gas – 33.50
Gasoline – 50
Calorific value – The calories of thermal units contained in one unit
of a substance and released when the substance is burned. Caloric
values of Fuels (k cal/kg) Fossil Fuels
Coal – 4000-7000 Carbon – 8000
Petrol – 10800
Biogas – 4700-6000

Q5. Consider the following statement about Jet engine and a


rocket:
(1) A Jet engine uses the surrounding air

for its oxygen supply and so is unsuitable for motion in space.

(2) A rocket carries its own supply of oxygen in the gas form and fuel.
(a) 2 only (b) 1 only
(c) Both 1 and 2 (d) Neither 1 nor 2
Ans. – (c) The fuel used in rockets is
called rocket fuel. A propellant is an explosive used to fire a projectile
from a gun. It
is a combination of an oxidizer (like liquid
hydrogen, liquid oxygen or nitric acid) and
a fuel which when ignited undergoes combustion to release large
quantity of hot
gases.
Q6. Consider the following statements:
(1) The images produced by plane mirrors are always virtual.
(2) Concave mirrors forms always real images.
(3) Concave mirrors is of converging in nature.
(4) Converging lens can produce virtual as well as real images.
Which of the above is/are true?
(a) Only 3 (b) 2 and 3
(c) 1, 3 and 4 (d) All of these Ans. – (c) Plane mirror never form real
images because never really converge. Concave mirrors form real as
well as virtual image (when object is in between (P and F).
Converging lens can produce virtual as well as real images.
Q7. Assertion (A): A quick collision between two bodies is more
violent than a slow collision, even when the initial and final velocities
are identical.
Reason (R): Impulsive force is greater in the first case.
Codes:
(a) A is true, but R is false.
(b) A is false, but R is true.
(c) Both A and R is true and R is the correct explanation of A.
(d) Both A and R is true and R is not the correct explanation of A.
Ans. – (c) Impulse = Force × Time
For quick collision, time t is small so
greater amount of force is applied. For this
reason quick collision is more violent.
Impulsive force is greater in quick collision.
Q8. Which of the following is the basis for
The Airy’s Theory of Iso stasy:
(a) Law of compensation
(b) Principle of uniform depth
(c) Law of flotation
(d) Law of compensation
Ans. – (a) The law of flotation tells that
floating body displaces its own weight of
the fluid. It is introduced by Archemedies.
There must be perfect balance between the
cause and effect between the antecedent
and consequence. The law of compensation keeps up the balance
and establishes
peace, concord, equilibrium harmony and
justice in nature.

Q9. Consider the following statements: (1) Salt water has the
potential to pump pollutants from the ocean into coastal fresh water
aquifers making wells and other water sources contaminated.
(2) The major fraction of oil

contaminating the marine water is because of oil spilling.


Which of the above statements is/are
correct.
(a) Both 1 and 2 (b) Only 1
(c) Only 2 (d) Neither 1 nor 2
Ans. – (b) Almost 90% of all floating
materials in the ocean are plastic. It is one
of the most harmful pollutants, because it
does not readily break down in environments.
Saltwater has the potential to pump pollutants from the ocean into
coastal fresh
water aquifers making wells and other
water sources contaminated.
Q10. Who among the following scientists
shared the Nobel prize in physics with
his son:
(a) Max Planck (b) Max Weber
(c) Henry Fayol (d) Henry Bragg
Ans. – (a) Sir William Henry Bragg and
his son, Sir William Lawrence Bragg, both
received the Nobel prize in physics in
1915. The Braggs are the first and only
father/son team to has jointly received the
Nobel Prize. William Lawrence Bragg was
awarded the Nobel Prize at age of 25 and
as of yet he is still the youngest ever
laureate.
Q11. Rain bearing clouds look black. Give
probable reasons.
(1) Part of light is scattered by the tiny droplets of water in clouds.
(2) Part of light is absorbed by the tiny droplets of water in clouds.
(3) There is a lot of dust condensed in water vapour.
Which of the above statements is/are true?
(a) 1 and 2 (b) 2 and 3
(c) 1 and 2 (d) All of the above Ans. – (c) Most clouds appear white,
because sunlight reflects off the water droplets. If the clouds are
thick, droplets scatter or absorbs the light and less solar radiation can
travel through them, so storms looks dark, black or gray.
Q12. Which of the following used by computed tomography
employed for virtualisation of the internal structure of human
body:
(a) Sound waves
(b) X-rays
(c) Magnetic resonance

(d) Radio isotopes


Ans. – (b) CT is a powerful non-destrucCT is a powerful non-destruc
D and 3-D across sectional images of an object from flat X-ray
images. Characteristics of the internal structures of an object such as
dimensions, shape internal defects and density are readily available
from CT images.

Q13. Find Resistance between A and B:

(a) 5 Ω (b) 10Ω


(c) 6 Ω (d) 2 Ω
Ans. – (a) Here Resistance between terminal X and Y is in parallel

So RXY = 2 × 2 = 1Ω
2+2
Net Resistance RAB = 2 Ω + RXY + 2 Ω RAB = 5 Ω
Q14. Match the following List-1 and List-2 correctly:
A. Ammeter 1. Potential differB. Anemometer C. Pyrometer
D. Galvanometer/

Voltmeter
ence/current 2. Electric current

3. Air speed
4. High temperature Codes:

AB C D
(a) 1234
(b) 1243
(c) 4312
(d) 2341
Ans. – (d)
Ammeter – It measures electric current. It is connected in series. It
has low resistance. Anemometer – It measures the speed of Air.
Pyrometer – It measures high temperature. Galvanometer – Current
Voltmeter – Potential difference

Q15. Extensive rice cultivation, some region may be contributing


to global warming. What is the possible reason:

(1) When nitrogen based fertilizers are used, nitrous oxide is emitted
from the soil.

(2) The anaerobic condition with rice cultivation causes the emission
of Methane (CH4)

Which of the option is/are correct: (a) 1 only (b) 2 only (c) Neither 1
nor 2 (d) Both Ans. – (d) Methane is produced under the anaerobic
conditions associated with rice cultivation. Excess uses of Nitrogen
base

fertilizers, nitrous oxide is emitted from the soil. NOX is one of the
main cause for global warming.
Q16. Consider the following statements: (1) Wi-Fi device
(2) Cordless phone
(3) Microwave oven
(4) Bluetooth device
Which of the above operate between 2.4 and 2.5 GH range of
frequency?
(a) 1 and 3 (b) 2 and 4
(c) 1, 2 and 3 (d) All of the above Ans. – (d) Bluetooth is very short
range device. It is used to send data from one PC to another PC
without through any cable network.
Frequency bands allocated for cordless phones in the range 2.4 GHZ
to 2.4855 GHZ, which is licensed and used for many communication
applications.

Q17. A ball hits the floor and rebounds after an inelastic


collision. In this case: (a) The momentum of the ball just after

the collision is the same as that just before the collision.

(b) Mechanical energy of the ball remains the same.


(c) The total momentum of the ball and the earth is conserved.
(d) The total energy of the ball and the earth is conserved.
Ans. – (c) In an inelastic collision only
momentum of the system may remain conserved. Some energy can
be lost in the
form of heat, sound etc.
Mechanical energy (U) = Kinetic energy +
Potential energy
Here, transformation of one form of energy
into another form. So mechanical energy
remains constant.
Q18. The dimension of the quantities in one
(or more) of the following pairs are the
same. Identify the pair(s):
(a) Torque and work
(b) Angular momentum and work
(c) Energy and Young’s Modulus
(d) Light year and wave length
Choose correct options:
(A) a & b (B) c & d
(C) a & d (d) All of the above

Ans. – (c)

(a) Torque and work both have the dimensions [ML2T–2]


(d) Light year and wave length both have the dimension of length i.e.
[L]. Q19. Two masses of 1g and 4g are moving
with equal kinetic energies. The ratio of
the magnitudes of their momenta:
(a) 4 : 1 (b) √2 : 1
(c) 1 : 2 (d) 1 : 16

Ans. – (c) Kinetic energy = 1 mv2


2

Momentum (P) = MV
Rel. between kinetic energy and momenta is P =√2KM
or P∝√M
M1 = 1 M2 4
P1 = 1
P2 2

Q20. During Paddling of a bicycle, the force of friction exerted by


the ground on the two wheels is such that it acts:
(a) in the backward direction on the front

wheel and in the forward direction on the rear wheel.

(b) in the forward direction on the front wheel and in the backward
direction on the rear wheel.

(c) in the backward direction on both the front and the rear wheels.
(d) in the forward direction on both the front and the rear wheels.
Ans. – (a) Friction is an electromagnetic
force. It always opposes the relative
motion between two bodies.
During Paddling front wheel moves in forward direction so frictional
force acts in
backward direction and rear wheels sleeps
in backward so frictional force will act in
forward.
Q21. Assertion (A): A cloth covers a table.
Some dishes are kept on it. The cloth can be pulled out without
dislodging the dishes from the table.
Reason (R): For every action there is an equal and opposite reaction.
(a) A and R is true and R is correct explanation of A.
(b) A and R is true but R is not correct explanation.
(c) A is true and R is false.
(d) A is false and R is true.
Ans. – (b) The cloth can be pulled out
without dislodging the dishes from the
table due to law of inertia, which is
Newton’s first law while the statement II is
true but it is Newton’s 3rd law.
Q22. Match the following:
List-1
A. Dynamo
B. Motor
C. Microphone
D. Battery
List-2
1. It converts Chemical Energy into Electrical Energy.
2. Mechanical to Electrical Energy
3. Electrical to Mechanical Energy
4. Sound Energy to Electrical Energy AB C D
(a) 1234
(b) 2341
(c) 3241
(d) 4321
Ans. – (b)
Dynamo – It coverts Mechanical
Energy into Electrical Energy
Motor – Electrical to Mechanical
Microphone –Sound Energy to
Electrical Energy
Battery – Chemical Energy into
Electrical Energy
Q23. When one body is in thermal equilibrium with another body
by which Rule: (a) Zeroth Law of thermodynamics (b) First Law of
thermodynamics (c) Second Law of thermodynamics (d) Third Law of
thermodynamics Ans. – (a) Zeroth Law of thermodynamics states
that

Temperature of body A is equal to temperature of body B and temp.


of body B is equal to temp. C then body A is in thermal equilibrium
with body ‘C’.

Q24. Assertion (A): In the visible spectrum of light, red light is more
energetic than green light.
Reason (R): The wavelength of red light is more than that of green
light.

Codes:

(a) A and R are true and R is correct explanation of A.


(b) A and R are true but R is not correct explanation of A.
(c) A is true but R is false.
(d) A is false but R is true.
Ans. – (d)
Light is an electromagnetic wave that has a
wavelength in about the range of 380-780 nano meter and that may
be perceived by normal eye. White light is a mixture of 7 colour.
VIBGYOR
Energy = hv (h = Plank constant v = Frequency of wave)
* Wavelength increases frequency decreases so red light is not more
energetic than green light.
Q25. Read the following statements carefully:
Y : The resistivity of semiconductor
decreases with increase of temperature.
Z : In a conducting solid, the rate of collisions between free electrons
and ions
increases with increase of temperature.
Select the correct statement:

(a) Y is true but Z is false.


(b) Y is false but Z is true.
(c) Both Y and Z are true.
(d) Y is true and Z is correct reason for Y.

Ans. – (c)

Resistivity of conductors increases with increase in temperature


because rate of collisions between free electrons and ions increase
with increase of temperature Resistivity of semiconductor decrease
with increase in temperature, so it has negative temperature gradient.

Waves and Currents

Q1. Which one of the following happen at absolute zero


temperature:
(a) All the molecules comes to rest and

have zero energy.


(b) Molecules vibrates more faster at this
temperature.
(c) Potential energy is zero.
(d) None of these.
Ans. – (d) Absolute zero is the theoretical temperature at which
entrophy reaches its minimum value. The laws of thermodynamics
states that absolute zero cannot be reached using only
thermodynamics means. A system at absolute zero still possesses
quantum mechanical zero point energy, the energy of ground state
cannot be removed. However, in classical interpretation, it is zero and
the thermal energy of the matter vanished.
Q2. Choose the following statements related to silver plating of a
brass spoon: (1) It is suspended in a aqueous solution
of sulphuric acid.
(2) The spoon is made the cathode and
anode is a plate of pure clean silver. (3) Before electroplating process
the
spoon is washed by caustic soda. Code:
(a) 1 and 2 (b) Only 3
(c) 2 and 3 (d) None of these Ans. – (c) For silver plating the spoon is
made the cathode and anode is a plate of pure silver. The spoon is
suspended in a both of sodium silver cyanide and made by adding
NaCN to a solution of AgNO3 until the precipitate form just dissolves.
If silver nitrate solution is used directly instead of double cyanide of
sodium and silver, the deposition of silver will be very fast, not very
smooth and uniform.
Q3. If a light bulb is placed between two plane mirrors inclined at
an angle of 60° the number of image formed are: (a) 9 (b) 7
(c) 5 (d) 2
Ans. – (c) If two mirror are included at an angle, we get many image
since the image formed by one mirror acts as an object for the other
mirror. The total number of images formed by two mirror inclined at
an angle is given by the relation,

number of image (n) = 360 − 1


θ
In this case n = 360 − 1 = 5
60

Q4. Which one of the following instruments is usedto measure


the depth of sea: (a) Lactometer (b) Fathometer
(c) Pyrometer (d) None of the above Ans. – (b) The phenomenon of
reflection of sound has been applied to measure the depth of sea.
Sound is produced from a place at know distance from the reflecting
body. The interval of time t during which echo reached the placed
where sound was produced is noted by a stopwatch. The velocity of
sound is calculated by the formula,
V = 2d/t m/s

Q5. When a bimetallic strip is heated it will:

(a) Bend toward the metal with higher thermal coefficient of


expansion.
(b) Twist itself into helix.
(c) Not bend at all.
(d) None of the above.
Ans. – (d) For higher temperature control,
bimetallic thermostat are used. The
bimetallic strip works as electric contact
breaker in a electric heating circuit. The
circuit is broken when the desired temperature of the both is reached.
Due to difference in the coefficient of linear expansion
of the two, when it is the metallic strips curved at the circuit broken.
For when the bimetallic strip is heated it will bend toward the metal
with lower thermal expansion coefficients.
Q6. Which of the following statement is/are correct:
(1) Carbon has tetravalency. Its atomic no. is six.
(2) It is a metal.
(3) Isoneutrons have same no. of electron.
(4) pH of Human blood is slightly ‘basic’.
Choose correct options:
(a) 1, 3 and 4 (b) 1 and 4
(c) 1, 2 and 3 (d) All of the above
Ans. – (b) Carbon is a non-metal, whose
atomic no. is 6 and atomic mass is 12. It
has large no. of compound.
pH of human blood is slightly basic. Its pH
value is 7.4. So it is basic in nature.
The elements having same no. of electrons
is known as Isoelectronic and same no. of
neutrons is known as Isoneutronic. Q7. A closed bottle containing
water at 40°C
is carried the moon in a spaceship. If it is
placed on the surface of the moon what
will happen to the water as soon as the
lid is opened?
(a) It will decompose into H2 and O.
(b) Water will freeze.
(c) Water will boil.
(d) Nothing will happen to it.
Ans. – (c) On the surface of the moon, the
pressure is much less than on earths surface, therefore water will boil
as soon as
the lid is opened. The boiling point
decreases with decreasing the pressure. So
option (c) is correct.
Q8. Consider the following statements and
select the correct code:
(1) Mercury thermometer can be used to measure temperature upto
360°.
(2) Mercury has low specific heat and hence, it absorbs little heat
from the body whose temperature is being measured.
Code:
(a) Only 2 is correct.
(b) Only 1 is correct.
(c) Both 1 and 2 correct.
(d) Neither 1 nor 2 are correct.
Ans. – (c) As the boiling point of mercury
is about 360°C. Therefore it can be used to
measure temperatures upto 360°C and also
option (2) is correct.
Q9. A cycle with well inflated tyres left in the
sun sometimes has its tubes burst open.
Which one of the following is reason for
this?
(a) Both volume and pressure increases.
(b) The tube melt due to intense heat.
(c) The volume of air inside the tube increases.
(d) The pressure of air inside the tube is increases.
Ans. – (d) In this case, the volume remain
constant and with increases in temperature
pressure of air inside the tube increases.
Pressure is inversely proportioned to temperature.
Q10. If a star emitted yellow light stars moving toward the earth
its colours as seen
from the earth will?
(a) Turn bright yellow
(b) Turn gradually blue
(c) Remain yellow
(d) Turn gradually white
Ans. – (b) For this case, source is moving
toward the observer. According to Doppler
effect apparent frequency will increases.
Therefore, its colour will gradually change
from yellow to blue.
Q11. Which one of the following is incorrect
with respect to the factor which affect
the velocity of sound in air or gas:
(a) The velocity of sound wave is more in the direction of the wind
and less in the opposite direction.
(b) Velocity of sound increases with the rise in temperature and
proportional to the square root of the temperature is absolute scale.
(c) a and b both.
(d) The velocity of sound decreases with the increase the humidity in
atmosphere.
Ans. – (d) The density of moist air is less
than the dry air. Hence, the velocity of
sound increases with the increasing of
humidity in the atmosphere.
Q12. Consider the following statements and
select the correct code:
(1) In order that an echo may occur, the reflecting surface must be at
least 33 meter away.
(2) The air is able to distinguish two sound as separates sound only if
they reach the ear at least 1/10 of the second apart.
Code:
(a) Only 1 is correct
(b) Only 2 is correct
(c) Both 1 and 2 are correct
(d) Neither 1 nor 2 are correct
Ans. – (b) In order that an echo may
occur, the reflecting surface must be at
least 16.5 meter away since sound travelling at 330 m/s will go 33
meter from the
observer to the reflector and back in 1/10 s. Q13. Which of the
following statements is
incorrect:
(a) The wavelength of sound wave varies directly as its frequencies.
(b) The shapness of a musical note depend on its pitch.
(c) Pitch enable is to differentiate between two sounds with equal
loudness coming from the same sources.
(d) None of the above.
Ans. – (a) Except option (a) all of the given option is correct, so
option (a) is incorrect because the wavelength of a sound wave
varies inversely to its frequency.
Q14. Consider the following statements and select the correct
code:
(1) Volcanic eruption emit ultrasonic waves.
(2) Ultrasonic sound wave are inaudible to human ear.
(3) Ultrasonic wave are used to destroy bacteria in water and to drill
holes in hard and brittle material.
Code:
(a) 1 and 2 are correct

(b) 1, 2 and 3 are correct


(c) Only 1 and 3 are correct
(d) 2 and 3 are correct
Ans. – (d) Volcanic eruption emit infrasonic waves, ultrasonic have
frequencies higher than 20000 HZ (Audible range of frequency for
human is between 20 HZ to 20000 HZ).

Q15. Which one of the following is correct sequences of velocity


of sound in different mediums in decreasing order: (a) Iron,
glass, water, air
(b) Glass, air, water, iron

(c) Air, water, iron, glass


(d) Water, iron, glass, air
Ans. – (a)
Velocity of sound in iron = 5200 m/s Velocity of sound in glass = 4540
m/s Velocity of sound in water = 1498 m/s Velocity of sound in air =
346 m/s So option (a) is a decreasing order of velocity of sound in
different medium.
Q16. Which one of the following colour of light will travel with
minimum speed through glass?

(a) Yellow (b) White


(c) Red (d) Violet
Ans. – (d)

Here,
μ
=
C air
C medium
C medium = C airμ

As μ for violet light is more, the velocity of light through glass will be
minimum. Q17. A diffraction pattern is obtained using a

beam of red light. What will happen if red light replaced by blue
light? (a) Band disappear
(b) Band become breeder
(c) No change
(d) Diffraction bands becomes narrower

and crowder together


Ans. – (d) As blue light has less wavelength than red light the band
diffraction becomes narrows and crowder together. We used red light
for back side of vehicle because it has more high wavelength.

Q18. Total internal reflection can occur when light passes from:
(a) Denser to rarer medium.
(b) It occur when light travel from one

medium to another medium of equal refractive index.

(c) It occur irrespective of medium of travel.


(d) Rarer to denser medium.
Ans. – (a) The total internal reflection
occur when light passes from a denser
medium to another rarer medium and the
angle of incidence is more than the critical
angle.
Q19. The second’s hand of a watch is 3 cm
long. Which of the following is the speed
of the tip of this hand:
(a) 0.5192 cm/s (b) 1 cm/s
(c) 0.3142 cm/s (d) 0.4190 cm/s
Ans. – (c) The second’s hand makes one
complete rotation in one minute. Distance moved by the tip of the
hand in one minute = 2πr = 2 × π × 3 = 6 π cm

Speed = Distance = 6π cm/s


time 60
= 0.3142 cm/s

Q20. A ball thrown vertically upward with an initial velocity of


19.6 m/s returns in 4 seconds. Which one of the following is the
maximum height attained by the ball:
(a) 20 m (b) 22 m
(c) 19.6 m (d) 17 m
Ans. – (c) The time taken by the ball go to the highest point is 4/2 = 2
seconds. The time taken to come down from the highest point also 2
seconds, therefore,
h = 1/2 gt2 = 1/2 × 9.8 × 4 = 19.6 m h = 19.6 m

Q21. Which one of the following denotes mass of inertia:


(a) F/M (b) V × d
(c) M/V (d) F/a
Ans. – (d) We know that the equation f = M × a where f is force and
M is mass and a acceleration.
f=M×a
M = f/a
This mass is called the inertial mass and defined as the ratio of the
force and acceleration produced in that mass due to applied force.

Q22. Which one of the following acts as an anode in the


leclanche cell:
(a) Copper rod (b) Carbon rod (c) Zinc rod (d) Aluminium rod Ans. –
(b) In a leclanche cell a carbon red is placed in a porous pot which is
the packed with a mixture of powdered carbon and magnese oxides.
The porous pot is then placed in a glass jar containing the solution of
aluminium chloride and a zinc rod. The carbon rod and the zinc rod
act as positive and negative poles respectively.

Q23. Newton postulated his corpuscular theory of light on the


basis of:
(a) Black and white of thin films (b) Newton’s ring
(c) Colour of thin films
(d) Rectilinear propagation of light Ans. – (d) According to the
Newton’s corpuscular theory, a luminous body continuously emit tiny,
light and elastic particle called corpuscular in all directions. These
particle are so small that they can readily travel through the
interstices of the particle of matter with the velocity of light and they
possess the property of reflection from a polished surface and
transmission through a transparent medium. This explain rectilinear
propagation of light.

Q24. The intensity of sound waves depend upon which one of


the following: (a) Velocity of the medium
(b) Amplitude
(c) Density of the medium
(d) b and c both
Ans. – (d) Intensity of sound is the time rate of at which the sound
energy flow through the unit area. Intensity of sound varies directly as
the square of the amplitude of the vibrations. Intensity varies
inversely as the also of the square of the distance between the
sources and hearer. Intensity is directly proportional to the density of
the medium in which the sound originated.

Q25. Which one of the following is produced by laser:


(a) A beam of fast moving neutron rays. (b) Highly penetrating X-rays.
(c) A beam of monochromatic coherent

light.
(d) None of the above.
Ans. – (c) A laser is device that emit (electromagnetic radiation)
through a process of optical amplification based on the stimulated
emission of photon. The terms ‘laser’ originated as an acronym for
light amplification by stimulated emission of radiation. The emitted
laser light in notable for its high degree of spatted and temporal
coherence unattainable using other technologies. The laser produced
a beam of monochromatic coherent light.

Electromagnetic

Q1. Which of the following statements is/are correct:


(1) Producer gas is a mixture of CO

and N2.
(2) Water gas burn with a smokeless
flame.
(3) Producer gas used in manufacture of
coal gas and to operate gas engines. Choose correct options.
(a) 1 and 2 (b) 2 and 3 (c) All of the above (d) None of these Ans. –
(c) Producer gas is made by air passing over red hot coke. As a
result, some of the coke burns to CO2 at the bottom of the furnace.
This gas reduced to carbon monoxide (CO) as it passes through the
upper layer of red hot coke. Formation of producer gas is exothermic
reaction. Water gas is a mixture of carbon monoxide and hydrogen
(CO + H2) is prepared when steam is passed over red hot coke. Q2.
Match the following List-1 and List-2: (1) Philospher wool
(2) Green vitriol
(3) Marble
(4) Blue vitriol
Chemical Formulae:
(A) ZnO
(B) CaCO3
(C) FeSO4 . 7H2O
(d) CuSO4 . 5H2O
Code:
AB C D (a) 1324 (b) 4213 (c) 2413 (d) 1234 Ans. – (a)
Philospher wool – ZnO Marble – CaCO3
Green vitriol – FeSO4 . 7H2O Blue vitriol – CuSO4 . 5H2O Q3.
Demagnetisation of magnets is effected by:
(a) Heating
(b) Magnetising in the opposite direction (c) Rough handling
(d) All of the above
Ans. – (d) If a coil wrapped on a magnet and an alternating current is
passed through the coil, the magnet is demagnetised. It can be
demagnetised by holding it in E-W direction. The rough handling of a
magnet also demagnetised it. If a magnet heated and then
hammered its magnetism is destroyed. If two similar magnet are
placed in such a way that their likes pole are near each other and
they becomes weak strength.
Q4. Diamagnetic substances differ from the para-magnetic and
ferromagnetic substance in which one of the following ways:
(a) They settle with long axis normal to
the magnetic field.
(b) Diamagnetic substances are repelled
by magnet.
(c) Diamagnetic substances are feebly
attracted by a magnet.
(d) Both (a) and (c).
Ans. – (d) Diamagnetic substances tend to move from stronger to
weaker parts of the magnetic field. Diamagnetic substances are
bismuth, copper, mercury, antimony gold, water, alcohol, air and
hydrogen. When a diamagnetic material is suspended sets itself at
right angle to the direction of the field. The concentration of magnetic
lines of force is more outside the specimen. Q5. Which one of the
following is essential differences between AC dynamo and DC
dynamo:
(a) AC dynamo has a coil wound on soft
iron but the coil in DC dynamo is
wound on copper.
(b) AC dynamo will generate a higher
voltage.
(c) AC dynamo has shp ring but DC
dynamo has a commutator.
(d) None of the above.
Ans. – (c) The DC dynamo has a commutator or current reverie. In
AC dynamo the end of Armature coil are connected then insulated
ship ring which are mounted on the armature shaft. So in above
option (c) is essential difference between AC and DC dynamo.
Q6. Which one of the following is correct about Canada balsam:
(a) It is type of jet engine.
(b) It is a device to store electric charge. (c) It is a protein deficiency
syndrome. (d) It is a transparent adhesive for glass. Ans. – (d)
Canada balsam is a transparent adhesive for glass. Having the same
refractive index as glass. It is used for cementing together precision
optical element so option (d) is correct about Canada balsam. Q7.
There are streaks of colour on a car windshield shortly after it
has been swiped by a windshield wiper or a squeeze at a gas
station. It result from which one of the following:
(a) Dispersion of light
(b) Scattering of light
(c) Absorption of heat
(d) Interferences of light
Ans. – (d) These streaks of colour are the result of interferences of
light by the very thin film of oil that is spread over the water surface.
This from the interference is commonly called thin film interference.
When light wave from two independent, near by a coherent sources
or LASER beam is interact with each other. They may either destroy
or strengthen each other giving rise to alternate dark and light band.
Q8. Which one of the following required for converting Ammeter
into Voltmeter: (a) A low resistance either in series or
parallel.
(b) High resistance in series and removal
of shunt.
(c) Low resistance in parallel.
(d) Low resistance in series.
Ans. – (b) A voltmeter is always connected across that part of circuit
where potential difference is to be measured. In order that the
insertion of a voltmeter in the circuit does not change the current. The
voltmeter must have high resistance since the coil of the instrument
does not have a large resistance of its own. To convert an Ammeter
into voltmeter high resistance is series and removal of shunt required.
Q9. On which one of the following hydrogen bomb is based:
(a) Controlled fusion
(b) Uncontrolled fusion
(c) Controlled fission
(d) Uncontrolled fission
Ans. – (b) In the hydrogen bomb fusion of light nucleus take place
and tremendous amount of energy released. In this part bomb fission-
fusion process is applied. The central part contains a fission bomb
containing uranium-235 or plutonium-239. Q10. The specific
resistances of a wire depends upon which of the following: (a)
Mass
(b) Cross section
(c) Length
(d) Nature of the material of wire Ans. – (d) The specific resistance of
a wire do not depend upon mass, cross section and length of wire. It
will depend only on the nature of the material of the wire so option (d)
is correct answer.
Q11. Consider the following statements and select correct code:
(1) The hammering decreasing the
agitation of the molecules.
(2) When a magnet heated and hammered
it magnetised is destroyed.
Code:
(a) 1 and 2 are correct
(b) Only 1 correct
(c) Only 2 correct
(d) 1 and 2 both are incorrect
Ans. – (c) When a magnetic is heated, the agitation of its molecules
increases not decreases. The hammering further increasing it and
when we hammered magnet the magnetised is destroyed.
Q12. Which one of the following statement is incorrect:
(a) Human eyes respond best in yellow
light.
(b) The largest wavelength we can see is
deep red about 700 nm.
(c) White light is a mixture of all the
colour in visible spectrum.
(d) None of the above.
Ans. – (a) Except option (a) all of the above option is correct because
human eyes is respond best to green light at 550 nm which is
approximately the brightest colour in sunlight at the surface of the
earth.
Q13. Consider the following statement and select the correct
code:
(1) All electromagnetic wave have the
same velocity in space.
(2) Light wave and radio wave are
actually electromagnetic wave which
differ only in their frequencies. Code:
(a) Both 1 and 2 correct
(b) Only 1 correct
(c) Both 1 and 2 are incorrect
(d) Only 2 correct
Ans. – (a) All electromagnetic wave have equal velocity in space.
Light wave and radio wave may seem to be very different, but
actually they are the same thing. The electromagnetic wave have
equal velocity but different frequencies. The velocity of
electromagnetic wave is 3 × 1010 cm/sec. Q14. The equation Zn +
2nH4Cl → 2NH3 + ZnCl2 + 2H+ + Ze represents the equation for
which one of the following cell: (a) Leclanche cell
(b) Dry cell
(c) Standard cell
(d) None of the above
Ans. – (a) In Leclanche cell carbon rod and zinc rod act as positive
and negative poles respectively. The solution is that of ammonium
chloride. So from the above option (a) is relates with question. Q15.
Which among the following is a poor conductor of electricity:
(a) Carbon (b) Dry wood
(c) Paraffin (d) None of these Ans. – (b) Dry wood, moist wood,
leather, moist earth, tap water, rubber, clean water, etc. are poor
conductor of current. Glass, mica, paraffin, rubber and amber are
insulator.
Q16. Two resistance are joined in parallel whose resistant is 6/5
ohms. One of the resistance wire is broken and effective
resistance becomes 2 ohms. The resistance in ohms of the wire
that broken is: (a) 4 (b) 9/5
(c) 3 (d) 6/5
Ans. – (c) Here we noted that
5/6 = 1/X + 1/2
So X = 3 ohms
Q17. If a train moving very fast with the velocity of light, its
length would be: (a) Half of the original
(b) Zero
(c) Infinite (d) Unchanged
Ans. – (b)
We noted that
Here L = L0√1 − V2/C2
as V = C
Then L = L0√1 − 1/1
L= 0
Q18. If a magnet is left to itself which one of the following option
would happen: (a) Its magnetic effect would first get
strengthened and then weakened. (b) There would be no change in
its
magnetic properties.
(c) Its magnetic effect would be
unchanged.
(d) It would have demagnetising effect. Ans. – (d) If a magnet left to
itself, it has demagnetising effect upon itself due to indiction of their
own poles through the material of the magnets. So option (d) is
correct.
Q19. Which one of the following is spectrum of carbon
monoxide:
(a) Continuous spectrum
(b) Band spectrum
(c) Fluorescence spectrum
(d) None of the above
Ans. – (b) Light emitted by chemical compounds in vapour states
give Band spectrum. Therefore, spectrum of carbon monoxide will be
Band spectrum. So in above option (b) is correct about spectrum of
carbon monoxides.
Q20. The same notes is being played on sitar and veena. In
which of the following they would differ:
(a) Pitch (b) Quality
(c) Both of the true
(d) None of the ture
Ans. – (b) From the above option (b) is correct. As sitar and veena
are being played for the same note, frequency is the same but the
sound produced from the both are different tunes. Therefore they
different in quality.
Q21. Snell’s law is related to which one of the following:
(a) Velocity of sound
(b) Velocity of light
(c) Reflection
(d) Refraction
Ans. – (d) The Snells law state that the product of the refractive index
and sine of the angle of incidence of ray in one medium is equal to
product of the refractive index and sine of the angle of refraction in a
successive medium. Also the incident ray, and the normal to the
boundary at the point of incidence all lies in the same plane. Q22.
Many creatures from bacteria to fish make light by
chemiluminescence (chemical reaction producing molecules
with electron in existing energy level that can radiates light). The
manufacturing substances in this case in: (a) Surface tension (c)
Emilase
(b) Luciferase (d) Valinifine Ans. – (b) When chemiluminescence
occur in plants or animal it is called bio luminescence. Many creature
from bacteria to fish make light this way by manufacturing substance
is called luciferase and luciferin. Luciferase help luciferin combine
with oxygen and resulting reaction creates existing molecule that emit
light. Q23. Consider the following statements and select the
correct code:
(1) Resonance can be used to determine
the velocity of the sound at room
temperature in the laboratory. (2) In Resonance vibration with very
large amplitude can be set up. (3) A Reasoning column of air contain
transverse progressive wave. Code:
(a) 1, 2 and 3 (b) Only 1
(c) Only 3 (d) 1 and 2
Ans. – (d) In a reasoning air column, stationery longitudinal wave are
formed. Resonance occurs when a body is made to vibrate with its
own natural frequency due to impressed periodic force of a body
which is also vibrating with same frequencies.
Q24. Two sources of wave are coherent if: (a) Both produce waves
of same
wavelength having constant phase
differences.
(b) Both have same amplitude vibration. (c) Both produce waves of
same
wavelength.
(d) Both produce waves having same
velocity.
Ans. – (a) According to question option (a) is correct match with two
sources of wave are coherent. Two sources are said to be coherent if
they emits light wave of the same frequencies (same wavelength)
nearly the same amplitude and are always have constant phase
difference.
Q25. Consider the following statements: (1) Isotopes are atoms of
same elements. (2) Isotopes have same atomic numbers. (3) Isobars
have different atomic numbers.
(4) Isochoric is a thermal process when volume is constant.
Choose the correct options:
(a) 1 and 4 (b) 2 and 3
(c) 2 and 4 (d) All of the above Ans. – (d) Atoms of same elements
having same atomic number and different atomic mass is known as
Isotopes. Ex. : C12 & C14 Isobars – Atoms of different element having
same atomic mass and different atomic no. is known as Isobar.
Isochoric – It is a thermodynamical process in which volume of the
system remains constant i.e. (DV = 0). In Isochoric process work
done is equal to zero (PDV = 0).

Heat and Thermodynamics

Q1. According to Kepler’s law of planetary (a) motion, the square


of planets is propor(b)tional to: (c)(a) The cube of their mean distances from (d)
the sun. Ans. – (d) A rocket works on conservation(b) The mean distance from
the sun. of linear momentum. The operation of a(c) The square of their distances from the
rocket illustrates the conservation of linearsun. momentum. Just before launching the
(d) None of the above. momentum of rocket is zero. When rocket

Ans. – (a) Kepler formulated that three fired, it forces a jet of hot
gasses with a basic laws of planetary motion: velocity through nozzle.
The jet of gases (1) Every planet moves in an elliptical acquires a momentum
downward. Hence,

orbits with the sun being at one of its the rocket acquires a momentum of equal
face.
magnitude in opposite direction and then
rocket moves upward. (2) The radius vector drawn from the sun Q4. Consider the
following statements andto the planets sweep out equal
areas in equal intervals of time.

(3) The square of the time period of


revolution of a planet around the sun
proportional to the cube of semi major
axis of the orbit.

Q2. Which one of the following is not prop


erty of Radiants heat:
(a) Radiant heat does not obey the law of

Reflection.
(b) Radiant heat can be transmitted
through vacuum.
(c) Radiant heat transmitted in straight
line.
(d) None of these.
Ans. – (a) The Radiant heat obey the laws
of reflection using a thermopile, we can
detect that the reflected heat radiation from
a plane polished surface obey the law of
Reflection (i.e. the angle of incidence is
equal to the angle of Reflection. The inci
dence heat radiation, the reflect heat radia
tion and the normal to the polished surface
at the point of incidence are in the same).
Q3. A rocket works on which one of the folQ5.
lowing:
select the correct code: (1) Ice is a bad conductor of heat. (2) The
volume of water is minimum at
4°C.
(3) If the expansion of water had not been
anomalous, the ponds and lakes
would have frozen from top to bottom
and all the animal living in water
would have died. Code:
(a) All 1, 2 and 3 are correct (b) Only 2 and 3 are correct (c) 3 and 1
are correct (d) 1 and 2
Ans. – (a) On being cooled, water contract upto 4°C. If it is further
cooled, it begins to expand instead of contracting. In winter the water
on the upper surface of the ponds and the lakes freezes, but below
the frozen layer water remain at 4°C which has maximum density.
The surface layer being lighter, float on it. The aquatic animals can
live in water 4°C below the frozen layer. Ball pen functions on
which one of the following principle of physics:

Conservation of angular momentum. Conservation of mass.


Conservation of energy.
Conservation of linear momentum. (a) Surface tension
(b) Viscosity
(c) Gravitational force
(d) None of these
Ans. – (a) The ball pen functions on the principle of capillarity and
surface tension. Rise of oil in the wicks of the oil lamps, rise of sap in
plants, rise of ink in the narrow slit of the pen and soaking up of ink in
the blotting paper are all example of capillarity.

Q6. The general rule is that when liquid solidifies its volume
decreases and when solids liquefies, its volume increases.
Which one of the following when converted into liquid decreases
in volume: (a) Brass (b) Ice
(c) Wrought Iron (d) All the above Ans. – (d) When wrought iron, ice,
brass etc. are converted into liquid their volume decreases. Ice is
seen to fleet of water. This mean that the volume of the given mass of
ice is greater than the volume of some mass of water.

Q7. Which one of the following substances has the highest heat
capacity:
(a) Ice (b) Paraffin
(c) Water (d) None of these Ans. – (c)
Specific heat capacity
of water = 4200 J/kg °C Specific heat capacity
of paraffin = 220 J/kg °C Specific heat capacity
of Ice = 2108 J/kg °C Specific heat capacity
of glycerine = 2436 J/kg °C In the given option water has highest
specific heat capacity.

Q8. Consider the following statements and select the correct


code:
(1) The high temperature of sun and stars

are sufficient to bring about the fusion of hydrogen nuclei to form


nuclear helium with tremendous release of energy.

(2) Carbon acts as a catalyst for this reaction.


Code:
(a) Only 2 is correct
(b) Only 1 is correct
(c) Both 1 and 2 are incorrect
(d) Both 1 and 2 are correct
Ans. – (d) For the hydrogen atom fuse
together in the presence of carbon, which
acts as a catalyst give one atom of helium
with little loss mass of and release of energy.
Q9. Weightlessness experienced while orbiting the earth is a
result of which of the
following:
(1) Acceleration
(2) Zero gravity
(3) Centre of gravity
(4) No result
Code:
(a) 1 and 2 (b) 2 and 3
(c) Only 4 (d) Only 1
Ans. – (d) When a object/person in spaceship is orbiting the earth,
the necessary
centripetal acceleration provided by acceleration due to gravity (g).
Q10. Which one of the following would hold
true if a gymnast sitting on a rotating
stool, with his/her arms outstretched,
suddenly lower her/his arms:
(a) Angular velocity remain constant.
(b) The angular momentum decreases.
(c) The angular momentum increases.
(d) Moment of inertia decreases.
Ans. – (d) For the above condition the
moment of inertia of gymnast decreases
but the momentum of the body is remain
constant. The angular velocity of the body
is increases not remain constant.

Q11. If an iron ball and wooden ball have same radius are
released from the same height in a vacuum. The time taken by
both ball to reach the ground is: (a) Zero
(b) Unequal
(c) Iron ball take minimum and wooden

ball take maximum time.


(d) Equal
Ans. – (d) In the question given that the radius and the height are
same and also as the both ball fall in the vacuum, therefore the time
taken by both ball is exactly equal. When ball fall from a height time
cannot be zero.

Q12. An object projected upward with velocity of 100 m/s. It


approximately what time will ball strike the ground? (a) 15
second (b) 20 second (c) 35 second (d) 10 second Ans. – (b) When
a body projected upward from the earth then V = u + gt.

here V = – 100
[∵ ball goes upward]
g = 9.8 m/s

[acceleration due to gravity]


so V = u + gt
– 100 = 100 + 9.8 × t
t = 20 approx.
Q13. The length of a second’s pendulum on earth (980 cm/s2) is
100 cm. The length of it on a planet where g = 490 cm/s2 would
be:
(a) 25 cm (b) 75 cm
(c) 300 cm (d) 50 cm
Ans. – (d)
Here, t = 2π√llg... for second’s pendulum t = 28

l1= l2
g1 g2
l1 × g2
= 100 × 490 g1 980
l2 =
l2 = 20 cm

Q14. Consider the following statements and select the correct


answer/code:
(1) Those substances which expand on

freezing have their melting point lowered by the increases of


pressure, whereas those substance which contract on freezing have
their melting point raised by the increase of pressure.

(2) The phenomenon that ice melt on application of pressure and


again solidifies when the pressure released is known as regelation.

Code:
(a) Only 1 is correct
(b) Only 2 is correct
(c) Only 1 and 2 are correct
(d) Neither 1 nor 2 are correct
Ans. – (c) The substances that expand on freezing are ice, bismuth,
antimony, wrought iron etc. whereas substances which contract on
freezing include wax, copper, etc.

Q.15 Which one of the following represents the boiling point of


water in Rankine scale:
(a) 272° (b) 168°
(c) 672° (d) 373°
Ans. – (c) In the Rankine scale, the boiling point of water is 672° and
freezing point is 492°, in Fahrenheit scale it is 212° to 32°, in
centigrade scale 0° minimum and 100° maximum. In Reaumur scale
it is 80° and 0° and in Kelvin scale 373° to 273°, respectively. The
relation between centigrade, Fahrenheit and Reaumur scale is

C = F − 32 = R
594

Q16. Consider the following statements and select the correct


code:
(1) Sphygmomanometer is a device used

to measure blood pressure.


(2) Manometer is a device for measuring differences in pressure
usually by noting the differences in the height of two liquid column.

Code:
(a) Only 1 is correct
(b) Only 2 is correct
(c) Both 1 and 2 are correct
(d) Neither 1 nor 2 are correct
Ans. – (c) A simple manometer is U-tube device form by bending a
glass tube in the form of the letter U. If a pressure to be measured is
fed one arm of the U-tube and other kept open to the atmosphere, the
difference in the level of the liquid in the two arms gives a measure of
the unknown pressure.
Q17. A cyclist turn round a curve at 15km/hour. If he/she turn at
double the speed, the tendency of overturn is: (a) Unchanged (b)
Quadrupled (c) Tripled (d) None of these Ans. – (b) If the cyclist turn
at double of the speed the tendency of overturn is quadrupled. We
noted that

tan
θ

=
V2
rg

Therefore, when speed is doubled the tendency to overturn (2)2 = 4,


i.e., quadrupled. Q18. Which one of the following is the
temperature at which the Celsius and the Fahrenheit reading are
numerically equal but opposite in sign:

(a) 69° (b) 80°/9


(c) 97° (d) 80°/7
Ans. – (d)
We let X be the requiring reading X°f = – X°C

X − 32 = −X
95

or (X – 32) × 5 = – 9X
or 5X – 160 = – 9X
14X = 160
X = 80/7

Q19. When the moon passes between the sun and earth, solar
eclipse occurs. People who are on the surface of earth just
below the top of the moon’s Umbra will see which one of the
following:
(a) Diamond ring effect only
(b) Annular solar eclipse
(c) Total solar eclipse
(d) None of the above
Ans. – (b) These on the part of the earth, which is in Penumbra a
partial solar eclipse is seen and to the observer on the part of earth
which passes through umbra annular solar eclipse is visible.

Q20. In which one of the following has the lowest refractive


index:
(a) Quartz (b) Diamond
(c) Ice (d) Water
Ans. – (c) The refractive index of given option is follow:
Quartz = 1.5
Diamond = 2.40
Ice = 1.31
Water = 1.33
Air = 1.00

Q21. A gradual increase in the barometric height indicate which


one of the following:
(a) Nano meter
(b) Modern barometer
(c) Aneroid barometer
(d) None of these
Ans. – (c) A gradually increases in the barometric height indicated by
Aneroid barometer. Aneroid barometer is form of a barometer in
which no liquid is used. It is light and portable. It can be taken to
higher place.

Q22. Pressure at any point of a liquid increases with which of


the following:
(a) Density of the liquid.

(b) Depth of the point from the free surface.


(c) Acceleration due to gravity.
(d) All of the above.
Ans. – (d) Pressure at any point of a liquid
increase with all of the above options. In
atmospheric pressure acting on a free surface of a liquid is also taken
into account
then total pressure at any point of a liquid
is a given by hag + atmospheric pressure. Q23. Lactometer is a
form of hygrometer used
to measure the purity/quality of milk. If
in milk the lactometer sinks up to the
mark of 1, which one of the following
conclusion can be drawn:
(a) The milk is 100% pure.
(b) The milk in the liquid 1/3rd of it.
(c) The milk in the liquid 1/4th of it.
(d) None of these.
Ans. – (c) The milk and water, milk is
heavier than water, the lactometer sinks
more in water than milk. If it sink up to
mark 3, it means that the milk in the liquid
is 75%. If it sinks up to mark 2 it is 50%
pure and if it sink up to 4 it is 100%. In the
question it sinks up to 1 so it is 25% pure. Q24. A force of 800
dynes acts on a glass ball
placed on a smooth surface. If the mass
of the ball is 250 g, what distance will
travel it in 12 seconds?
(a) 315 cm (b) 430 cm
(c) 230.4 cm (d) 500 cm
Ans. – (c) Mass = 250 g, f = 880 dynes f = M × a, or a = f/M

a = 880 = 16 cm/sec2
250
t = 12 second, u = 0
S = ut +
1
a+2
= 0 × 12 × 1 × 16/5 × 12 × 12
2
S = 0 + 8 × 144 = 1152
55
or S = 230.4
Q25. Which one of the following is the reason for the moon
having no atmosphere: (a) The value of gravitational pull is not

uniform on moon.
(b) There is no plants and population on
moon.
(c) RMS velocity of all gases is more
than the escape velocity from the
moon’s surface.
(d) None of these.
Ans. – (c)
The value of g is less on the moon in comparison to earth. The
escape velocity is less on the moon’s surface than the RMS velocity
of the all gases. Due to this reason gases have escaped the moon
surface and results no atmosphere on the moon.

Wave and Optics

Q1. A type of wave disturbance in which wave particle vibrates


along the direction of wave movement is known as: (a) Stationary
wave
(b) Progressive wave
(c) Electromagnetic wave
(d) Longitudinal wave
Ans. – (d) Wave is a disturbance. It has energy and momentum and
propagates from one point to another point. It has certain frequency
and wavelength.
It is of two types (1) Transverse waves (2) Longitudinal waves.
Transverse wave – Medium particle vibrates perpendicular to the
direction of propagation of wave.
Longitudinal wave – Medium particle vibrates along the direction of
wave movement.

Q2. Match the following List-1 and List-2: List-1


(Indian Physicists)
1. S.N. Bose 2. C.V. Raman 3. S. Chandrashekhar 4. H.J. Bhabha
List-2
(Contribution)
A. Evolution of stars
B. Quanthem statistics
C. Cosmic radiation
D. Inelastic scattering of light by

molecules
Code:

AB C D
(a) 3142
(b) 1234
(c) 1324
(d) 3124
Ans. – (a)
S.N. Bose – Quanthem statistics

C.V. Raman – Inelastic scattering of light by molecules

H.J. Bhabha – Cosmic radiation S. Chandrashekhar – Evolution of


stars

Q3. Hepatitis ‘B’ vaccine is categorised as: (a) Interferon


(b) Second generation vaccine
(c) First generation vaccine
(d) Third generation vaccine
Ans. – (b) First generation vaccines produced by conventional
technique. It’s suc

cess rate is low and not safer. Second generation vaccines are better
and safer vaccine, they consists of a few antigens present on the
surface of the infecting agents rather than whole inactivated
organism. Hepatitis ‘B’ is caused by virus. In this disease liver is
affected. One of the main cause of Hepatitis ‘B’ is contaminated
water.

Q4. Which of the following statements is correct:


(a) Wave velocity is the product of
wavelength and frequency.
(b) The sharpness of a musical tone
depends on its pitch.
(c) Wavelength of sound wave is
inversely related to frequency. (d) Wave energy depends on
frequency of
the wave.
Code:
(A) a and b (B) b and c
(C) a, b and c (D) All of the above Ans. – (D) Wave velocity (C)
= Wavelength × frequency
C=λ×v

λ =Cv

Wave frequency is inversely related to wavelength i.e. frequency


increases wavelength decreases.
The sharpness of a musical tone depends (c) 3241
on its pitch. More the pitch (frequency), (d) 1432more will be sharpness. Ans. – (b)Energy of
wave (E) = hv Magnifying glass – Double convex lensev = frequency Galilian telescope –
A convex and a conh = Planck’s constant
cave lense
h = 6.67 × 10–34 J-sec.

Q5. Automatic door open are usually trig


gered by sensor which sense the pres
ence of a person near the door. These
sensor sense which of the following to
activate the opening of the door:
(a) Pressure
(b) Interruption of a beam of visible

infrared light.
(c) Change in the magnetic field.
(d) a and b
Ans. – (d) Basically two types of sensor,
which either sense pressure or interruption
of a beam of visible or infrared light. In the
beam sensor type door opener the source of
the beam and the detector are placed oppo
site to each other in the passage ways in
front of the door in a such way that the
beam crosses the passage ways. If the sen
sor is of the pressure type it is placed below
a movable platform in front of the door.

Q6. Match the List-1 to List-2 and select the


correct code:
List-1
A. Magnifying glass
B. Galilian telescope
C. Compound microscope
D. Terrestrial telescope
List-2
1. Two convex lense
2. Three convex lense
3. A convex lense and concave lense
4. Double convex lenses
Code:
Compound microscope – Two convex lense
Terrestrial telescope – Three convex lense

Q7. Rain sensor fitted on the windshield wiper of some


automobiles are based on the principle:
(a) Heat radiation
(b) Total internal reflection (c) Surface tension
(d) All of the above
Ans. – (b) Rain sensor fitted on windshield wiper of automobiles are
based on total internal reflection. This case of infrared light/spectrum
falls onto windshield from the inside of the automobile when the drop
of water appear on the glass, the reflected light is less. On sensing
this the wiper on the windshield start functioning.

Q8. When a pigments are mixed, each one absorb certain


colours or each one substract from white light and resulting
colour is depend on the light waves that are not absorbs.
Primary pigments are the complements of three primary colours.
Consider the following statement and select correct code: (1)
When three primary pigments are

mixed, we get the black colour. (2) Cyan is the complement of green,
magenta the complements of blue and
yellow the complements of red. Code:
(a) Only 2 is correct
(b) Only 1 is correct
AB C D
(a) 1 2 3 4
(b) 4 3 1 2
(c) 1 and 2 both are correct (d) Both are false
Ans. – (b) The cyan is the complements of red, magenta is the
complement of green and yellow is the complements of blue. When
three primary colours/pigments (cyan, magenta and yellow) are
mixed in proper proportion all the colours are substract from white
light and his mixture is black. Q9. The surface temperature of
which of the following planets according to latest researches is
higher than that of any other planets and hotter than most
ovens:
(a) Mercury (b) Pluto
(c) Uranus (d) Venus
Ans. – (d) The surface temperature of venus planet is about 462°C.
The sun’s radiants energy filter into the planets atmosphere. The
large droplet of sulfuric acid (H2SO4) is present venus cloud and the
great quantity of CO2 in the atmosphere seem to trap of solar energy
of the planets surface.
Q10. Consider the following statements and select the correct
code:
(1) The near point of human eyes
decreases with age.
(2) Presbyopia is caused due to ageing
when it becomes very difficult to
change the lense of curvature. Code:
(a) Only 2 is correct
(b) Both 1 and 2 are correct
(c) Only 1 is correct
(d) Both 1 and 2 are incorrect
Ans. – (a) A standard near point distance of 25 cm is typically
assumed in the design of optical instruments and characterizing
optical deviced such a magnifying glasses. Without correction, the
near point is 3 inches (7 cm) at age of 10, 6 inches (16 cm) at age of
40 to 39 inches (1 meter) at age of 60. As a result a 60 year old must
use corrective lense to read books. Presbyopia is a caused in the
advance stage when muscle become flexible.
Q11. When an eye fails to see vertical and horizontal lines
equally distinctly, it is said to be a suffering from which one of
the following: (a) Presbyopia (c) Flatter lens (b) Adermatoglyphia (d)
Astigmatism
Ans. – (d) When the person suffering from astigmatism, rays of light
from the horizontal and vertical lines in a plane are not focussed in
the same plane on the image. This defect is caused when the
curvature of the cornea becomes different in different axial plane.
This defect is corrected by using the spectacles with cylindrical
lenses. Q12. In the making of leather, which of the following is
involved in the process of tanning:
(a) Lubrication and aesthetic appeal. (b) Finishing to obtain proper
thickness. (c) Displacement of water and
combination with fibrous protein,
collegn.
(d) Removal of hair, flesh and fat. Ans. – (c) The making of leather is
involve in the process of tanning is displacement of water and
combination with fibrous, protein and collegn. Tanning increasing
resistance to heat and decomposition caused by water and micro
organism. So option (c) is correct.
Q13. Total internal reflection occur when light passes from:
(a) Rarer to denser
(b) Denser to rarer
(c) It occurs irrespective of medium (d) It is same as refraction
Ans. – (b) TIR occurs when light passes from denser to rarer medium
and angle of incidence <i is more than critical angle <ic. Through this
condition light does not pass to another medium. In optical fibre
communication TIR principle is used. It has greater use in medical
science like endoscopy. If <i> <ic then 100% reflection occurs. Q14.
Which one of the following is used in fingerprints on a piece of
paper:
(a) b-rays
(b) Ultraviolet radiation
(c) Gamma-rays
(d) X-rays
Ans. – (b) The ultraviolet radiation is used in finger print of any
object/person on a piece of paper. Finger print on a piece of paper
may be detected by sprinkling fluorescence powder and then
examining it in the ultraviolet radiation/light.
Q15. Which one of the following is true about total reflection of
prism:
(a) One angle of prism is 180° and other two are of 0° each.
(b) Each angle of prism is of 60°. (c) One angle of prism is 90° and
other two are of 45°.
(d) One angle of prism is 80° and other two are of 50° each.
Ans. – (c) About the total reflection of prism one angle of prism is 90°
and other two angle are 45° each. The total reflection are used in
optical instruments such as microscope, telescope, spectroscope,
prism binoculars etc.
Q16. Consider the following statements and select the correct
code:
Which of the following is/are not a property of Infrared
spectrum:
(1) They have large wavelength. (2) They eject electrons from metals
on which they falls.
(3) They show heating effect.
(4) They are observed by the glass but not by rock salt.
Code:
(a) 1 and 4 (b) Only 2
(c) 2 and 3 (d) 1, 2, 3 and 4 Ans. – (b) Except code (b) all are
property of infrared spectrum. The eject electrons from metal on
which they fall is not property of infrared spectrum because it is
property of ultraviolet radiation.
Q17. The angle of incidence for which the angle of refraction is
equal to 90° is know as critical angle. Which of the following has
largest critical angle: (a) Glycerine (b) Diamond (c) Water (d) Crown
glass Ans. – (c)
Critical angle of crown glass is 41°25’ Critical angle of water is 48.5°
Critical angle of diamond is 42°25’ Critical angle of glycerine is 44°30’
Critical angle of paraffin is 44°12’ Q18. Which one of the following
concave mirror is not used:
(a) Rear-view mirror
(b) Reflector in search light
(c) Dental used
(d) None of these
Ans. – (a) The concave mirror is not used in rear-view mirror. The
convex mirror used in rear-view mirror. It is also used in street lamps
as a reflector to diverge the light over a large surface. Reflector of
search light dental used of the concave mirror.
Q19. Which one of the following instruments is used in a trench
to view what is the happening outside:
(a) Spectrometer (b) Clinostate (c) Simple periscope (d) None of
these Ans. – (c) The simple periscope to used to witness the game
by a person sitting in back row over the heads of the observer can
seen wether a ship is coming while lying in deep sea. It is consist a
long tube with opening at the top and the bottom and two plane mirror
fixed at an angle 45° to this axis of the tube such that the mirror are
parallel to each other.
Q20. Ear of a beat is an example of which one of the following
options:
(a) Lever of the fourth order
(b) Lever of the fifth order
(c) Lever of the second order
(d) None of these
Ans. – (c) Ear of a beat is an example of lever of the second order. In
lever of second order, lead is situated between effort and fulcrum.
Nut, cracker, wheel borrow, ear of a beat, etc. are lever of the second
order.
Q21. Which one of the following measure the intensity of
earthquakes:
(a) Stevenson scale
(b) Mercalli scale
(c) Ritcher scale
(d) All of the above
Ans. – (b) The severity of an earthquakes can be measured either by
measuring its magnitude or intensity. Magnitude is measure of
strength of earthquakes at its sources. It is assessed on 8-point scale
called Ritcher scale. Intensity is a measure of the local effect caused
by an earthquakes which varies according to distance from the
sources. The intensity is measured in Mercalli scale.
Q22. Which one of the following is most suitable reason for
breaking up of the lizard’s tail:
(a) It is a defence mechanism.
(b) Scaly tail is shed off as a periodically
routine.
(c) (a) and (b)
(d) None of these.
Ans. – (a) The breaking up of the tail of lizard’s is a defence of
mechanism which helps the lizard’s to run away unharmed

from the attacker. The broken part continue to turn and twist for while
thus detracting at attackers attention. The movements last till the
energy stored in the tail muscle get exhausted.

Q23. Consider the following statements: (1) Light of longer


wavelength is scattered much more than the light of shorter
wavelength.

(2) The speed of light is maximum in vacuum.


(3) Red light has larger wavelength with respect to cosmic rays.
(4) Radio waves are produced by rapidly oscillating currents.
Which of the statements is/are correct.
(a) 2 and 3 (b) 1 and 4
(c) 2, 3 and 4 (d) All of the above
Ans. – (c) Light is an electromagnetic
wave. It has dual character. Light is a mixture of 7 colour. Speed of
light is maximum
in vacuum (3 × 108 m/s). Red light has larger wavelength in
comparison to cosmic ray.
Larger wavelength light scattered less.
Radio waves are produced by oscillating
currents.
Q24. Match the following List-1 and List-2:
List-1
(1) Henry (2) Tesla
(3) Newton/Coulomb (4) Ampere
List-2
(a) Inductance
(b) Magnetic field
(c) Electric field intensity
(d) Electric current
Code:
1234

(A)ab cd (B) a b d c (C) ad cb (D) a c b d Ans. – (A)


Henry – Inductance Magnetic field– Tesla
Newton per coulomb
Ampere
– Electric field intensity
– Electric current

Electricity

Q1. The tail of comet is directed away from the sun due to:
(a) The tail of comet always exists in the

same orientation.
(b) Radiation emitted by sun exerts a
radial pressure on the comet, throwing
its tail away from the sun.
(c) Comet rotates around the sun, lighter
mass of the comet is pushed away due
to centrifugal force.
(d) As the comet rotates, lighter mass of
comet is attracted by stars situated in
the direction of its tail.
Ans. – (c) Comet is a celestial bodies which moves around the sun. It
is made up by gas, dust particles, ice etc.
Tail of comet is directed away from the sun due to centrifugal force.
Centrifugal force always acts away from the centre when bodies is in
circular motion. Examples – Tumble – 1, Helly.
F = M V2
cf
r

Q2. Assertion (A): Temperature increases when electric current is


passes through metal.
Reason (R): Collision of metal atoms with each other produces heat
energy.
Codes:
(a) Both A and R are true but R is not

correct explanation of A.
(b) A is true but R is false.
(c) A is false but R is true.
(d) Both A and R are true and R is correct explanation of A.
Ans. – (d) The heating effect of an electric current is caused by
collisions of electrons with atoms of material. In these collisions, the
energy of electrons is lost to positive ions.
Metals have positive temperature gradient. It resistance increases
when temperature increases.

Q3. Which of the following statements are correct about gold


ornaments:
(1) Cadmium is used as a solder in

making gold ornaments.


(2) KDM in the gold ornaments stands for
cadmium.
(3) Cadmium is the cause of lung cancer
in Artisans and Fabricators.
(a) 2 and 3 (b) 1 and 2
(c) 1 and 3 (d) All of these Ans. – (d) KDM stands cadmium. The
finish of gold ornament depends upon the quality of its soldering
material. Cadmium has been proven to expose artisans and
fabricators to lung cancer and other respiratory disorders.
Q4. Consider the following statements: (1) Wavelength of red light
is less than
radio waves.
(2) Frequency is the fundamental
quantity of waves.
(3) A widely used musical scale called
diatonic scale has seven frequencies. (4) The frequency of the note
sa (sa) is
256 HZ and that of Ni 512 HZ. Which of the statements are
correct? (a) 1 and 2 (b) 2 only
(c) 1, 2 and 3 (d) 1, 2, 3 and 4 Ans. – (c) Radio waves < Micro waves
< Infrared < Visible light
decreasing wavelength/increasing frequency
* Frequency is most fundamental
quantity.
* The frequency of each swar is
different that’s why we can identify.
sa – 240 HZ
Re – 270 HZ

Q5. Which one of the following types of waves are used in a


Night vision apparatus:
(a) Radio waves
(b) Infrared waves
(c) X-rays
(d) Micro waves
Ans. – (b) Night vision is the ability to see in a dark environment by
biological or technological means.
Night vision device is a device containing infrared image intensifier
tube in a rigid casting, commonly used by military forces. It has
greater application in film technology like, Night vision filming and
photography.

Q6. Indiscriminate disposal of used fluorescent electric lamps


causes mercury pollution in the environment why mercury used
in manufacture of these lamps? (a) When the lamp is on, the
mercury in
the lamp causes the emission of UV radiation.

(b) A mercury coating on the inside of the lamp makes the light bright
white.
(c) Mercury converts UV light into visible light.
(d) None of the above.
Ans. – (a) There are two main parts in a
CFL (1) the gas filled tube (also called bulb
or burner) and the magnetic or electronic
ballast. An electrical current from the ballast flows through the gas
(mercury
vapour) causing it to emit UV light. The
UV light excites the phosphorus coating. It
emits visible light.
Q7. Which of the followings are correct:
(1) Steam at 100°C and boiled water at 100°C contain same amount
of heat.
(2) Latent heat of vapourisation of water is not equal to the latent heat
of fusion of ice.
(3) In AC, heat is extracted from the room air at the evaporator coils
and is rejected out at the condenser coils.
Which of these statements are correct?
(a) 2 and 3 (b) 1, 2 and 3
(c) Only 2 (d) None of these
Ans. – (a) Latent heat of vapourisation of
water is greater than latent heat of fusion of
ice.
Lf(ice) = 80 cal/g
Lv(water) = 536 cal/g
Q8. When water will be heated from 0°C to
10°C, then volume:
(a) Increases
(b) Decreases
(c) Remains constant
(d) Ist decreases then increases
Ans. – (d) Generally volume of liquids
increases due to heating but water shows
reciprocal behaviour between 0°C to 4°C.
At 4°C water has minimum volume and
maximum density.

Density =
Mass

Volume
Q9. Modern Industrial robot is:
(a) Programmable
(b) Multifunctional
(c) Servo controlled
(d) All of the above
(1) a (2) b
(3) Both a and b
(4) All of these

Ans. – (4) Robot is an electronic device. It can do different types of


work, so it is multifunctional. It is a programmable and servo
controlled system.

Q10. Consider the following statements: A 4-wheel vehicle moving


in a sharp circular path at high speed, what will happen: (1) Skid
outward
(2) Skid inward
(3) Overturn about its outer wheels (4) Overturn about its inner
wheels Which of the statements are correct? (a) 2 and 4 (b) 1 and 3
(c) 1 and 4 (d) 2 and 3
Ans. – (b) For completing circular motion, centripetal force must be
equal to centrifugal force. This centrifugal force is produced by friction
between type and road. If the centrifugal force is disturbed then
vehicle will skid in outward direction and turn about its outer wheel.

For safely turning MV2≤μsmg r

V ≤ √μgr Q11. (1) When altitude increases boiling point of water


decreases.
(2) Pressure varies with height.
Choose correct options:
(a) 1 & 2 are true
(b) 1 is true but 2 is false
(c) Both 1 & 2 are false
(d) None of these
Ans. – (a) Pressure varies with height. i.e. P =δ gh
P = pressure
δ = density
h = height
Boiling point of water at mountains is low due to pressure decrease
because less density of air is available there.

Q12. Two variable will be changed when light waves comes from
Air to glass.
(a) Wavelength, frequency and velocity. (b) Velocity and frequency.
(c) Wavelength and frequency.
(d) Wavelength and velocity.
Ans. – (d) When light waves goes from one medium to another
medium then speed of light and wavelength will change but frequency
remain constant. Frequency is fundamental quantity. It will be never
changed.

Q13. Consider the following:


In a nuclear reactor, self-sustained chain reaction is possible,
because:
(1) The fast neutrons are slowed by

graphite.
(2) Every neutron released in the fission
reaction initiates further fission. (3) More neutrons are released in
each of
the fission reactions.
(4) The neutrons take part in the fission
process.
Which of these statements are correct? (a) 1 and 3 (b) 1, 4 and 3
(c) 2 and 4 (d) None of these Ans. – (b) In self sustained chain
reaction in each fission reaction three more neutrons are produced.
The three neutrons then cause the fission of other thorium nuclei
producing nine neutron and so on. In nuclear reactor graphite is used
for moderator. All neutrons do not take part in reaction because
cadmium rod absorbs some neutrons.
Q14. Consider the following statements: (1) A digital library is a
collection of
documents in an organized electronic
form available on internet only. (2) Smart card is a plastic card with an
embedded microchip.
(3) Digital technology is primarily used
with new physical communication
medium such as satellite and fibre
optics transmission.
(a) 1 and 2
(b) 3 only
(c) 2 and 3
(d) All of these
Ans. – (c) The answer 2 and 3 is correct. A digital library is a
collection of documents in organized form. It can be available extra
than internet. Ex.: Mobile, computer etc. Q15. Match the following:
List-1
1. Pressure
2. Frequency 3. High speed 4. Electric field

List-2
A. Hertz
B. Mach
C. Newton/Coulomb D. Pascal

Codes: AB C D
(a) 2341
(b) 2134
(c) 1234
(d) 3241
Ans. – (a)
High speed – Mach
Frequency – Hertz
Electric field – Its strength is measured in Newton/Coulomb
Pressure – Pascal
Q16. Match List-I with List-II:
List-I
A. Potassium Nitrate
B. Potassium bromide
C. Potassium sulphate
D. Monopotassium tartrate
List-II
1. Fertilizer 2. Photography
3. Gun powder 4. Bakery
Code:
AB C D
(a) 2413
(b) 3214
(c) 2134
(d) 1234
Ans. – (b)
Potassium Nitrate
(KNO3) – Gun powder
Potassium Bromide
(KBr) – Photography
Potassium Sulphate
(K2SO4) – Fertilizer Monopotassium
Tartrate – Bakery Q17. Cryogenic engines used in:
(a) Sub marine propulsion
(b) Rocket technology
(c) VLSI
(d) Aeroplane engine
Ans. – (b) It has greater application in rocket technology. This type of
engine is made by USA and Russia. It is an important techniques in
space technology. Q18. Explosions in mines happen due to the
mixing of:
(a) H2 and O2
(b) Oxygen and Methane
(c) CO2 and Ethane
(d) Oxygen with Acetylene
Ans. – (d) Oxygen reacts with acetylene and burns with explosion.
Acetylene is used in vulcanization of rubber and acetic acid
formation. It is also used in welding to join iron layer.
Q19. Consider the following:
Which of the following causes acid rain: (1) Oxides of hydrogen
(2) Oxides of nitrogen
(3) Oxides of sulphur
(a) Both 1 and 2 (b) 1, 2 and 3 (c) 3 only (d) Both 2 and 3 Ans. – (c) It
is caused by emissions of compounds of carbon, nitrogen and
sulphur which reacts with the water molecules in the atmosphere to
produce acids like H2SO4, HNO3. Acid rain occurs in Norway,
Finland, Sweden.
Q20. Match the following are correctly matched:
Organisation Location (1) Centre of DNA
finger printing
and diagnostics Pune (2) National Brain Research Centre (3)
National Centre for Cell Science Hyderabad

Gurgaon Codes:
(a) 1, 2 and 3 (b) 3 only
(c) 1 and 3 (d) None of these Ans. – (d)
(1) DNA finger printing & diagnostics –

Hyderabad
(2) National Centre for Cell Science –
Pune
(3) Brain Research Centre – Gurgaon Q21. Three identical vessels
A, B and C are filled with water, mercury and kerosene
respectively up to an equal height. The three vessels are
provided with identical taps at the bottom of the vessels. If the
three taps are opened simultaneously, then which vessel is
emptied first: (a) Vessel B
(b) All the vessels A, B and C will be
emptied simultaneously
(c) Vessel A
(d) Vessel C
Ans. – (d)
Viscosity of materials
Kerosene < Water < Mercury
Kerosene vessel emptied first then water vessel and lastly mercury
vessel.
Q22. Which of the following statements is/are correct:
(1) Black powder was the first chemical
explosive.
(2) It is a mixture of sulphur, Charcoal
and Salt peter.
(a) Only 2 (b) Both 1 and 2 (c) Only 1 (d) Neither 1 nor 2 Ans. – (b)
Black powder is another name of gun powder which was the first
chemical explosive. It is a mixture of sulphur, charcoal and potassium
nitrate (salt peter). Here sulphur and charcoal act as fuel, while the
salt peter acts as an oxidizer. Q23. Match the following:
List-I
A. Oxide glass
B. Pyrex glass
C. Sodalime glass
D. Silica glass
List-II
1. High thermal expansion
2. Resistant against weathering 3. Extremely clear
4. Less vulnerable to cracking from
thermal shock
Code:
AB C D
(a) 3412
(b) 1324
(c) 3421
(d) 1234
Ans. – (a)
Silica glass has very low thermal expansion, so it is most resistant
against weathering.
Pyrex glass, it is also called sodium borosilicate glass, has low
coefficient of thermal expansion thus it is less vulnerable to cracking
from thermal shock.
Sodalime glass also called window glass has high thermal
expansion.
Oxide glass contains alumina and germanium oxide and extremely
clear.
Q24. (1) The hottest natural chemical flame
is cyanogen.
(2) Acetylene is sometimes used for
carburisation.
(3) Calcium carbide was used in
portable lighting.
(4) Acetylene was the first substance
from which organic semiconductors
are prepared.
Which of the above statements are correct:
(a) 1 and 3 (b) 1, 2 and 4 (c) 2, 3 and 4 (d) Only 1 Ans. – (c)
(1) Poly acetylene, the first organic
semiconductor was obtained from
acetylene.
(2) When object is large to fit into a
furnace acetylene is used for
carburisation (hardening) of steel. (3) Calcium carbide is used to
generate
portable lighting.
(4) Decyano acetyl is the hottest natural
chemical flame.

Light and Mechanics

Q1. Consider the following statements regarding a motor car


battery:
(1) Capacity is expressed in ampere-hour. (2) The voltage is usually
12V.
(3) Electrodes are lead and copper. (4) Electrodes used is
hydrochloric acid

(HCl).
Which of the above statements is/are correct?
(a) 1 and 2 (b) 2 and 4
(c) 3 and 1 (d) 2 and 3
Ans. – (a) In the motor car battery is of the voltage of 12V usually.
Capacity of car battery is measure/expressed in ampere-hour but
statements 3 and 4 is not correct because the electrolyte used in car
battery is sulfuric acid (H2SO4) and electrodes are lead and red
oxide.

Q2. Why wall of Dam is made thick in bottom because:


(a) Liquid pressure increases with depth. (b) Liquid pressure
decreases with depth. (c) Viscosity increases with increase in

depth.
(d) Density of liquid increases.
Ans. – (a)

PB = PA + δgh
where PA is atmospheric pressure i.e. PB∝ Pδ h

PB∝ h
Pressure varies with height.

Q3. In thermodynamics, a process is called reversible when:


(a) Surrounding and system change into each other.

(b) There is no boundary between system and surrounding.


(c) The surroundings are always in equi librium with the system.
(d) The system changes into the surroundings spontaneously.
Ans. – (c) In thermodynamics, a process
is called reversible when the surroundings
are always in equilibrium with the system.
Reversible process takes long time to complet. Chemical equilibrium
exists in
reversible reaction.
It is very slow process.
Q4. What is the Order of Magnitude of electric resistance of the
human body (dry)?
(a) 102 ohm (b) 106 ohm
(c) 1010 ohm (d) Zero ohm
Ans. – (b) Human body contain 70-75%
of water. It is a good conductor of electricity and heat. Current can be
flown through
body it means its resistance is not so large.
When human body (wet) this time it shows
lower resistance than dry body. Dry body
resistance is about 1 mega ohm.
Q5. A ‘black hole’ is a body in the space
which doesn’t allow any radiation to
come out. This property is due to its:
(a) Very low size
(b) Very higher density
(c) Very large size
(d) None of these
Ans. – (b) Black hole is reason in space in
which the gravitational force is so great
that no object can escaped from it. So, it
has very high density.
Q6. A spherical body moves with a uniform
angular velocity (W) around a circular
path of radius (r). Which one of the following is true:
(a) It has zero acceleration.
(b) It has radial acceleration W2r directed towards centre of path.
(c) Net force along the centre is zero.
(d) Tangential acceleration is equal to W2.
Ans. – (b) When body moves on circular
path with uniform velocity its tangential
acceleration is equal to zero but it has centripetal acceleration.
aTotal = √an2 + at2
aTotal = an

Q7. Escape velocity from moon surface is less than earth escape
velocity because: (a) Atmosphere is not present on moon. (b)
Radius of moon is less than earth

radius.
(c) Moon is more closer to sun.
(d) Mass of moon is less.
Ans. – (b)
Escape velocity – It is the maximum speed, with which body is
projected never come back to earth gravitation field.

Ve = √2gR
Where g = acceleration due to gravity R = Radius of the earth/moon
Here, Radius of moon is less than earth radius so Ve is less.

Q8. The memory of a computer is commonly expressed in terms


of kilobytes or megabytes. A byte is made up of: (a) Two binary
digits
(b) Eight binary digits
(c) Eight decimal digits
(d) None of these
Ans. – (b) The memory of computer is measured/expressed in
kilobytes or megabytes or gigabytes. One bytes in made of binary
digits.
1 byte = 8 binary digits or 8 binary digits = 1 bytes.

Q9. For reproducing sound, a CD (compact disc) audio player


uses:

(1) Quartz crystal and laser beam. (2) Titanium needle and quartz
crystal. (3) Laser beam.
(4) Barium titanate ceramic and laser

beam.
Which of the following statements is/are true:
(a) 1 and 4 (b) 2 and 4
(c) Only 3 (d) 1 and 3
Ans. – (c) In a compact disc audio player laser beam is used for
reproducing the sound.

Q10. When a compact disc is seen in sunlight, rainbow like


colour are seen. This can be explained/based on the basis of
phenomena of:
(a) Diffraction and transmission. (b) Reflection and transmission. (c)
All refraction, transmission and
diffraction.
(d) None of these.
Ans. – (c) When we look the CD we see rainbow like colour. All these
three phenomenon are the causes for the rainbow like colours.

Q11. Assertion (A): In a motion picture, usually 24 frames are


projected every second over the whole length of the film. Reason
(R): An image form on the retina of eyes persist for about 0.1 after
the removal stimulus.
Codes:
(a) Both A and R are true but R is not

correct explanation of A.
(b) A is true but R is false.
(c) Both A and R are true but R is the

correct explanation of A.
(d) A is false but R is true.
Ans. – (b) In a motion picture 24 frames are projected in a second to
create the illusion of continuity. The visual sensation of a particular
object for about 1/16s after changes. So A true but R false.

Q12. When a particle moved from A to B and the distance


between these is 20 m after he reached at B he return to A. What
is the displacement and distance travelled by that particle?
(1) Distance is 40 m and displacement

20 m.
(2) Distance is 40 m and displacement
0 m.
(3) Displacement and distance both zero. (4) Distance zero but
displacement 40 m. Ans. – (2) The shortest distance between two
point is called displacement and when any particle moved from one to
another point distance never be zero.
Q13. An oil tanker is partially filled with oil and moves forward
on the level of road with uniform acceleration. The free surface
of oil then:
(a) Assumed parabolic curve.
(b) Remain horizontal.
(c) Is inclined to the horizontal with large depth at the rear end.
(d) None of these.
Ans. – (c) An oil tanker is partially filled with oil and moves forward
on the level of road with uniform acceleration. Then free
space/surface of oil is inclined to the horizontal with larger depth at
the rear end. So option (c) is correct.
Q14. Consider the following statements: An ordinary light bulb
has rather short life because the:
(1) Bulb cannot be evacuated completely. (2) Filament wire is not
uniform. (3) Wire is supporting filament melt at high temperatures.
Which of the following statements are correct:
(a) 1, 2 and 3 (b) 1 and 2
(c) 2 and 3 (d) 1 and 3
Ans. – (c) The ordinary bulb the filament is not uniform and the glass
chamber is not evacuated completely. So, due to process of gases, at
high temperature the supporting filament melt at high temperature.
Q15. Assertion (A): Information is fast becoming a very important
field of activity in India.
Reason (R): Software is one of the major exports of the country and
India has very strong base in hardware.
Codes:
(a) A is true but R false
(b) R is true but A false
(c) Both A and R are true but R is the correct explanation of A
(d) Both A and R are true and R is the not correct
information/explanation of A Ans. – (a) It is correct that information
technology is fast becoming a very important field activity in India. It is
also true that software is the one of the major export in India but India
does not have strong base in hardware.
Q16. Consider the following statements: Glass can be etched or
scratched by: (1) Carbonic acid
(2) Diamond
(3) Aquaregia
(4) Hydrofluoric acid
Which of these statements are correct? (a) 1 and 4 (b) 2 and 4
(c) 3 and 4 (d) 1, 2, 3 and 4 Ans. – (b) The hardest elements
diamond is used to cut or scratched the glass. Writing on the glass
with the help of hydrofluoric acid.
Q17. Which of the following is/are present in the largest amount
in terms of percent by mass in the earth crust:
(a) Oxygen
(b) Oxygen and Calcium
(c) Silicon
(d) Carbon
Ans. – (c) Silicon is presents in the largest in terms of percentage by
mass on earth. The lithosphere is called siol and sima due to the
major presence of silicon.
Q18. Which of the following elements is/are paramagnetic in
nature:
(a) Iron and Nitrogen
(b) Oxygen and Hydrogen
(c) Nitrogen and Hydrogen
(d) Only Oxygen
Ans. – (d) Oxygen is paramagnetic in nature. In these substances
each atom has a permanent magnetic moments and it behaves like a
magnetic dipole or a tiny atomic magnet which is called the atomic
magnets.
Q19. A simple machine helps a person is doing:
(a) The same amount of work much faster.
(b) The same amount of work slowly. (c) Less work.
(d) The same amount of work with lesser force.
Ans. – (d) Simple machine help us in doing same amount of work
with lesser force. Simple machine has multiplies force. Force apply
for force at a convenient direction and points. But a machine does not
work itself when energy is supplied to it, it does useful work.
Q20. The velocity changing with respect to time is called:
(a) Displacement
(b) Speed
(c) Force
(d) Acceleration
Ans. – (d) The velocity changing with respect to time is known as
acceleration. It is calculated by a = V− u/t where V is final velocity and
u is initial velocity and t is time.
Q21. Which two of the following question is a vector quantity:
(a) Momentum (b) Energy
(c) Acceleration (d) Current (1) a and b (2) b and d
(3) a and c (4) a and d
Ans. – (3) Quantity which have magnitude as well as direction is
called vector quantity. For vector quantity it must obey the addition
rule of Parallelogram Quantity which have only magnitude is called
scaler quantity.
Current has magnitude as well as direction but it does not obey
addition rule of parallelogram so it is a scaler quantity. Q22. The
tendency of a liquid drop to contract and occupy minimum area
is due to:
(a) Density
(b) Viscosity
(c) Surface tension
(d) Vapour pressure
Ans. – (c) It is found that when a liquid is free from the external
forces (such a gravity) it is always takes the shape of spherical drop.
It is because for given volume, a sphere has the least surface area. It
means that the surface of every liquid has always a tendency to have
least surface area and it behaves like a stretched membrane having
tension in all direction parallel to surface. It also called surface
tension.
Q23. Domestic electrically wiring basically a: (a) Series
connection
(b) Series connection each room and parallel connection elsewhere
(c) Parallel connection
(d) None of these
Ans. – (c) For domestic use electrically wiring is basically a parallel
connection. In parallel circuit each branch received equal current. In
the parallel connection if one of the bulb removed from the circuit
electric current will flow continuously in other branches and other bulb
will continue to glow.
Q24. A fuse is used in main electric supply as a safety device.
Which one of the following statements about the fuse is correct:
(a) It must have a very high resistance. (b) It must have low melting
point. (c) It is made mainly from silver alloy. (d) It is connected in
parallel with the main switch.
Ans. – (b) A fuse is a short piece wire of a material of low melting
point and high resistances but not a very high resistance. If the
resistance becomes very much than even in normal current situation
the fuse wire gets heated up and melts. The fuse is connected in
series with the main switch. Q25. The organic molecules in sun
screen absorb the ultraviolet radiation and release is as heat.
There are three regions of ultraviolet light:

(1) UV-C is not absorb by the ozone and penetrates deeply into the
skin and can lead to cancer and premature skin aging.

(2) UV-A completely absorb’s by the earth atmosphere.


(3) UV-B is partially blocked by the ozone layer and involved in
tanning and burning of your skin as like sunburns.
Which of the statements given above is/are
true?
(a) 1 and 3 (b) 1 and 2
(c) Only 3 (d) 1, 2 and 3
Ans. – (c) The portion of the sun light that
is filtered or blocked is ultraviolet radiation. There are three reason of
ultraviolet
light–
* UV-C is completely absorb by the earth atmosphere.
* UV-B is partially blocked by the ozone layer and involved in tanning
and burning of your skin i.e. sunburn.

General Physics

Q1. Which one of the following is capillarity not the reason:


(a) Rising of water from the roots of a

plant to its foliage.


(b) Blotting of ink.
(c) Spread of water drop on a cotton

cloth.
(d) Rising of underground water. Ans. – (d) Capillary is hair like tube.
It is found that when a capillary tube open at both end is dipped in a
liquid, the liquid rises up in the tube. The example of capillarity are
blotting of ink, spread of water drop on cotton cloth and rising of water
from the root of plants to its foliage.

Q2. Which one of the following is the correct sequences of the


given substances in the decreasing order of their densities? (a)
Steel > Gold > Mercury
(b) Gold > Steel > Mercury
(c) Gold > Mercury > Steel
(d) None of these
Ans. – (c)
The density of Gold = 19.3 gm/cm3, Mercury = 13.6 gm/cm3 and Steel
= 7.6 gm/cm3. So the option (c) is correct. Density is calculated by
Mass/Volume

Q3. A weightless rubber balloon filled with 200cc of water. Its


weight in water is equal to:
(a) 9.8/5 N (b) 9.8/4 N (c) 9.8/10 N (d) None of these Ans. – (d) Since
balloon is filled up with water and put into water, it is in stable
position. The downward force exerted by its equal to the upward
buoyant force exerted by water. Hence its weight in water is zero as
the resultant force on its is zero.

Q4. Match the following lists:


List-I List-II

A. Electric charge 1. Watt B. Electric resistance 2. Volt C. Power 3.


Ohm D. Potential differences 4. Coulomb Codes:

AB C D (a) 1 2 3 4
(b) 3 4 2 1
(c) 4 3 1 2
(d) 3 2 4 1
Ans. – (c)
SI Unit Quantity
Coulombs Electric charge Volt Potential differences Watt Power
Ohm Resistance
Q5. Match the following lists:
List-I List-II A. Angle 1. m3
B. Volume 2. m2 C. Frequency 3. Radian D. Area 4. Hertz Codes:
AB C D
(a) 1 2 3 4
(b) 3 1 4 2
(c) 4 2 1 3
(d) 3 4 2 1
Ans. – (b)
Quantity SI Unit
Angle radian
Volume m3
Frequency Hertz
Area m2
Q6. Match the following lists:
List-I List-II
A. Diesel 1. Redolf Diesel B. Fountain pen 2. Levis Waterman C.
Steam engine D. Telephone 3. James Watt 4. Grahm Bell Codes: AB
CD
(a) 2 3 4 1
(b) 1 2 4 3
(c) 1 2 3 4
(d) 3 2 4 1

Ans. – (c) In the above option is already correctly match as like A –


1, B – 2, C – 3, D – 4.

Q7. Match the following lists:


List-I
A. Television B. Microphone

C. Gramophone D. Bicycle

List-II
1. Mackmilan
2. Thomas Alva Addition
3. J.L. Beard
4. Grahm Bell (c) 1324
(d) 2314
Ans. – (b)
Kinetic energy = 1/2 mv2
(m = Mass, v = Velocity)
Potential energy = Mgh
(h = height, g = acceleration due to gravity) Distance = Time × Speed
Current = Q/T = Charge/Time Q9. Match the following lists:
List-I List-II
A. Manometer 1. Depth of sea B. Fathometer 2. Gas pressure C.
Cryometer 3. Density of gas D. Dasymeter 4. Low temperature
Codes:
Codes: AB C D
(a) 3 1 4 2
(b) 3 4 2 1
(c) 1 2 3 4
(d) 3 2 1 4
Ans. – (b)
Television – J.L. Beard Microphone – Grahm Bell Gramophone –
Thomas Alva Addition
Bicycle – Mackmilan Q8. Match the following lists:
List-I List-II
A. Kinetic energy 1. Q/T
B. Potential energy 2. 1/2 mv2 C. Distance 3. Time × Speed D.
Current 4. Mgh Codes:
AB C D (a) 1 2 3 4 (b) 2 4 3 1 AB C D
(a) 2143
(b) 2431
(c) 3142
(d) 3124
Ans. – (a)
Manometer – Gas pressure
Fathometer – Depth of sea
Cryometer – Low temperature Dasymeter – Density of gas
Q10. Graphene, a newly discovered allotrope of carbon can have
application in: (1) Solar cells
(2) Sensors to diagnose diseases (3) Lithium ion batteries
(4) High frequency transistors
(5) Display screens
Which of the following statements is/are correct:
(a) 1, 2 and 3 (b) 2, 3 and 4 (c) Only 5 (d) All of these Ans. – (d)

Due to large surface area, graphene is used to prepare sensors and


display screens. High frequency transistors can also be prepared with
the help of graphene because of the higher speed at which electrons
in graphene move as compare to electrons in silicon.
Carbon solar cell uses graphene as electrodes in lithium-ion batteries
graphene is used as anode.

Q11. A satellite S is moving in an elliptical orbit around the earth.


The mass of the satellite is very small compared to the mass of
the earth:
(a) The acceleration of S is directed

towards the centre of the earth. (b) The angular momentum of S


about the
earth changes in direction, but its
magnitude remains constant. (c) Mechanical energy of S varies
periodically with time.
(d) Linear momentum of S remains
constant in magnitude.
Ans. – (a) Force on satellite is always towards earth, therefore
acceleration of satellite S is always directed towards the centre of the
earth. Net torque of this gravitational force about centre of the earth is
zero. Therefore, angular momentum of S about centre of earth is
constant throughout.
Q12. Consider the following statements: (1) A geostationary
satellite is at an
height of 24,000 km.
(2) FM transmission of music is a very
good quality because the atmospheric
or man made noises which are
generally frequencies variation can do
little harm.
(3) Geostationary satellite moves around
the earth.
Which of the statements is/are correct? (a) 1 and 3 (b) 2 and 3
(c) 1, 2 and 3 (d) 1 and 2
Ans. – (b) A geostationary satellite is at an height of 36,000 km. It
revolves around the earth from west to east in an orbit coplanar with
equatorial plane.
FM transmissile is quite immune to noise as AM transmission. Noise
is an unwanted signal.
Q13. A converging lens is used to form an image on a screen
when the upper half of the lens is covered by an opaque screen.
(a) Half of the image will disappear. (b) Complete image will be
formed. (c) Intensity of the image will increase. (d) Intensity of the
image will decrease. Choose correct option:
(A) a and b (B) b and d (C) All of the above (D) b and c Ans. – (B)
When upper half of the lens is covered, image is formed by the rays
coming from lower half of the lens or image will be formed by less
number of rays. Therefore, intensity of image will decrease. But
complete image will be formed.
Q14. Match the following list:
(A) High speed (1) Pascal (B) Pressure (2) Joule (C) Energy (3) Mach
(D) Wavelength (4) Angstrom Codes:
AB C D (a) 3 1 2 4 (b) 1 2 3 4 (c) 3 2 4 1 (d) 2 4 1 3 Ans. – (a)
Quantity Unit High speed – Mach Pressure – Pascal Energy – Joule
Wavelength – Angstrom

Q15. One astronomical unit is the average distance between:


(a) Earth and the Sun
(b) Jupiter and the Sun
(c) Pluto and the Sun
(d) None of these
Ans. – (a) One astronomical unit in the mean distance of earth from
the Sun. Its value is given
1 AU = 1.5 × 1011 m

Q16. Which one of the following hormones regulate blood


calcium and Phosphate: (a) Growth hormone
(b) Thyroxine
(c) Glucagon
(d) Parathyroid hormone
Ans. – (d) Parathyroid hormone controls distribution of calcium and
phosphates. Parathyroid hormone is important in bone development.
Thyroxine – It faster the rate of cells respiration and essential for
normal growth of bones and hairs.

Q17. Consider the following statements: A simple pendulum is


set into oscillation then:
(1) The amplitude of oscillation of the

simple pendulum decreases with time. (2) In each cycle the bob
attain a given
velocity twice.
(3) Both acceleration and velocity of the
bob zero when reach extreme
position during the oscillation.

(4) The acceleration is zero when the bob passes through the mean
position.

Which are correct?


(a) 1 and 2 (b) 2 and 3
(c) 2, 3 and 1 (d) None of these Ans. – (c) At the extreme position the
acceleration will be maximum.

Q18. A car is running on a read at a uniform speed of 60 km/h.


The net resultant force on the car is:
(a) Driving force in the direction of car’s

motion.
(b) Equal to zero.
(c) An incline force.
(d) None of these.
The net resultant force on the car running on the road with a uniform
velocity/speed is zero. Because at uniform speed acceleration of car
is zero so force is also zero. Force = Mass × Acceleration

Q19. Which of the following distance time graph (n – t) represent


one dimensional uniform motion:
XX
(a) (b)
ttXX
(c) (d)
tt

Ans. – (c) Figure (c) is the correct option of the given option. Figure
(c) the time distance graph for one dimensional uniform motion.

Q20. When the light waves pass from air to glass, the variable
affected are: (a) Wavelength and velocity
(b) Velocity and frequency
(c) Wavelength and frequency
(d) Wavelength, velocity and frequency Ans. – (a) The wavelength
and frequency both are affected. Velocity decreases when it passes
from rarer to denser medium. While the more a rare band in passing
through different medium the shorter will be wavelength. There is no
effect on frequency on changing medium.

Q21. The variation of displacement d with time t in the case of


particle freely under gravity from rest is correctly shown in
graph:

(a) (b)
(c) (d)
Ans. – (d) Graph (d) represented the particle falling freely under
gravity from rest. Q22. Which one of the following statements is
not correct:

(a) When the displacement of a body is zero then distance also zero.
(b) When acceleration of a particle is zero then resultant force on the
particle is zero.
(c) The velocity calculated by
Distance/Time.
(d) When velocity of a particle is zero then momentum of it is also
zero.

Ans. – (a) The shortest distance between two points is called


displacement and the length of covered path is called distance.
Q23. Which one of the following statements is not correct:
(a) The velocity of sound in air is

independent of pressure.
(b) The velocity of sound in air decreases
as the humidity decreases.
(c) The velocity of sound in air increases
with the increases temperature. (d) The velocity of sound in air is not
affected by the change in amplitude
and frequency.
Ans. – (b) All the above option are correct except (b) because the
sound travel faster in moist air than in the dry air.
Q24. In electrical network, which one of the following statements
is true:
(a) Ammeter and voltmeter is connected
parallel and series respectively. (b) Ammeter is connected in series
and
voltmeter is connected in parallel. (c) Both are connected in series.
(d) Both are connected in parallel. Ans. – (b) Ammeter is an electrical
device which measures current between two point. It’s internal
resistance is very low and it is connected in series.
Voltmeter – It measures potential difference between two point. It’s
resistance is very large and connected in parallel. Q25. Which of the
following is semiconductor:
(1) Silicon (2) Quartz
(3) Ceramic (4) Cadmium (5) Germanium
Choose correct options:
(a) 1 and 2 (b) Both 3 and 5 (c) 1 and 5 (d) Only 1
Ans. – (c)
Semiconductor – It is a solid device whose conductivity lies between
good insulator and good conductor. At room temperature it behaves
like insulator. It has negative temperature gradient. It is used in chip
fabrication.
Semiconductor is of two types:
(i) N-type (ii) P-type.

Physics : Technology
Q1. Assertion (A): In a motion picture usually 24 frames are
projected every second ever the whole length of film.
Reason (R): An image formed on the retina of eyes persists for about
0.2 second after the removal of stimulus.
Codes:
(a) Both A and R are true and R is the

correct explanation of A.
(b) Both A and R are true but R is not
correct explanation of A.
(c) A is true but R false.
(d) R is true but A false.
Ans. – (c) A is true but R is false. In a motion picture 24 frames are
projected in a second which create the illusion of continuity. The
visual sensation of a particular object as scene persist for about 1/16
and after change.
Q2. WIMAX is related to which one of the following:
(a) Missile technology
(b) Communication technology
(c) Space technology
(d) None of these
Ans. – (b) WIMAX is a emerging technology for broadband wireless
access. It promises very much high data rates, high reliability, good
efficiency and low cost. WIMAX enjoys strong industry support and
standardization. Because of its low cost, it is used to provide
broadband internet access to suburban rural areas and thus bridge
the digital divide.
Q3. In order of their distances from which of the following
planets lie between Mars and Uranus:
(a) Saturn and Neptune
(b) Saturn and Earth
(c) Jupiter and Saturn
(d) Earth and Jupiter
Ans. – (c) As per distances from such Jupiter and Saturn lie in
between Mars and Uranus. The solar system comprises the sun,
eight planets (Mercury, Venus, Earth, Mars, Jupiter, Saturn, Uranus,
Neptune revolving around the sun. The direction of the planets in
their revolution around the sun is from West to East. The earth is a
third nearest planet the sun. The planets nearest to the earth is
Venus.
Sun → Mercury → Venus → Earth → Mars → Jupiter → Saturn →
Uranus → Neptune
Q4. A person sitting in a car which is at rest. The reaction from
the road at each of four wheels of the car is R. When the car run
on the straight level road, how will the reaction at either of the
front wheel vary?
(a) It shall depend on the material of
the road.
(b) It will be greater than R.
(c) It will be less than R.
(d) It will be equal to R.
Ans. – (d) The dynamic friction in any object is proportional to instant
reaction and it is less than static friction. When the car runs on a
straight level of the road the reaction either of front wheel will be
equal to R.
Q5. Optical fibre works on the principle of: (a) Interference
(b) Scattering
(c) Refraction
(d) Total internal reflection
Ans. – (d) Optical fibers works on the principle of total internal
reflection. When the light is incident at small angle at one end, it gets
refracted into the fiber and gets incident on the interface of the fibers
and coating. The angle of critical is less than angle of incidence. So,
the light suffers the internal reflection again and again till the angle of
incidence being greater than the critical angle for the fiber.
Q6. When a mirror is rotated by an angle θ the reflected ray will
be rotate by: (a) 0° (b) θ
(c) 2θ (d) None of these Ans. – (c) For a given incident ray, if plane
mirror is rotated through a certain angle, then the reflected ray rotates
through twice of that angle. So if a mirror is rotated by an angle θ, the
reflected ray will rotate by 2θ.
Q7. The internal reflection can take place when light travel from:
(a) Air to water
(b) Diamond to glass
(c) Air to glass
(d) Water to glass
Ans. – (b) Total internal reflection takes place when light travel from
optically denser medium to optically rarer medium. (Refractive index
for glass = 1.57, Refractive index of Diamond = 2.45) Q8. A ball is
dropped from the top of a high building with a constant
acceleration of 9.8 m/s2. What will be its velocity after 3
seconds?
(a) 9.8 m/s (b) 49.6 m/s (c) 23.9 m/s (d) 29.4 m/s Ans. – (d) We know
that
V = u + gt
(Where V is velocity, g is acceleration due to gravity and t is time)
V = 0 + 9.8 × 3
V = 29.4 m/s
Q9. Match the following list:
(A) Force 1. m/s2
(B) Acceleration 2. Joule
(C) Velocity 3. Newton
(D) Work 4. m/s

Codes: AB C D
(a) 1 2 3 4
(b) 3 1 4 2
(c) 2 3 1 4
(d) 4 3 2 1
Ans. – (b)
Force – Newton
Acceleration – m/s2
Work – Joule
Velocity – m/s
Q10. A noise level of 100 decibel would correspond to:
(a) Noise from machine shop
(b) Ordinary conversation
(c) Sound from a noisy street
(d) None of these
Ans. – (a) Loudness of sound is measured
on the scale called decibel scale. The softest audible sound on the
decibel scale is
zero db. The noise level of 100 decibel
would correspond to noise from a machine
shop.
Q11. The cloud fleet in the atmosphere
because of their low:
(a) temperature and pressure
(b) pressure and density
(c) only temperature
(d) None of these
Ans. – (c) When air rises, it is cooled by
expansion. After low point has reached
cooling lead to condensation of water
vapour in the atmosphere. Tiny droplets of
water vapour which are too small to fall as
rain shower, will be suspended in the air
and fleet as clouds.
Q12. Consider the following natural phenomena:
(1) Diffraction of light
(2) Scattering of light
(3) Refraction of light
(4) Terrestrial of light
Due to which of these phenomena is mirage formed?
(a) 1 and 3 (b) 3 and 4
(c) 1 and 4 (d) None of these Ans. – (b) Due to refraction of light and
terrestrial of light the mirage forms in deserts. Due to total internal
reflection of light which is causes of continuous refraction of rays in
different layer of air, the observer think of water where it does not
exist.
Q13. A hunter aims his gun at point between the eyebrow of a
monkey sitting on a branch of a long tree. Just as hunter fires,
the monkey jumped down. The bullet will:
(a) Miss the monkey altogether. (b) Hit the monkey at the point
aimed. (c) Hit the monkey above the point aimed.
(d) None of these.
Ans. – (b) The bullet will hit the monkey at the point aimed because
the bullet will cover the same vertical distance as of monkey covered
the distance.
Q14. In an atom, the order of filling up the orbitals is governed
by:
(a) Aufbau’s principle
(b) Heisenberg uncertainty principle (c) Hund’s rule
(d) Pauli exclusion principle
Ans. – (a) The order of filling up of the orbital is given by Aufbau’s
principle which conveys that electron are added to an atom one at a
time, starting with lower energy orbital until all the electron have been
placed in an appropriates orbital. According to Pauli’s exclusion
principle, no two electron in an atom can have same value for all the
four quantum number, hence there can be maximum of two electrons
in an orbitals.

Q15. Which one of the following can be used to conform whether


drinking water contains gamma emitting isotopes or not: (a)
Lead plate
(b) Microscope
(c) Spectrophotometer
(d) Scintillation counter
Ans. – (d) Scintillation counter is a device used to detect radioactive
isotopes. Generally Alpha, Gamma and Beta rays emitted from
radioactive substance emit radiation on scintillation counter.

Q16. The Alpha particle carries two positive charge. Its mass is
very nearly equal to that of:
(a) Sum of masses of two positron and

two neutron.
(b) Two positrons as each positrons
carries a single positive charge. (c) Two protons.
(d) An atom of helium.
Ans. – (d) Each Alpha particles contains two protons and two
neutrons, so its mass is equal to an atom of helium. The molecular
weight of the helium atom is same as alpha particle.
Q17. In addition to protein and carbohydrates other nutritional
substances found in milk may be:
(1) Potassium (K) (2) Cobalt (CO) (3) Iron (fe) (4) Calcium (Ca) (a) 1,
2 and 3 (b) 1, 2 and 4 (c) 1, 3 and 4 (d) 2, 3 and 4 Ans. – (c) Milk
contains protein, carbohy
drates, sugar, fats, sodium, potassium, calcium, iron, vitamins A, B12,
C and D, magnesium, phosphorus, thiamin, niacin and riboflavin.

Q18. Which one of the following elements is alloyed with iron to


produce steel which can resist high temperature and also have
high hardness and abrasion resistance:

(a) Aluminium (c) Chromium (b) Tungstun (d) Nickel

Ans. – (c) Chromium generally mixed with iron to produced steel,


which can resist high temperature and also have high hardness and
abrasion resistance. Hardness of steel depends on percentage of
chromium.

Q19. Who among the following is considered as the inventor of


the World Wide Web (www):
(a) Vinod Dham
(b) Edward Kasner
(c) Tim Berners-Lee
(d) None of these
Ans. – (c) The World Wide Web is a system of interlinked hypertext
documents access via the internet, with the web browser, a user
views web pages that may contain text image and other multimedia
and navigates between them using hyperlinks. It was created around
1990 by the English man Tim-Berners-Lee and the Belagian Roberts
Cailling working at CERN in Geneva, Switzerland.

Q20. India has recently landed its Moon Impact Probe on the
moon. Among the following countries, which on landed such
probe on the Moon earlier: (a) USA (b) Russia
(c) Japan (d) Denmark
(1) Only a (2) All a, b, c, d (3) a, b and c (4) None of these Ans. – (3)
In 2008, 14th November India has landed its Moon Impact Probe on
the moon. USA, USSR (Russia) and Japan are three countries to
have landed previously Moon Impact Probe on the moon. So, the
correct answer is (3).

Q21. Consider the following statements:


(1) A flute of smaller length produced waves of lower frequency.
(2) Sound travel in rocks in form of longitudinal elastic wave only.
Which of these statements given above is/are correct?
(a) 1 and 2 both
(b) Only 1
(c) Neither 1 nor 2
(d) Only 2
Ans. – (d) The flute of smaller length produced wave of higher
frequency. So, statement one (1) is incorrect. Sound travel in rocks in
the form of longitudinal waves only so statement (2) is correct.
Q22. In dry cell (battery), which of the following are used as
Electrolytes:
(a) Ammonium Chloride and Calcium Chloride.
(b) Sodium Chloride and Zinc Chloride.
(c) Ammonium Chloride and Zinc Chloride.
(d) None of these.
Ans. – (c) The Leclache cell or dry cell is
also known as Zinc carbon cell. Dry cell
has zinc negative electrode and a manganese dioxide positive
electrode and the
electrolyte is a solution of ammonium
chloride. The carbon rod is in contact with
the positive electrode (but takes no part in
chemical reaction) and is called current
collector.
Q23. Consider the following statements:
(1) First LNG terminal in India was built in Hassan.
(2) Natural gas liquid are separated from LPG and these include
ethane, propane, butane and natural gasoline.
(3) Liquefied Natural Gas (LNG) is liquefied under extremely cold
temperature and high pressure to facilitate storage or transportation
in specially designed vessels.
Which of the following statements given
above is/are correct:
(a) 2 and 3 (b) 1 and 2
(c) 1 only (d) 1, 2, 3 all Ans. – (a) LNG as an alternative to oil and
naphtha is about to land on Indian shores. LNG is just natural gas
cooled to an extremely low temperature (160°C). Dahej is the first
regasification project in India. Q24. Assertion (A): In the desert
during the period of water loss the camel utilizes water stored in the
hump of its back. Reason (R): Fatty acids in the hump of camel
following metabolism yield water called metabolic water.
Codes:
(a) A is true, R is false
(b) A is false, R is true

(c) Both A and R are true and R is the correct explanation of A

(d) Both A and R are true but R is the not correct explanation of A
Ans. – (b) In a camel excess water is
stored in small vessel all around the stomach.
The hump of the camel consists of fatty tissues which store energy. At
the time of scarcity of food these fat tissues are metabolised yielding
energy and water. Q25. Kinetic energy of a truck, a car and a
motorcycle are equal. If same resistive force is applied on it and
they will be stopped at a distance of X, Y and Z respectively:
(a) X > Y > Z (b) X < Y < Z (c) X = Y = Z (d) X = 4Y = 8Z Ans. – (c)
KE = 1/2 mv
Where M = Mass of the body
V = Velocity
Here change in kinetic energy is equal to work done by resistive
force.
i.e. KE = FS
A/q kinetic energy of bodies are equal and resistive force are also
equal so distanced travelled by body is also equal.

Sound and Other Mechanics

Q1. Which one of the following is correct about consequent


pole:
(a) It is a pole of magnet which gets

weaker due to demagnetisation. (b) It is a pole developed due to


faulty
magnetisation.
(c) It is the pole created by magnetisation
through induction.
(d) It is the pole of paramagnetic sub
stances.
Ans. – (b) Due to faulty magnetisation, a magnet is sometimes found
to have developed more than two poles. For example, in the method
of divided touch, if both the

strikers poles are north poles, then the steel bar will developed south
poles at the two end with the north pole at the middle. The extra pole
that is developed at the middle is known as consequent pole.

Q2. A hollow metal sphere of radius 5 cm is charged such that


the potential on its surface is 10V. The potential at the centre of
the sphere would be:
(a) Same as at a point 5 cm away from the

surface.
(b) Zero.
(c) Same as the point 25 cm away from

the surface.
(d) None of these.

Ans. – (d) In the case of hollow sphere, the potential at any point
inside the sphere is equal to the potential on its surface. As the
potential on the surface is 10V, therefore potential at the centre of the
sphere would be also 10V.

Q3. Consider the following statements and select correct code:


(1) The amount of current flowing

through the circuit decreases by increasing the turns of wire


rheostats.

(2) The Reheostats are used for increasing or decreasing the


strength of a current in a circuit.
Codes:
(a) Only 2 is correct
(b) Neither one nor two correct
(c) Both 1 and 2 are correct
(d) Only 1 is correct.
Ans. – (c) Since the electric resistance of a material is dependent on
its length, more the turns of wire on the rheostats, greater is its
resistance and smaller the amount of current that flow through the
circuit. So code (c) is correct.

Q4. Consider the following statements and select correct codes:


(1) When external magnetic field is

applied to diamagnetic substances, the field inside the substance is


slightly weakened and the substances possessed a minute magnetic
effect.

(2) Paramagnetic substances contain equal number of electron


spinning is opposite direction.

Codes:
(a) 1 and 2 are correct
(b) Only 2 is correct
(c) Only 1 is correct
(d) Neither one (1) nor 2 are correct Ans. – (b) Diamagnetic
substances contain equal number of electron spinning is opposite
direction. These electron neutralize the magnetic force of each other,
but when an external magnetic field is applied their orbit slightly
disturbed.

Q5. Which one of the following acts as a defect of simple cell:


(a) Polarisation (b) Oxidisation (c) Local action (d) a and c
Ans. – (d) Polarisation is the deposition of hydrogen bubbles on the
copper plate in simple cell which continuously weaken the main
current and after sometime the current stops flowing. Local action
occur because of the use of commercial zinc as metallic impurities
contained only impure zinc acts a positive plates and small local cells
are formed on the zinc plate.
Q6. The amplification factor of a triode value depend upon which
one of the following:
(a) relative position of cathode, grid and

plate.
(b) plate voltage.
(c) temperature of the cathode.
(d) temperature of the plate.
Ans. – (b) Here M = RP × gm Where RP is the plate resistance and
gm is the mutual conductance. Hence the amplification factor of a
triode value depend upon the plate resistance.

Q7. The orbital velocity of an earth’s artificial satellites if it


revolving near the earth’s surface would be which of the
following:
(a) 5.98 km/s (b) 11.20 km/s (c) 7.92 km/s (d) 20.50km/s Ans. – (c)
Here
V=√gr
g = 9.8m/s2 and R = 6.4 × 106 m V = 7.92 × 103 m/s
V = 7.92 km/s

Q8. The energy generation in star is due to which one of the


following:
(a) Chemical reaction
(b) Fusion of heavy nuclei
(c) Fusion of light nuclei
(d) None of these
Ans. – (c) The energy of generation in the stars is due to fusion of
nuclei into a heavy nucleus. The mass of a single nucleus is less than
the initial mass. The differences in mass is converted into energy of
stars.

Q9. The radio station broadcast at 30 meter band. The frequency


of electromagnetic waves transmitted from the station could be:
(a) 19 × 1011 HZ (b) 18 × 109 HZ (c) 10 MHZ (d) 4 × 1010 MHZ Ans. –
(c) We know that
V = C/X
V=
3 × 108
30

V= 107
V = 10 MHZ

Q10. Which one of the following would fill the empty space in the
given below nuclear reaction:
(a) Alpha particle (b) Proton
(c) Neutron (d) None of these Ans. – (c) Here,
5B10 + 2He4 + 7N13 + 0X1
as Z = 0 and A = 1 ∴0X1 = 0H1
So it is a neutron.

Q11. The half life of radium is 1600 years the fraction of the
sample of the radium that would remain after 6400 year would
be: (a) 1/10 (b) 1/12
(c) 1/16 (d) Zero
Ans. – (c) After each 1600 year (64000 year), the substance
remaining behind will be
1 – [1/2 + 1/4 + 1/16]
= 1/16
So option (c) is correct.

Q12. Consider the following statements and select the correct


code:
(1) In a step-up of transformer, the

number of turns in the secondary coil is more than in the primary coil.
(2) In the step-down of transformer the

number of turns in the primary coil is more than the secondary coil.
Codes:
(a) Only 2 is correct
(b) Only 1 is correct
(c) Neither 1 nor 2 are correct
(d) Both 1 and 2 are correct
Ans. – (d) In a step-down transformer the voltage of secondary coil is
also lower than primary coil. In step-up transformer, the voltage of
primary coil is lower than in the secondary coil.
Q13. Which one of the following is/are in correct about infrared
spectrum:
(1) They have large wavelength.
(2) The penetration power of these rays are very high.
(3) They show heating effect.
(4) They are absorbed by rock salt.
Codes:
(a) 1 and 4 (b) Only 4
(c) 1, 2, 3 and 4 (d) 1 and 3
Ans. – (b) The infrared spectrum are
absorbed by glass not by rock salt. The
penetration of these are very high on
account of which these ray can cross the
clouds mist and fag.
Q14. Which of the following are magnetic
substances:
(a) Iron
(b) Nickel
(c) Cobalt
(d) All of the above
Ans. – (d) The magnetic substances can be
defined those substances which can be
attract by ordinary magnets and whose
magnetic properties can be easily detected.
Iron, Nickel, Steel and Cobalt are magnetic substances so option (d)
is correct.

Q15. The resistance of a uniform wire of length 80 cm and of


diameter 0.4 mm is found to be 1.34 ohms. The specific
resistances of the material would be: (a) 11 × 10–3 ohm
(b) 16 × 10–6 ohm cm

(c) 15 × 20–8 ohm cm


(d) 21 × 20–6 ohm cm
Ans. – (d)
R=S×
l
a
(here R = 1.34, l = 80 cm, a = π (0.02)2 cm2)

1.34 = S ×
80
π × 0.0004
S = 21 × 20–6 ohm cm.

Q16. The combined resistance of 8 electric bulb of resistance


200 ohm each connected in parallel would be which one among
the following:

(a) 0 ohm (b) 200 ohm


(c) 25 ohm (d) None of these Ans. – (25 ohm), (c)
According to question,
8 bulbs connected in parallel

So 1 = 8 ×1
R 200
=1
25
R = 25.
Q17. Consider the following statements and select the correct
codes:

(1) The resistance of alloys generally increases with increases in the


temperature.

(2) The resistance of pure metallic wire increases when heated and
decreases when cooled.

(3) There is no effect of temperature on a carbon and insulator.


Codes:
(a) 1 and 3 (b) Only 2 is correct
(c) 1 and 2 (d) 1, 2 and 3
Ans. – (c) Carbon, insulator etc. have their
resistance decreases with the rise of temperature, i.e., they have
negative coefficient of temperature, so option (c) is correct.

Q18. Which one of the following correct about shunt:


(a) It is a combined resistance of a circuit. (b) It is a resistance
connected in series

with the galvanometer.


(c) It is a fuse wire of a galvanometer. (d) It is a resistance connected
in parallel

with the galvanometer.


Ans. – (d) Shunt is the resistance connected in parallel with the
galvanometer conveniently made for safety of galvanometer coils. So
option (d) is correct.

Q19. Which one of the following is used as a fuse in a circuit of


larger current: (a) Tinned copper wire
(b) Lead alloy
(c) Silver wire
(d) Tungsten wire
Ans. – (a) The tinned copper wire is used as a fuse in a circuit of
larger current and lead alloy used in a circuit for small current. We
also know that the silver is very good conductor of current. Tungsten
wire is used in electric bulb filament because it have more and more
resistance.

Q20. Which one of the following is correct sequences of the


increasing frequencies of electromagnetic waves:
(a) Radio waves, gamma rays, x-rays,

infrared, ultraviolet.
(b) Radio waves, infrared, ultraviolet,
x-rays, gamma rays.
(c) Radio waves, ultraviolet, x-rays,
infrared, gamma rays.
(d) Radio waves, ultraviolet, infrared,
gamma rays, x-rays.
Ans. – (b)
Radio waves – 104 to 1012 HZ
Infrared – 10 to 4 × 1014 HZ Ultraviolet – 7.6 × 1014 to 1017 HZ X-rays –
10 to 1020 HZ
Gamma rays – 109 × 1024

Q21. The arranging in which the magnetic needle indicated the


presence of an electric is called which one of the following
option:
(a) Ammeter
(b) Galvanometer
(c) Galvanoscope
(d) Alternator
Ans. – (c) If the reading taken from the galvanoscope can compare
the strength of the current, Galvanometer. The Ammeter measure the
current of circuit connected in series. The voltameter measure the
voltage of the circuit path.

Q22. Which one of the following statements is incorrect:


(a) The two poles of a magnet have the

same strength.
(b) There is no effect of heat in magnet. (c) When a bar magnetised, it
slightly

increased in length.
(d) None of these.
Ans. – (b) When a magnet strongly heated, the speed of the
molecules is accelerated resulting in the decreases of their rigidly. It
causes them to go back to the closechain arrangement. Thus the
magnet is demagnetised on heating. When a bar is magnetised, it
slightly increases in length due to the setting of molecular magnet
along straight chain.

Q23. Which one of the following statements is in correct about


LAZER:
(a) It is a technique to produce a beam of
light wave of the same wavelength, travelling in the same direction
and phase.

(b) The light produced by sun and electric bulbs is coherent whereas
the light produced by the lazer is incoherent.

(c) Laser beam are used to remove unhealthy tissue from the body.
(d) Laser is also used to weld a detached retene back into space.
Ans. – (b) In the case of light produced by the sun and electric bulb
excited atom release light irregularly. As a result, the light has
different frequencies and travel in different path/direction. This is
incoherent light but the light produced in a laser are coherent.
Q24. The poles of Daniells cell of E.M.F. = 1.08 volt and internal
resistance 0.2 ohm are joined by a wire having resistance of 5.2
ohm. The strength of the current in this case would be:
(a) 4 amp (b) 11 amp
(c) 0.2 amp (d) None of these Ans. – (0.2 Amp), (c)
Here E = 1.08 Resistance of wire = 5.2
Internal Resistance (r) = 0.2
We know that,
E
= 1.08 = 0.2 amp
I=
R + r 5.4

Q25. Which one of the following acts electrolytes in dry cell:


(a) A moist paste of ammonium chloride

and sulfuric acid.


(b) Sulfuric acid in the form of paste. (c) Moist ammonium chloride
only. (d) None of these.
Ans. – (d) Zinc generally forms the outer causing of the dry cell and
act both as a

container and the negative pole. The positive pole is a rod of carbon
surrounding by a mixture of magnese dioxide and graphite and all
enclosed in a muslin bag. The electrolytes is a moisture paste of
ammonium chloride solution and plaster of Paris.
Functional Properties

Q1. Consider the following statements regarding Electron:


(1) Atomic mass is equal to number of

electron plus number of proton.


(2) Mass of an electron is 1 of mass of hydrogen atom. 1831

(3) e/m ratio of electron is 1.76 × 108 coulomb/gram.


(4) In neutral atom number of electrons is equal to number of proton.
(a) 2 and 3 (b) 2, 3 and 4
(c) 1 and 2 (d) None of these
Ans. – (b) Atomic mass (Z) = No. of protons + No. of electrons.
Electron is a negatively charge particle
whose mass is 9.1 × 10–31 kg and charge =
– 1.6 × 10–19 C.
Mass of sub atomic particle is as follows:
Neutron > proton > electron
e/m ratio of electron is 1.76 × 108
coulomb/gram.
Q2. Consider the following statements related to blue-ray disc:
(1) It has the same thickness as DVDs and CDs.
(2) The first prototype blue-ray disc player was released in USA.
(3) The name blue-ray disc refers to the blue laser used to read the
disc, which allows information to be stored at a greater density than is
possible with longer wavelength red laser used for DVDs.
Codes:
(a) 1, 2 and 3 (b) 1 and 2
(c) 2 and 3 (d) 1 and 3
Ans. – (d) The first prototype blue ray disc
player was released in April, 2003 in Japan.
Dimension of DVD’s and CDs are almost
same.
DVD is called Digital Video Device. It stores data and information. It is
a secondary memory.
Q3. Consider the following statements and choose correct
option:
(1) Sonar is a system that uses sound echo to detect and locate
objects.
(2) High frequency sonar has limited range whereas low frequency
sonar has a greater range.
(3) The sonar of dolphins is considerably more sophisticated than any
man made sonar in the world.
Codes:
(a) All of the above
(b) Only 2 and 3
(c) None of these
(d) 1 and 3
Ans. – (a) SONAR is based on the principle of sound wave reflection.
High frequency SONAR is used for locating small
targets or for observing large targets in
great details but has a limited range. Low
frequency SONAR has a greater range.
Some of the properties of dolphin sonar
are:
(1) It is multiband and multimode in nature.
(2) Frequency-hopping capability.
(3) Penetrating single-pulse system. Q4. Consider the following
statements related to optical fiber and select correct
option:
(1) Light is propagated down on optical fiber by refraction and internal
reflection.
(2) Electrical signals are converted into pulses of light by
semiconductor devices known as LED.
(3) It has large bandwidth, less noisy and cost effectiveness
compared to copper cables.
Codes:
(a) 1, 2 and 3 (b) Only 1
(c) 2 and 3 (d) None of the above
Ans. – (a) Optical fiber is a thin cylindrical cable made of glass fiber
used for transmitting light signals. Light is propagated in
optical fiber by refraction and total internal
reflection. It has large bandwidth, less
noisy and very economic. The bandwidth
of optical fiber transmission system is upto
1 million GHZ.
Q5. Match the following List-1 and List-2: List-1 List-2
1. Silver A. Cinebar
2. Mercury B. Horn silver
3. Copper C. Galena
4. Lead D. Cuprite
Codes:
AB C D
(a) 2143
(b) 1234
(c) 2134
(d) 1243
Ans. – (a)
Silver – Horn silver
Mercury – Cinebar
Copper – Cuprite
Lead – Galena (PbS) Q6. Which of the following statements
regarding ester is not correct:
(a) The reaction between an ester and an acid is termed as
Saponification.
(b) Ester is formed by reaction of acids and Alcohol.
(c) The conversation of an acid into an ester termed as esterification.
(d) All of the above are correct.
Ans. – (a) Saponification is the reaction
between a metallic base and an easter. When decomposition of an
easter occurring upon its reaction with water, the easter said to
be/know as hydrolyzed. So in the given option (a) is not correct about
easter. Q7. Consider the following statements and select the
correct code/answer:
(1) Methyl is usually referred to simply as ether.
(2) Most ether are volatile, light, flammable liquids.
(3) Chemically, the ether are inert, stable compounds, not readily
affected by alkales of acid.
Codes:
(a) 1 and 3 are correct
(b) 2 and 3 are correct
(c) Only 3 is correct
(d) 1, 2 and 3 are correct
Ans. – (b) Ethyl ether is simply referred to
as ether/edger. The chemical formula of an
edger is ‘R–O–R’, in which O is an oxygen
atom and R and R represent either the same
of different organic radicals.
Q.8 Consider the following statements and
select the correct code/answer:
(1) Monel metal is an alloy of nickel, magnese, iron and copper.
(2) Element like gold, copper and silver which are in the group of II
they periodic table are known as coinage metal.
Code:
(a) Only 1 is correct
(b) Only 2 is correct
(c) 1 and 2 are correct
(d) 1 and 2 are incorrect
Ans. – (c) In the case of coinage metal,
they have some common properties like,
they are generally found in native state as
they are not very reactive, acid, water and
air have slow activity on these elements,
these are heavy, hard, ductile and malleable.

Q9. Given below are name of some alloys and their constituents.
Match them and select correct answer/code:
Composition
(A) Copper and nickel
(B) Mg, Zn, Al and Mn
(C) Iron, Mn and Carbon
(D) Antimony, Sn (tin), Cu and Bismuth Alloy
(1) Constantum
(2) Electrum
(3) Poweter
(4) Splege leisen
Codes:
AB C D (a) 1 2 4 3
(b) 1 2 3 4
(c) 2 3 1 4
(d) 4 2 1 3
Ans. – (a)
Cu and Ni – Constantum Mg, Zn, Al and Mn – Electrum

– Spiege leisen Iron, Mn, Carbon Antimony, Sn, Cu and Bismath


– Poweter

Q10. Given below are the name of certain chemicals match with
their medicinal use and select correct code:
List-1
(A) Iodex
(B) Asprin
(C) Malarial drug
(D) Orthopedic surgery
List-2
(1) Chloroquine
(2) Methyl salicylyte
(3) Cephalosporins
(4) Saticyclic acid
Codes:

AB C D (a) 2413
(b) 4213
(c) 3421
(d) 1234
Ans. – (a)
Iodex – Methyl salicylyte Asprin – Salicyclic acid Malarial drug –
Chloroquine
Orthopedic surgery – Cephalosporins Q11. Which one of the
following type of penicillin is active against Pseudomonas:
(a) Extended spectrum penicillin
(b) Ampicillin
(c) Penicillin-G
(d) None of these
Ans. – (a) The extended spectrum penicillin are used against
infection cause by

gram – negative pseudomonas bacteria, a particular problem in the


hospital or clinic. So in the given option (a) is the answer of given
question.

Q12. Consider the following statements and select the correct


code/answer:

(1) Penicillinase resistants are bacteria that have developed


resistance to
penicillin-G.

(2) Penicillin used to treat disease as


gonorrhea, meningitis, anthrax, yaus and syphilis.

Code:
(a) 1 and 2 are incorrect
(b) 1 and 2 are correct
(c) Only 1 is correct
(d) Only 2 is correct
Ans. – (d) Penicillinase resistant are penicillin that combat bacteria
that have developed resistance to penicillin-G. So only option (d) is
correct.

Q13. Which of the following drug is used to treat osteoarthritis:


(a) Tramadol (b) Endolac
(c) Penthrane (d) None of these
Ans. – (b) Endolac is used to treat osteoarthritis. It is non-steroid,
mild analgesic that work by reducing concentrations of certain
chemicals of modify tissue are produce inflammation and pain,
relieving swelling and stiffness.

Q14. Match the following drug with their uses and select the
correct code/answer: (A) Meningitis (1) Rifamicin (B) Headache and

structural muscle
aches (2) Macrolides (C) Diptheria (3) Cephalosporins
(D) Tuberculosis (4) Accetaminophen Codes:

AB C D
(a) 1234
(b) 3421
(c) 1432
(d) 4213
Ans. – (b)
A– 3
B–4
C–2
D–1
Q15. Consider the following statements and select correct code:
(1) Hess law is used in calculating heat of formation and also heat of
combustion.

(2) Hess law states that the quantity of heat absorbed or evolved in a
chemical change varies with the step in the process.

Code:
(a) 1 and 2 are not correct
(b) 1 and 2 are correct
(c) Only 2 is correct
(d) Only 1 is correct
Ans. – (c) According to hess law constant heat summation the
quantity of heat absorbed in a chemical change is always same
irrespective of the fact that process take place in one step or several
step.

Q16. Which of the following is general formulae of Ketones:


(a) R–CO–R’ (b) R–O–R (c) R–COOH (d) RNH2
Ans. – (a)
The chemical of formula of Ketones is R–CO–R’. Ketone is a group of
chemical compound containing a carbonyl group, C = O attached to
two organic groups, for example, acetone which has chemical
formulae is CH3COCH2.
Q17. Which of the following is burned in oxygen, producing a
very hot flame using in cutting of metal and welding in metal: (a)
Acetylene (b) Butane
(c) Dopamine (d) Ethylene Ans. – (a) For cutting and welding of
metal Acetylene is used. Acetylene is also used in the manufacturing
of vinyl chloride for plastic, acetaldehyde and neoprene type of
synthesis rubber.

Q18. Match the following compound with their chemical


formulae:
List-1 List-2
(A) Propane (1) CH4 (B) Butane (2) C4H10 (C) Pentane (3) C5H12 (D)
Methane (4) C3H8 Codes:

AB C D
(a) 1 2 3 4
(b) 4 2 3 1
(c) 3 2 1 4
(d) 4 2 3 1
Ans. – (b)
Propane – C3H8
Butane – C4H10
Pentane – C5H12
Methane – CH4
Q19. Match the following lists and select correct code:
List-1
(A) Orange (B) Lemon (C) Curd
(D) Tamarind

List-2
(1) Citric acid (2) Lactic acid (3) Tarataric acid (4) Citric acid

Codes: AB C D
(a) 1 2 3 4
(b) 1 4 2 3
(c) 1 3 2 4
(d) 4 2 3 1
Ans. – (b)
Orange – Citric acid
Lemon – Citric acid
Curd – Lactic acid
Tamarind – Tarataric acid Q20. A bird flies down vertically towards
a
water surface, to a fish inside the water,
vertically below the bird, the bird will
appear to:
(a) be further away than its actual distance.
(b) be closer than its actual distance.
(c) move faster than its actual speed.
(d) move slower than its actual speed. Choose correct options:
(1) a and c (2) Only d
(3) All of the above (4) a, b and c
Ans. – (1) For refraction at plane surface,
use
μ2 = μ2
Vu
Let n = height of bird above the water surface
For travelling from bird to the fish
μ1 = 1, μ2 = μ (R.I of water), μ = – x

μ=1
V −n

V= – μn
|V| = μn
Speed of bird is greater than actual speed.

Q21. What is the reason behind twinkling of stars?


(1) Atmospheric Refraction
(2) Scattering of Light
(3) Tyndal Effect
(4) Total internal Reflection
Ans. – (1) The twinkling of a star due to atmospheric refraction of
starlight. The starlight, on entering the earth atmosphere, undergoes
refraction continuously before it reaches the earth. The atmospheric
refraction occurs in a medium of gradually changing R.I. Since the
atmosphere bends starlight towards the normal, the apparent position
of stars is slightly different from its actual position. The apparent
position of star fluctuates due to change in physical condition of earth
atmosphere and the amount of starlight entering the eye flickers.

Q22. The human eye can focus objects at different distances by


adjusting the focal length of the eye lens. This is due to: (a)
Presbyopia
(b) Accommodation
(c) Near sightedness
(d) Far-sightedness
Ans. – (b) The ability of eye to focus both near and distant objects,
by adjusting its focal length, is called the accommodation of the eye.
The smallest distance, at which the eye can see objects clearly
without strain, is called the near point of the eye or the least distance
of distinct vision. For a young adult with normal vision, it is about 25
cm.
The common refractive defects of vision include myopia,
hypermyopia and presbigopia.
Myopia (short sightedness – the image of distant objects is focus
before the retina).

Q23. Match the following List-1 and List-2: Institutions


(1) Indian institute of Astrophysics (2) Indian institute of Petroleum (3)
Indian institute of Chemical

technology
(4) Indian institute of Chemical Biology Location
A. Banglore
B. Dehradun
C. Kolkata
D. Hyderabad
Codes:

AB C D
(a) 1243
(b) 1243
(c) 3412
(d) 1324
Ans. – (a)
Indian institutes of
Astrophysics – Banglore Indian institutes of
Petroleum – Dehradun Indian institutes of Chemical Technology
Indian institutes of Chemical Biology
– Hyderabad

– Kolkata

Q24. Match the following List-1 and List-2: List-1 Compound


(1) Sodium bicarbonate (NaHCO3) (2) Zipsum (CaSO4 . 2H2O)
(3) Calcium Carbide (CaC2)
(4) Mercuric Oxide (HgO)
List-2 Uses
A. Fire extinguisher
B. Cement C. Acetelene D. Poison Codes:

AB C D
(a) 1 2 3 4
(b) 1 2 4 3
(c) 2 3 4 1
(d) 2 3 1 4
Ans. – (a)
Sodium bicarbonate
(NaHCO3) – Fire extinguisher Zipsum – Cement Calcium carbide –
Acetelene Mercuric Oxide – Poison
Q25. Match the following List-1 and List-2: List-1
(1) Antibiotics (2) Antiseptics (3) Antipyretics (4) Anaesthetic

List-2
A. Phenols
B. Streptomycin C. Aspirin
D. Penthol sodium
Codes: AB C D
(a) 2 1 3 4
(b) 1 2 4 3
(c) 2 3 1 4
(d) 2 3 4 1
Ans. – (a)
Antibiotics – Streptomycin
Antiseptics – Phenols
Antipyretics – Aspirin
Anaesthetic – Penthol sodium

Technological Developments

Q1. Match the following lists: List-1


A. Concave mirror
B. Convex mirror
C. Concave lens
D. Convex lens
List-2
1. Only virtual image
2. Virtual and real image 3. Only virtual image
4. Both virtual and real image Codes:

AB C D
(a) 1 2 3 4
(b) 2 1 3 4
(c) 1 2 4 3
(d) 2 3 1 4
Ans. – (b)
Concave mirror – Real and virtual image
Convex mirror – Only virtual image Concave lens – Only virtual image
Convex lens – Both real and virtual image
Q2. Match the following lists:
List-I List-II
A. Mirror formula 1. Radius of curvoture (R) 2

B. Focal length 2. 1 + 1 + 1
vuf
C. Magnification 3. h’/h
D. Lens formula 4. 1 − 1 + 1 v u f
Codes:

AB C D (a) 2 1 3 4
(b) 1 2 4 3
(c) 2314
(d) 1243
Ans. – (a)

Mirror formula – 1 + 1 + 1
vuf
Lens formula –1 − 1 + 1 v u f
Radius of curvoture (R)Focal length – 2
Magnification – h’/h

Q3. When a body running with the acceleration (a) suddenly his
mobile ring and he stop; when body stop then acceleration of
the body will be:
(a) Increases
(b) Decreases
(c) Does not change
(d) None of these

Ans. – (b)
The body when stop suddenly then velocity of the body is also
decreases so according to definition of acceleration the velocity
changing with respect to time is called acceleration it means velocity
increases then acceleration also increases and when velocity
decreases then acceleration also decreases. So option (b) is correct.

Q4. Match the following lists:


A. Coulomb’s law 1. k × q1 × q2
r2
B. Ohm’s law 2. I = V/R
C. Gravitation law 3. G1 × M1 × M2 R2
Codes:
AB C (a) 1 3 2
(b) 2 3 1
(c) 1 3 2
(d) None of these

Ans. – (d)
Coulomb’s law

k × q1 × q2

r2
ohm’s law – I = V/R

Gravitation law – G1 × M1 × M2 R2

Q5. Assertion (A): A piece of copper and a piece of glass are heated
to the same temperature. When touched, thereafter, the copper piece
appear hotter than the glass piece.
Reason (R): The density of copper is more than that of glass.
Codes:
(a) Both A and R true but R is not correct

explanation of A.
(b) A is true but R is false.
(c) Both A and R true and R is the correct

explanation of A.
(d) None of these.
Ans. – (a) The copper piece appear hotter because of low specific
heat capacity not because density.

Q6. The image form by a concave mirror is observed to be a


virtual, erect and larger than the object. Where should be
position of the object?
(a) Between the principal and the centre

of curvature.
(b) At the centre of curvature.
(c) Beyond the centre of curvature. (d) Between the pole of the mirror
and its

principal axis.
Ans. – (d) When an object situated between pole of the mirror and
principal focus of the concave mirror then the image of that object
should be formed erect, larger than object and virtual.

Q7. Assertion (A): With the increases of temperature viscosity of


glycerine increases. Reason (R): Rise of temperature increases
kinetic energy of molecules.
(a) A is true but R is false.
(b) A is false but R is true.
(c) Both A and R are true but R is not

correct explanation of A.
(d) Both A and R true and R is correct
explanation of A.
Ans. – (d) The viscosity of liquid is directly proportional to
temperature. Rise of

temperature increases the kinetic energy of molecules that result


increases of temperature of glycerine that ultimately increases the
viscosity of glycerine.

Q8. An electric bulb of 60 W is used for 6 hour per day. What is


the ‘units’ of energy consumed in one day by the bulb? (a) 1 unit
(b) 2 unit

(c) 0.36 unit (d) 0 unit


Ans. – (c)
Power of electric
bulb = 60 W

= 0.06 KW
Time used, t = 6 h
Energy = Power× time taken
= 0.36 KW × 6 h = 0.36 unit
1 Kilowatt = 1000 W

Q9. The rate change of momentum of an object is proportional to


the applied unbalanced force in the direction of the force. This
rule is know as:

(a) Newton, first law of motion


(b) Gravitational law
(c) Newton, third law of motion (d) None of these
Ans. – (d) The rate change of momentum of an object is proportional
to the applied unbalanced force in the direction of the force is known
as Newton’s second law of motion. This rule is know as Newton 2nd
law of motion and given by Sir Issac Newton.

Q10. ‘To every action, there is an equal and opposite reaction


and they act on two different bodies.’ This rule is known as: (a)
Coulombs law
(b) ohm’s law

(c) Newton third law of motion


(d) None of these
Ans. – (c) To every action, there is an equal and opposite reaction
and they act on two different bodies. This rule is known as Newton
third law of motion. Given by Sir Issac Newton. Ohm’s and coulombs
law are related to current and electricity not motion.

Q11. The motion of a body could be uniform or non-uniform


depending on the: (a) Acceleration (b) Force
(c) Kinetic energy (d) Velocity Ans. – (d) The motion of a body could
be uniform or non-uniform is depend on the velocity of body. When
velocity uniform then motion of the body is also uniform and when
velocity non-uniform then motion of the body also non-uniform. When
velocity of the body uniform then acceleration will be zero.

Q12. Work done on an object by a force would be zero if the


displacement of the object will be:
(a) Increase (b) Decrease (c) Zero (d) None of these
Ans. – (c) Work done on an object by a force would be zero if the
displacement of the object is also zero. According to definition of
work, work = force × displacement. So when displacement zero force
also zero and when force zero then displacement also zero and
totally work done by the object is zero.

Q13. What is the momentum of an object of mass 20 kg and


moving with velocity 20 m/s?
(a) 200 kg m/s (b) 300 kg m/s (c) 400 kg m/s (d) None of these Ans.
– (c) The momentum of a body/object is calculated by P = MV where
P = momentum, M = mass and V = velocity
So P = 20 × 20

P = 20 kg × 20 m/s P = 400 kg m/s

Q14. Match the following lists:


A. Momentum 1. Kg m/s
B. Electric charge 2. Coulombs C. Energy 3. Joule
D. Mass 4. Kg (Kilogram) Codes:

AB C D
(a) 3241
(b) 4231
(c) 1243
(d) 1234
Ans. – (d)
Quality SI Unit
Momentum – Kg m/s
Electric charge – Coulombs Energy – Joule
Mass – Kilogram Q15. The potential difference across the end of a
resistor is directly proportional to current through it, provided
its temperature remain the same. This rule is known as:
(a) ohm’s law
(b) Coulomb’s law
(c) Kepler’s law
(d) None of these
Ans. – (a) The potential difference across the end of a resistor is
directly proportional to current through it, provided its temperature
remain the same. This rule is given by Georg Simon ohm’s in 1827.
Q16. A particle travels 16 m in 4 s and then another 16 m in 2 s.
What is the average speed of the particle?
(a) 10 m/s (b) 4 m/s
(c) 5.33 m/s (d) 6.43 m/s

Ans. – (c)
Average speed of a particle
= Total distance travelled

Total time taken


So Total time = 4 + 2 = 6s
Total Distance = 16 + 16 = 32 m according to formula
Average speed = 32/6 = 5.33 m/s Q17. An object continuous to be
in a state of rest or uniform motion along straight line unless
acted upon by an unbalanced force. That rule is known as:
(a) Newton third law of motion (b) Newton second law of motion (c)
Newton first law of motion
(d) None of these
Ans. – (c) An object continuous to be in a state of rest or uniform
motion along straight path unless acted upon by an unbalanced force.
This rule is Newton first law of motion. Given by Sir Issac Newton.
Q18. Where should an object be placed in front of convex lens to
get real image of the size of the object?
(a) At the principal focus of the length. (b) At twice the focal length.
(c) At infinity.
(d) Between the optical centre of the lens
and its principal focus.
Ans. – (b) When an object situated at the twice of focal length, the
image formed real size of the object and opposite. Q19. A current
0.5A is flow through filaments of an electric bulb for 10 minutes.
What is the amount of electric charge that flow through the
circuit?
(a) 150 C (b) 200 C
(c) 300 C (d) None of these Ans. – (c) Given in the above question
are Current= 0.5 ampere
Time = 10 minutes = 10 × 60 = 600 C So I = Q/T or Q = I × T
Q = 0.5 × 600 = 300 C Q = 300 C
Q20. Match the following lists:
List-I List-II A. Work (W) 1. Volt B. Current (I) 2. Ampere C. Charge
(electric) 3. Joule D. Voltage (electric
potential difference) 4. Coulombs Codes:
AB C D
(a) 1 2 3 4
(b) 4 3 2 1
(c) 2 3 4 1
(d) 3 2 4 1
Ans. – (d)
Quality SI Unit Work – Joule Electric charge – Coulomb Current –
Ampere Electric potential difference – Volt Q21. A set of resistor
connected in parallel has been equivalent resistance RP is equal
to:

(a) 1 = 1 + 1 + 1 + 1 + .......
RP R1 R2 R3 R4
1 3 4
(b) 1 = R + R + R + .......
RP R2 R1 R2..
(c) 1 = 0
RP

(d) None of these


Ans. – (a) When set of resistor connected in parallel then equivalent
resistance (RP) is equal to

1 = 1 + 1 + 1 + 1 + .......
RP R1 R2 R3 R4
For Domestic use we set the circuit path in parallel not in series.

Q22. A set of a resistor connect in series has equivalent


resistance is Rs equal to: (a) 1 = 1 + 1 + 1 + .......

Rs R1 R2 R3

(b) RS = R1 + R2 + R3 + ....
(c) RS = 0
(d) None of these
Ans. – (b) When a set of resistor connected in series then equivalent
resistance (RS) is equal to RS = R1 + R2 + R3 + R4 .... In series
connection current same but voltage different.

Q23. A sound wave has frequency 2KHZ and wavelength 35 cm.


How long will it take travel 1.5 km?
take (1KHZ = 1000 HZ)
(a) 3s approx
(b) 2 second approx
(c) 4 second approx
(d) None of these
Ans. – (b) Frequency, V = 2KHZ = 2000 HZ, wavelength γ = 35 cm =
0.35 m We know that speed of the wave = wavelength × frequency
⇒V=λ × V
= 0.35 m × 2000 HZ = 700 m/s
Speed = 700 m/s

Time = 1.5 = 1500 = 2.1 Sec


700m/s 700
= 2 sec. approx
Q24. The mass of a body on earth is 100 kg

(acceleration due to gravity, ge = 10 m/s2). If acceleration due to


gravity on the moon is ge/6, then the mass of the body on the
moon is:
(a) 700 kg (b) 100 kg/6
(c) 100 kg (d) None of these

Ans. – (c)
The mass of the body does not change with respect to acceleration
due to gravity. The masses does not change from one place to other
places but weight can change from one place to other place. Weight
may be zero.

Q25. Assertion (A): The boiling point of water decreases as the


altitude increases. Reason (R): The atmospheric pressure increases
with altitude.
(a) A and R both false.
(b) A true but R false.
(c) A and R both are true and R is the

correct explanation of A.
(d) A and R both true but R is the not
correct explanation of A.
Ans. – (b)
A is true but R is false. At altitude the boiling point of water
decreases. This is so because at height the atmospheric pressure
decreases.

Miscellaneous Questions

Q1. Consider the following statements relates to gene therapy is


correct: (1) It can correct inherited disorders. (2) The idea of gene
therapy first

occurred to Dr. W. French Anderson. (3) It includes the cloning


process. (4) It is approved for genetic

manipulation of reproductive cells. Choose the correct option:


(a) 1, 2 and 3
(b) Only 4
(c) All of the above
(d) 3 and 4
Ans. – (a) Gene therapy is not approved for genetic manipulation of
reproductive cells. In this therapy faulty gene is identified first and
copies of its normal version are produced by cloning. Then suitable
host cells are selected from the patients body and genetic
engineering is used to transfer copies of the normal version of gene
into it.

Q2. Consider the following statements regarding ‘Aerogel’:


(1) It is pure silicon dioxide (SiO2). (2) It is the highest solid substance
ever
created.
(3) This hi-tech foam is used in space
exploration.
(4) It has 50% air in its tiny pockets. Consider the correct options:
(a) 1 and 2 (b) 3 and 4
(c) 1, 2 and 3 (d) Only 4
Ans. – (c) Aerogel is pure silicon dioxide (SiO2). It is the highest solid
substance ever created. It’s hi-tech foam is used in space
exploration.
Aerogel has 99.5% air in its tiny pockets called nanopores. If a cubic
inch of aerogel is faltened. It would yield a surface bigger than
football field.
Q3. Consider the following statements related to tissue culture:
(1) Tissue culture involves manipulation
at the genetic level.
(2) In culture, cells are produced under
controlled condition in a laboratory. (3) Tissue culture allows
multiplication
of selected elite plants in large
numbers in much shorter time. (4) National chemical lab. Pune is
credited with induced flowering of
bamboo.
Choose the correct options:
(a) 2, 3 and 4 (b) Only 1
(c) 2 and 3 (d) All of the above Ans. – (a) Tissue culture does not
involve manipulation at the genetic level. National Chemical Lab. had
a remarkable achievement when scientists succeeded in developing
a technique of germinating seeds of bamboo and then growing them
in culture medium containing coconut milk and some growth
hormones.
Q4. Match the following and choose correct code:
1. A member of carbon
reduction group 2. Inter stellar group 3. Plastic chemical 4.
Translation device A. Phthalates B. Local Fluff C. Toolkit
D. Cragger
Codes: AB C D (a) 3 2 4 1 (b) 1 2 3 4 (c) 3 2 1 4 (d) 1 2 4 3 Ans. –
(a)
Phthalates – Plastic chemical Local Fluff – Inter stellar cloud Toolkit –
Translation device Cragger – A member of carbon reduction group
Q5. Consider the following statements regarding ‘Fullerenes’:
(1) It is the allotropes of Fluorine. (2) It consists of hexagonal carbon
atoms only.
(3) It is spherical shape like football. (4) It was discovered by Harold
Kroto,
Richard Smalley and Robert Curl. Choose the correct codes:
(a) 3 and 4 (b) 1 and 2 (c) All of the above (d) None of these Ans. –
(a) Fullerenes consist of hexagonal and pentagonal carbon atom. It is
the allotropes of carbon. Fullerenes molecules are soccer ball shaped
(spherical). It may be used as lubricants or molecular ball bearing. It
was discovered by team of Harold Kroto, Richard Smalley and Robert
Curl.
Q6. If a body start from rest and moving with 10 m/s with
acceleration 2m/s2 then distance travelled by body is?
(a) 40 m (b) 60 m
(c) 25 m (d) 100 m
Ans. – (c) Here given,
Initial velocity = 0 (body start from rest) Final velocity = 10 m/s
Acceleration = 2 m/s2
Distance (S) = ?
V2 = U2 + 2as
(10)2 = (0)2 + 2 × 25 100 = 48
45 = 100
S = 25 m
Q7. When 10 N force exerted on a body but body does not move
or replace from its original position then work done by body?

(a) Zero (b) 10 Joule


(c) 20 Joule (d) None of these Ans. – (a) If 10 N force exerted on a
body but body does not move or replace from its original position then
work done by body is zero. Because work is the product of force and
displacement so if displacement or force any one quantity is zero
then work done by any object is also zero.

Q8. Match the following with List-1 to List2:


List-1
(A) Nuclear model

of atom
(B) Electron
(C) Proton
(D) Neutron

List-2

(1) Chadwick (2) Rutherford (3) Thomson (4) Goldstien

Codes: AB C D
(a) 1234
(b) 2341
(c) 4321
(d) 1342
Ans. – (b)
Nuclear model of atom – Rutherford
Electron – Thomson
Proton – Goldstien
Neutron – Chadwick Q9. When a body move on the circular path
then its acceleration is?
(a) Constant
(b) Does not constant
(c) Both a and b
(d) None of these
Ans. – (b) When any object or body moving on a circular path then its
acceleration
is change due to changing the direction of
body. Acceleration is a vector quantity so it
have magnitude as well as direction. When
velocity of body increase then also increases the acceleration of
body.

Q10. Which among the following is the instrument which


measure radiation level: (a) Radometer (b) Dosimeter (c)
Chronometer (d) Pyrometer Ans. – (b) A dosimeter measures an
individual or an object’s exposure to something in the environment
particularly to hazard inflicting cumulative impact over a long period of
time. A radiation dosimeter measures exposures to ionizing radiation.
In a bid to measure radiation levels, the government of India has
decided to install radiation dosimeter across 35 cities in the country.

Q11. Match the following lists:


List-1
(A) Sonar
(B) Photocell
(C) Optical fiber
(D) Computer
List-2
(1) Total internal reflection of light (2) Photoelectric effect
(3) Reflection of ultrasonic wave (4) Digital logic
Codes:

AB C D
(a) 1234
(b) 3214
(c) 4312
(d) 1243
Ans. – (b)
Sonar – Reflection of ultrasonic wave Photocell – Photoelectric effect
Optical fiber – Total internal
reflection of light Computer – Digital logic Q12. Which one of the
following is used as propellants for cryogenic engine:
(a) Ammonium perchlorate
(b) Nitrogen tetraoxide and hydrazine (c) Hydrogen peroxide and
hydrazine (d) Liquid oxygen and liquid hydrogen Ans. – (d) A
cryogenic engine is a liquid propellant rocket motor which uses liquid
oxygen and liquid hydrogen as propellants. The propellants have to
be stored at extremely low temperature of around 200 degree
Celsius. It has greater use in space exploration. America, Russia has
already cryogenic engine. Cryogenic engine works at low
temperature.
Q13. Which one of the following has been designated by
scientists as the thinnest substance in the world:
(a) Fullerene
(b) Carbon
(c) Quartz
(d) Graphene
Ans. – (d) Graphene has been called by the scientist as the thinnest
substance in the world. It is made up of single layer of carbon atoms.
Carbon atoms in Graphene are arranged in honeycomb shaped
hexagonal lattice. The substance is chemically very simple but
extremely strong, conducts electricity and heat. It is transparent. Q14.
Match the following lists:
List-1 List-2
(A) Length (1) Meter (B) Mass (2) Kilogram (C) Electric Current (3)
Second (D) Time (4) Ampere Codes:
AB C D (a) 1 2 4 3 (b) 1 2 3 4 (c) 4 3 2 1 (d) 1 4 3 2 Ans. – (a)
Length – Meter Mass – Kilogram Electric Current – Ampere Time –
Second Q15. Consider the following statements and select
correct code:
(1) Going at the pace, India diabetic patient would increase three fold
by the year 2030.
(2) India has the largest number of diabetic patient in the world.
Codes:
(a) 1 and 2 are not correct
(b) Only 1 is correct
(c) Only 2 is correct
(d) Both 1 and 2 are correct
Ans. – (a) About diabetic patient China has largest number of
diabetic patient in the world. The number of patient is 90 million that
will increase 130 million by 2020. India presently has 61 million and it
marks 100 million patient of diabetic at 2030. Q16. Which of the
following is not characteristics of Long Term Evolution
Technology:
(a) It increase latency.
(b) It increase spectral efficiency. (c) It reduce cost of transmission.
(d) None of these.
Ans. – (a) The latency is reduced to 10 ms from few 100 ms is 3G by
long term evolution technology. The objective of LTE is to improve
spectral efficiency. The long term technology reduces the cost of
transmission. The LTE technology does not increase the latency so
option is not correct about LTE.
Q17. Match the following lists:
Object Mass
(A) Electron (1) 10–30 kg (B) Proton (2) 10–27 kg (C) Sun (3) 1030 kg (D)
Milky way Galaxy (4) 1041 kg

Codes: AB C D
(a) 4312
(b) 1243
(c) 1234
(d) 4321
Ans. – (c)
Electron – 10–31 kg
Proton – 10–27 kg
Sun – 1030 kg
Milky way Galaxy – 1041 kg
Q18. Which one of the following statements

regarding Advance Technology Vehicle (ATV-DO1) is not correct:


(a) It is the biggest sounding rocket system of India.

(b) It’s speed is more than 6 Mach. (c) This vehicle is developed by
ISRO. (d) This rocket carries oxygen and fuel on

board which provides the forward thrust.


Ans. – (d) This vehicle is developed by
Indian Space Research Organisation. It’s
speed is more than 6 Mach (1 Mach = 320
m/s). It is an air breathing rocket which
uses atmospheric oxygen from their surroundings and burned it with
the onboard
fuel to produce the forward thrust. Q19. Match the following lists:
(A) Measurement of electronic charge
(B) Wave nature of matter
(C) Thermal ionisation
(D) Transistor
(1) Milikan (2) Borgile (3)M.N. Saha (4) Bardeen Codes: AB C D (a)
1234 (b) 4321 (c) 3421 (d) 2314
Ans. – (a)
Measurement of
electronic charge
Wave nature of matter Thermal ionisation
– Milikan
– Borgile
– M.N. Saha

Transistor – Bardeen Q20. Consider the following statements and


select correct code:
(1) Momentum of body is the product of its mass and velocity.
(2) Force on a body is the product of its mass and acceleration.
Codes:
(a) 1 and 2 are incorrect
(b) 2 is correct
(c) Only 1 is correct
(d) 1 and 2 are correct
Ans. – (d) The momentum of a body is the
product of its mass and velocity. If a body
have more mass or velocity then momentum of the body is also very
high. Mass and
acceleration are the product of force on a
body.
Q21. Consider the following statements and
select correct code:
(1) Distance is always equal to
displacement.
(2) Distance never be zero.
(3) Displacement may be zero.
Code:
(a) 1, 2 and 3 are correct
(b) 2 and 3 are correct
(c) Only 3 is correct
(d) Only 1 is correct
Ans. – (b) The distance or path length is
not always equal to displacement. Distance
never be zero because when anybody move
from his/her original position the distance
must be travelled. Displacement may be
zero when anybody start from A and after
sometime he comes back to A then displacement of body is zero.

Q22. If a body has initial velocity is 5m/s and final velocity is 15


m/s and time is 48 then acceleration of body is?
(a) 50 m/s2 (b) 25 m/s2
(c) 10 m/s2 (d) 2.5 m/s2 Ans. – (d)
We know that
V = U + at (where V = final velocity, u = initial velocity and t = time)
15 = 5 + 4a
15 – 5 = 4a
10 = 4a
4a = 10
a = 5/2 = 2.5 m/s2

Q23. Match NASA’s following space missions with their


respective objective and select the correct codes:
Mission
1. 2. 3. 4. Path finder
Voyager-1
Cassini-Huygens Juno

Probe
A. Jupiter
B. Outer solar system C. Mars
D. Saturn
Codes:

AB C D
(a) 4 2 1 3
(b) 4 2 3 1
(c) 1 2 3 4
(d) 3 2 4 1
Ans. – (a)
Path finder – Mars
Voyager-1 – Outer solar system Juno – Jupiter
Cassini-Huygens – Saturn
Q24. Given below are some new nuclear reactor sites of India.
Match them with the states in which they are located: Sites
A. Kumhariya B. Haripur
C. Bargi
D. Kovvadda Codes:
States
1. Haryana
2. West Bengal
3. Madhya Pradesh 4. Andhra Pradesh

AB C D (a) 1 2
(b) 1 2
(c) 3 2
(d) 3 4
Ans. – (a) Kumhariya Haripur
Bargi
Kovvadda 3 4 4 3 4 1 1 2

– Haryana
– West Bengal
– Madhya Pradesh
– Andhra Pradesh Q25. Consider the following statements relat

ed to cloning is correct:

(1) A clone is derived from a single parent by non sexual methods.


(2) A clone is not only physically but genetically identical to its parent.
(3) All cells in a body are genetically not identified to each other.
(4) At the heart of cloning lies ‘Nucleus transfer’.
Choose the correct options:
(a) 1 and 2
(b) 2 and 3

(c) 1, 2 and 4
(d) None of these
Ans. – (c) The breakthrough of cloning solves a mystery of embryo
development. All cells in a body are genetically identical except the
sperm and the egg. Yet the cells of skin, the eye, the heart and the
liver are all different. Though their blue prints are identical, they
become different by switching on certain genes at different times.

Q26. What is the reason for rising and settling sun appearing
reddish?

(a) Refraction is responsible for it. (b) Diffraction sends red ray to
earth. (c) It occurs due to total internal

reflection.
(d) Scattering of light due to dust particle
and air molecules.
Ans. – (d) Rising and settling sun looks reddish due scattering of light
due to dust particle and air molecules.
Refraction is a process, when light goes from one medium to another
medium. It moves away/towards the normal. When light goes from
rarer to denser it moves towards normal and vice versa.
Diffraction – It occurs when a wave encounter an obstacle. It is a
bending of wave around small obstacle.

Part - 4
Chhemiissttry
Radioactivity

Q 1. Which of the following statements is/are correct:


(1) α-particle is positively charge

particle.
(2) Penetrating power of β-ray is more
than γ-ray.
(3) Atomic no. of parents element is
reduced by two unit when α-ray
is emitted.
(4) Radioactivity is measured by G.M.
Counter.
Choose correct options:
(a) 1 and 2 (b) 1 and 3
(c) 1, 2 and 3 (d) None of these Ans. – (c) α, β, γ-rays are emitted by
radioactive elements. Radioactivity discovered by Henory Becquard.
α-particle is

doubly positive charge particle. When α- ray is emitted atomic


number of parent element is reduced by two unit and atomic mass
no. is reduced by 4 unit. Penetrating power of γ-ray is more than α
and β-rays. Radioactivity is measured by G.M. Counter.

Q 2. Consider the following statements:

(1) Semiconductor behaves like insulator at room temperature.


(2) Doping is a process in which impuri ties are added to intrinsic
semiconductor.
(3) Superconductivity is discovered by Kemerlingh.
(4) At 4.2 k mercury behaves like superconductor.
Choose correct options:
(a) Only 4 (b) 2 and 3
(c) None of the above (d) All of these
Ans. – (d) Semiconductor is a solid material whose conductivity lies
between good
conductor and good insulator. It has negative temperature gradient.
At room temperature it behaves like insulator. Intrinsic
semiconductor has low conductivity so for
enhancing conductivity doping process is
done. Doping is a process in which impurities are added to pure
semiconductor. At
4.2k mercury behaves like superconductor. Q 3. Which one of the
following statements is
correct:
(a) Silver and copper are found in nature free states as well as
combined states.
(b) Bromine is a liquid metal.
(c) The atomic number of sodium is 12.
(d) None of these.
Ans. – (a) Bromine is a non-metal liquid.
The atomic number of sodium is 11 and 12.
Sodium is highly react we with oxygen so
sodium keep in kerosene not as a free state.
Silver and copper which comes in the next
order of inactivity are found both in free
and state of combined.
Q 4. Which of the following is noble metals:

(a) Mercury (b) Zinc


(c) Copper (d) Platinum
Ans. – (d) Platinum in the given option is a noble metal which occurs
in the free states because it is completely unaffected by air, moisture
and other gases present in the atmosphere. It is this property and
perhaps their rarity which makes them expensive.

Q 5. Which of the following metals are/is noble metal:


(1) Silver (2) Gold
(3) Platinum (4) Copper Code:
(a) 1 and 2 only
(b) 1, 2 and 3 only
(c) 1, 2 and 4 only
(d) 1, 2, 3 and 4
Ans. – (b) Silver, gold, platinum are noble metals. They are
completely unaffected by air and moisture and are therefore given the
title of Aristocrats or Noble metals. It is this property and perhaps
their rarity which makes them expensive.

Q 6. Duraluminium used for aircraft construction consist of:


(a) Mg and Al
(b) Mg and Zn
(c) Mg, Al and Cu
(d) Mg, Al and Zn
Ans. – (c) Duraluminium used for aircraft construction consist of
Magnesium, Aluminium and Copper. So in the given option (c) is
correct. Magnesium, Aluminium and Copper all are good conductor of
heat and electricity.

Q 7. Consider the following statements and select the correct


code/answer:
(1) The light of burning Magnesium is

rich in ultraviolet.
(2) Magnesium is a bad conductor of heat
and electricity.
(3) Magnesium loses its luster when exposed to air due to the
formation of
a thin film of oxides.
Code:
(a) 1, 2 are correct
(b) 1, 2 and 3 are correct
(c) Only 3 is correct
(d) 1 and 3 are correct
Ans. –(d) Magnesium is a good conductor of electricity and heat. The
high burning magnesium is rich in ultraviolet rays that is why it is used
in making photoflash powder and bulb.
Q 8. Consider the following statements and choose the select
code/answer:
(1) Slab of impure metal are made the
anode and thin sheets of pure metal
the cathode is an electrolyte cell
using a suitable salt of the metal as
electrolytes.
(2) Electrolytes refining is the most
widely used method of refining
metals.
Code:
(a) 1 and 2 both are incorrect
(b) Only 1 is correct
(c) 1 and 2 are correct
(d) Only 2 is correct
Ans. – (c) During electrolysis, pure metal from electrolyte deposit on
the cathode while an equivalent amount of the metal from the anode
passes from the electrolyte. Less electropositive impurities fall down
while more electropositive impurities remain in electrolytes copper,
gold, aluminium, tin, silver, lead etc. are purified process.
Q 9. Babbitts is an alloy of which of the following principal
metal:
(a) Nickel (b) Copper
(c) Lead (d) Tin
Ans. – (d) Babbitts is an alloy which contains tin as its principal
metal. It also contains Antimony and Calcium. It is fusible and soft
and used for bearing. So option (d) is correct about principal metal of
babbitts. Q10. Which one of the following pairs of alloy and their
components is incorrectly matched:
(a) Solder – Pb, Sb
(b) Brass – Cu, Zn
(c) Nichrome – Ni, Fe, Cr, Mn (d) Gun metal – Cu, Zn
Ans. – (a) Brass contain zinc and copper, Nichrome contain nickel,
chromium, Magnese, Fe, Gun metal contain copper and zinc but
solder does not contain Pb and Sb because it contain Pb and Sn or
lead and tin.
Q11. Which was the first organic compound to be synthesized in
laboratory from non-living resources?
(a) Alcohol (b) Urea
(c) Ether (d) Formaldehyde Ans. – (b) A German Chemist Wohler
prepared urea in 1928. A typically animal product present in urine by
NH
4

CNO
Heat
−−→CO (NH2)2
Where (CONH2)2 is urea.

Q12. Which of the following variety or types of iron which


contain least/low amount of carbon:
(a) Alloy Steel (b) Cast Iron (c) Wrought Iron (d) Mild Steel Ans. – (c)
Cast iron contain 4 to 5% of impurities mostly carbon with low amount
of phosphorus, sulphur and silver. Cast iron has comparative low
melting point. In the case of wrought iron it contain least amount of
carbon about 0.15 to 0.5. So wrought iron is correct answer.

Q13. In the content of steel which one of the following correct


about annealing: (a) The process include slow cooling of

red hot steel.


(b) The hardness of steel decreasing
through this process.
(c) The hardness of steel is increasing
through this process.
(d) Both a and b.
Ans. – (d) The quality of anneal steel depend on the temperature to
which steel is heated and the process by which is cooled. The
softeners of steel increases when it heated to high temperature and
cooled slowly.
Q14. Which of the following is the fine/best method for
manufacturing steel from pig iron:
(a) Puddling process
(b) Open heart process
(c) Bessemer converter process
(d) Lintz-Donawich process
Ans. – (d) Lintz-Donawich (D-L, L-D) process in the most
modern/newly process for using in manufacturing steel. The rate of
production is fast. 100 tones steel produced in approx. 50 minute.
The quality of the steel is also very high.
Q15. Consider the following question and select correct
code/answer:
(1) Bakelite is prepared from starch and
for malin.
(2) Shollac is an example of natural
plastic.
(3) Celluloid was first plastic which is
made by man.
Code:
(a) Only 3 is correct
(b) Only 1 and 2 are correct
(c) All 1, 2 and 3 are correct
(d) 2 and 3 are correct
Ans. – (d) Bakelite is made from ammonia, phenol, formalin. It was
prepared by Doctor Backelend to lay the real foundation of the plastic
industry. Celluloid was the first plastic which is made by man. Q16.
Consider the following statements: (1) Boyle’s law states that at
constant tem
perature pressure is inversely propor
tional to volume.
(2) Real gases behaves like ideal gas at
low pressure and high temperature. (3) Charles’ states that at
constant pres
sure, volume of the gas is directly pro
portional to temperature.
(4) At normal temperature and pressure
volume of 1 gram molecule of
different gas contain 22.4 litre. Choose correct options:
(a) Only 4 (b) 2 and 3 (c) All of the above (d) 1 and 2 Ans. – (c)
Boyle’s law states that at constant temperature and given amount of
gas, volume of gas is inversely proportional to pressure.
Real gas behaves like ideal gas at low pressure and high
temperature. Charle’s law states that at constant pressure, volume of
gas is directly proportional to temperature. Q17. Consider the
following statements: (1) pH-scale represents Acidity and
Basicity of a solution.
(2) Blood is acidic in nature.
(3) Mixture of Sodium Acetate and Acetic
acid forms buffer solution.
(4) A substance is called Acidic if its pH
value is less than 7.
Choose the correct answer:
(a) 1, 3 and 4 (b) 2 and 3 (c) All of the above (d) 1 and 2 Ans. – (a)
pH = – log10 [H+]
of measures the Acidity and Basicity of a solution. pH value of blood
is 7.4 so it is basic in nature. The substance whose pH value is less
than 7 behaves like acid and pH value is greater than 7 behaves like
base.
Q18. Which one of the following is incorrect about cast iron:
(a) It cannot be welded or tempered. (b) It is bright and lacks
strength. (c) It is impure form of commercial iron. (d) It has
comparatively highly melting
point.
Ans. – (d) Cast iron is unsuitable for construction of bridge or other
devised where strength is imperative. It has comparative lower
melting point not higher. It contain 4 or 5 percent of impurities mostly
carbon with smaller quantity of sulphur, phosphorus and silicon. It is
suitable for small casting railway, weights fire-gates.
Q19. Consider the following statements and select correct
answer/code:
(1) slag is the compound formed by the
combination of flux and gangue. (2) Flux is the rocks in which ores
are
embedded.
(3) Gangue is a substance which is added
to ore during smelting.
Code:
(a) 1, 2 are correct
(b) 2, 3 are correct
(c) 3 and 1 are correct
(d) Only 1 is correct
Ans. – (d) Flux is a substance added to the ores during smelting to
combined with gangue and form an easily fusible compound so that it
may be tapped out. Gangue is a rock in which ores are embedded. In
the extraction of iron, gangue a sand flux is limestone. These to react
to form calcium silicate.
Q20. Below are pair of metal and their respective ores. Which
one of the following is incorrectly matched:
(a) Sodium – Malachite
(b) Lead – Cerrucite
(c) Zinc – Calamine
(d) Aluminium – Cryolite
Ans. – (a) Malaclachite CuSO4. CuCO3 is an ore of copper. Copper
pyrites (CuFeS2) and copper glance (Cu2S) are other ores of copper.
Galena (PbS) and cerrucite (PbCO3) are ores of lead. Zinc blend
(ZnS), clamine (ZnCO3) and Zincite (ZnO) are ores of zinc.
Q21. Match the following metal and their ores and select correct
code:
A. Zinc (1) Cerrucite
B. Lead (2) Cryolite
C. Aluminium (3) Calamine
D. Copper (4) Malachite
Codes:
AB C D
(a) 1234
(b) 4321
(c) 2431
(d) 3124
Ans. – (d)
Zinc – Calamine
Lead – Cerrucite
Aluminium – Cryolite
Copper – Malachite
Q22. Match the following metals with their ores and select
correct code:
(A) Zinc
(B) Aluminium (C) Lead
(D) Copper
(1) Zincite
(2) Bauxite
(3) Galena
(4) Copper glance (a) Gold occur in nature in free states. (b) Silver
copper are found in combined
only.
(c) Mercury is a liquid metal.
(d) Bromine is a liquid non-metals. Ans. – (b)
Silver and copper which come next in the order of inactivity are found
both in the free states as well as combined states. Gold being least
active of all metals is the only metals which occur in the free states
because it is unaffected by air.
Q24. Match the following List-1 and List-2: (A) Audiometer (1)
Intensity of sound (B) Cyclotron (2) Measurement of time
(C) Chronometer (3) Accelerator of charge particles (D) Barometer
(4) Air pressure Codes:
AB C D (a) 1 2 3 4 (b) 1 3 2 4 (c) 1 3 4 2 (d) 4 3 2 1 Ans. – (b)
Audiometer – Intensity of sound Codes: AB C D
(a) 1243
(b) 3241
(c) 1234
(d) 4312
Ans. – (c)
Zinc – Zincite
Aluminium – Bauxite
Lead – Galena
Copper – Copper glance
Q23. Which one of the following statements is not correct:

Barometer – Chronometer – Cyclotron – Air pressure


Time measurement It accelerates the charge particle like proton,
electron etc.

Q25. Consider the following statements:

(1) Acid is a compound which convert red litmus to blue.


(2) The compound which releases H+ in aqueous solution is called
acid.
(3) In neutralization reaction acid and base reacts and froms strong
acid.
(4) H2SO4 is strong acid.

Choose correct options:


(a) 1 and 2
(b) 1 and 3
(c) 2 and 4
(d) 1, 2 and 3
Ans. – (c) Acid is a substance which releases H+ ion in aqueous
solution. It’s taste is sour. Aqueous solution of acid converts blue
litmus to red. Base is a substances which releases OH– ion. Strong
acids are HCl, HNO3, H2SO4. Neutralization is a chemical process
through which salt is formed i.e.
Acid + Base → Salt
HCl + NaOH → NaCl + H2O

Organic and Inorganic Chemistry

Q1. Consider the following statements regarding Aqua regia:


(1) It is the mixture of Conc HCl and

Conc CH3COOH.
(2) It is highly corrosive, fuming liquid. (3) It can dissolve platinum.
(4) HNO3 is a strong oxidising agent. Select correct option:
(a) 1, 2 and 4 (b) 2 and 4
(c) 2, 3 and 4 (d) All of the above Ans. – (c) HNO3 is a strong
oxidising agent. It oxidises other and reduced itself. Aqua regia (Latin
for royal water) is a freshly prepared mixture of Conc HCl and Conc
HNO3 in the ratio of 3 : 1. It can dissolve gold and platinum, even
though neither of these acids can do so alone. It is highly corrosive,
fuming liquid.

Q2. Consider the following statements regarding Reactivity


series of metals: (1) Metals low in the activity series are

very reactive.
(2) Metals in the middle of the activity
series are moderately reactive. (3) Gold, silver and copper are in the
top
of activity series.
(a) Only 2 and 3 (b) 1 and 2
(c) Only 2 (d) 1, 2 and 3 Ans. – (c) The reactivity series is a list of
metals arranged in the order of their decreasing activity. Metals low in
the activity series are less reactive like Ag, Au etc. They are often
found in a free state for ex.: Ag, Au.
The metals in the middle of activity series such as iron, zinc, lead are
moderately reactive. These are usually present as sulphides or
carbonates in nature.
Q3. Consider the following statements regarding α-particle
scattering experiment:

(1) Most of the fast moving α-particles pass straight through gold foil.
(2) Some of the α-particles were deflected by the foil by small angles.
(3) Some of the α-particle deflected by 180°.
Select correct options:
(a) 1, 2 and 3
(b) Only 4
(c) 2 and 3
(d) Only 1
Ans. – (a) On the basis of α-ray scattering
experiment Rutherford put forward the
Nuclear model of an atom which had the
following features:
(i) There is positively charge centre in the atom called the Nucleus.
(ii) Electron revolve around the nucleus in well defined orbit.
(iii) The size of Nucleus is very small. Q4. Consider the following
statements:
(1) A chemical formula of a compound shows its constituents
elements and no. of Atoms of each combining element.
(2) Mass of 1 mole of a substance is called Atomic mass.
(3) Mass = Molar mass × Number of moles
(4) Number of mole

= Weight ×
1
Molecular mass

Select correct option:


(a) 1 and 4
(b) 2 and 3
(c) 3 and 4
(d) 1, 3 and 4
Ans. – (d) Chemical formula of a com

pound shows its constituents elements and no. of atom of each


associated element. Ex.: CaCO3 (1 atom Ca, 3 atoms of oxygen and
1 atom of carbon)

Mass of 1 mole of substance is called Molar Mass. No. of Mole =


=
=
= Weight

Molecular Mass Volume


Volume at STP V
224l
No. of particle
Avagadro number Q5. Match the following element with their
Atomic number and select correct code: Element Atomic number
(A) Sodium (B) Helium (C) Neon (D) Lithium Codes:

(1) 2
(2) 10 (3) 3
(4) 11

AB C D
(a) 4 1 2 3
(b) 4 2 1 3
(c) 3 4 2 1
(d) 1 2 3 4
Ans. – (a)
Element Atomic number
Sodium – 11
Helium – 2
Neon – 10
Lithium – 3
Q6. For the following statements, choose correct option:
(a) J.J. Thomson proposed that the nucleus of an atom contains only
nucleons.
(b) A neutron is formed by an electron and a proton combining
therefore, it is neutral.
(c) The mass of an electron is about 1/2000 times that of proton. (d)
An isotope of iodine is used for making tincture iodine, which is used
as a medicine.
(A) Only c (B) a and b (C) None of these (D) a, b and c Ans. – (A)
J.J. Thomson discovered the electron. Electron is negatively charge
particle. Mass of an electron is 9.1 × 10–31 kg. It is a universal
constituents of any substance. Neutron is a neutral particle, it is not
formed due to combining of electron and proton. It was discovered by
N.C. Chadwick.
Q7. Match the following List-1 and List-2: List-1 List-2 Natural
Source (A) Vinegar
(B) Nettle sting (C) Ant sting (D) Tomato
Codes:
Acid
(1) Acetic acid (2) Methanoic acid (3) Methanoic acid (4) Oxalic acid

AB C D
(a) 1 2 3 4
(b) 1 2 4 3
(c) 3 2 4 1
(d) 1 4 2 3
Ans. – (a)
Vinegar – Acetic acid (CH3COOH) Nettle sting – Methanoic acid
Tomato – Oxalic acid (C2H2O4) Lemon – Citric acid
Orange – Acetic acid
Q8. Consider the following statements and select correct option:
(1) Sodium carbonate (Na2CO3) is used in glass, soap and paper
industry. (2) Recrystallization of sodium carbonate gives washing
soda.
(3) Baking soda is also called sodium hydrogen carbonate (NaHCO3).
(4) Bleaching powder is produced by the action of chlorine on dry
slaked lime [Ca(OH)2].
(a Only 2 and 4
(b) All of the above
(c) 1, 2 and 3
(d) 1 and 4
Ans. – (b) Sodium carbonate is also called washing soda.
Recrystallization of sodium carbonate gives washing soda. (Na2CO3
+ 10H2O → Na2CO3 . 10H2O)
Baking soda is used in kitchen for making tasty cripsy Pakoras.
Sometimes it is added for faster cooking.
Chlorine gas is used for manufacturing bleaching powder.
Ca(OH)2 + Cl2 → CaOCl2 + H2O
Q9. Which of the following statements is not correct:
(a) Epoxy has outstanding bonding strength.
(b) Epoxy resins are tough, weather resistant and do not shrink.
(c) Epoxy has excellent electrical conduc tion properties.
(d) None of these.
Ans. – (c) Epoxy resins are named for epoxide groups (CyCl –
CH2OH; Cyclolic refer to the triangle form by this group) that
terminate the molecules. Because of epoxy chemical resistance and
excellent electrical insulation properties electrical parts such as relay,
coils and transformer are insulated with epoxy.
Q10. Consider the following statements and select correct code:
(1) Lithium is preserved under naptha. (2) Lithium is the least reactive
of the alkali metal, it tarnishes rapidly in air. (3) Lithium has no
naturally occurring iso topes.
Codes:
(a) 2 and 3 are correct
(b) 1 and 2 are correct
(c) 1, 2 and 3 are correct
(d) 1 and 2 are incorrect
Ans. – (d) Two naturally occurring isotopes of lithium are Li and 7Li.
They are not radioactive. Isotopes of lithium are 5Li and 8Li. lithium
can be used as tritium source for controlled fusion reactors. Its
compounds are reducing agents.
Q11. Match the following and select the correct code:
(A) Washing soda (1) NaHCO3 (B) Baking soda (2) Na2CO3 (C)
Quick lime (3) Ca(OH)2 (D) Slaked lime (4) CaO
Codes:
AB C D
(a) 1 2 3 4
(b) 4 3 2 1
(c) 2 1 4 3
(d) 3 2 4 1
Ans. – (c)
Washing soda – Na2CO3
Baking soda – NaHCO3
Quick lime – CaO
Slaked lime – CO(OH)2
Q12. Which of the following is not correct: (a) Every steel has
carbon contain. (b) Stainless steel contain 12 to 20% of chromium.
(c) Stainless steel is resistant to rusting and corrosion because of its
nickel content.
(d) None of these.
Ans. – (c) Stainless steel contain normally 70-90% of iron, 12-20% of
chromium, 0.1 to 0.7 percent of carbon. Stainless steel is resistant to
rusting and corrosion because its chromium content not nickel. Nickel
and Molybdenums are also be added to obtain improve variety of
stainless steel. Q13. Limestone used in the extraction of iron act
as which of the following:
(a) Flux
(b) Mineral
(c) Gangue
(d) Slag
Ans. – (a) Limestone is added as flux. It decompose into calcium
oxides and carbondioxides. Calcium oxides combined with sand
(SiO2) to form calcium silicate (CaSiO3) known as slag which is
tapped out of the furnace from time to time. So flux is the correct
answer about limestone added as flux.
Q14. Match the following metal with their alloys and select the
correct code: (A) Brass (1) Cu, Zn
(B) Solder (2) Pb, Sn
(C) Nichrome (3) Cu, Sn
(D) Gun metal (4) Ni, Fe, Mn, Cr Codes:
AB C D
(a) 1 2 3 4
(b) 1 2 4 3
(c) 4 2 3 1
(d) 2 4 3 1
Ans. – (b)
Brass – Cu, Zn
Solder – Pb, Sn
Nichrome – Ni, Fe, Mn, Cr
Gun metal – Cu, Sn
Q15. What will happen if volume of gas is decreased at constant
temperature then pressure will be:

(a) Decreases
(b) Increases
(c) Remains constant
(d) None of these
Ans. – (b) According to Boyle’s law, at constant temperature and
fixed amount of gas, volume is inversely proportional to the pressure,
i.e. pressure increases, volume decreases and vice versa.
From Ideal gas equation
PV = nRT
P = Pressure, V = Volume
n = No. of mole of the gas
R = gas constant
T = temperature

i.e.
P

1
V

Q16. Given below are three important air pollutants, match them
with instrument used for their detection and select correct
answer/code: (A) Flame
photometer (B) Infrared
spectrometer (C) Photo detector Codes:
AB C (a) 1 2 3 (b) 3 1 2 (c) 1 3 2 (d) 2 3 1 Ans. – (b)
Flame photometer (1) Carbon monoxides

(2) Nitrogen oxide


(3) Sulphur dioxide

– Sulphur dioxide Infrared spectrometer – Carbon monoxide Photo


detector – Sulphur dioxide

Q17. Consider the following statements: (A) Equivalent weight =

Atomic weight × Valency


(B) Atomic weight =
Equivalent weight × Valency (C) Valency =
Equivalent weight × Atomic weight (D) Atomic weight =
Equivalent weight × Valency Ans. – (b)

Equivalent weight =
Atomic weight

Valency Atomic weight = No. of Protons + No. of Neutrons


Valency – It is the required no. of electron in outermost orbit for
completing octet and no. of electrons present in outermost orbit is
called as valence electron.

Q18. Match the following List-1 and List-2: List-1 List-2


(A) Simple salt (1) Alum
(B) Acidic salt (2) Barium chloride (C) Basic salt (3) Sodium
bicarbonate (D) Double salt (4) Ferric hydroxide Codes:

AB C D (a) 3241
(b) 2341
(c) 1234
(d) 2314
Ans. – (b) Simple salt Acidic salt Basic salt Double salt
– Barium chloride
– Sodium bicarbonate
– Ferric hydroxide
– Alum
Q19. Consider the following statements: (1) Soap and detergents
are highly fatty acids like oleic acid.
(2) Long chain of hydrocarbon is present in detergent.
(3) Soft soap contains highly fatty acidic potassium salt.
(4) Hard soap contains highly fatty acidic sodium salt.
Choose the correct answers:

(a) Only 2 and 4


(b) 1 and 2
(c) All of the above
(d) 1, 2 and 3
Ans. – (c) Soap and detergents are the mixture of highly fatty acid
like oleic acid and potassium salt. Soft soap contains highly fatty
acidic salt. Potassium, which is used for bathing but soap which
contains sodium salt is used for washing clothes.

Q20. Match the following metals with the colours they emit and
select correct answer:
List-1 List-2
(A) Sodium (1) Yellow

(B) Aluminium (2) White


(C) Copper (3) Blue
(D) Barium (4) Green
Codes:

AB C D
(a) 4 2 3 1
(b) 1 2 3 4
(c) 3 2 4 1
(d) 4 3 2 1
Ans. – (b)
Sodium – Yellow
Aluminium – White
Copper – Blue
Barium – Green
Q21. Consider the following statements:

(1) Diamond is a bad conductor of electricity and heat.

(2) In Diamond carbon is SP3 hybridized and in graphite carbon is


SP2 hybridized.

(3) Graphite is a bad conductor of electricity.


(4) Refractive index of diamond is 2.417.
Choose the correct options:
(a) 1 and 2 (b) 1, 2 and 4
(c) Only 4 (d) None of these Ans. – (b) Carbon is a nonmetal. It’s
atomic no. is 6. It shows Allotropic property. It has two allotropes (1)
Diamond (ii) Graphite.
Diamond is the hardest material in the world. It has tetrahedra
structure. It’s crystal is cubical type. It’s refractive index is 2.417.
Graphite is a good conductor of electricity. It has large no. of free
electrons to conduct electricity.
Q22. Match the following List-1 and List-2: Chemical substance
(A) Nitrous oxide (N2O)
(B) Zinc chloride (ZnCl2)
(C) Magnessium carbonate (MgCO3) (D) Heavy water (D2O)
Uses
(1) Nuclear Reactor
(2) Tooth Paste
(3) Textile Industry
(4) Surgery
Codes:
AB C D
(a) 4 3 2 1
(b) 1 2 3 4
(c) 1 2 4 3
(d) 4 3 1 2
Ans. – (a)
Nitrous oxide – Surgery
Zinc chloride – Textile Industry
Magnessium carbonate – Tooth Paste
Heavy water (D2O) – Nuclear Reactor Q23. Consider the following
statements:
(1) Cesium is used in solar cell.
(2) Silver iodide is used for artificial rain.
(3) Polonium has highest number of iso topes.
(4) Nitrogen can be used in place of inert gas.
Choose the correct options:
(a) Only 2 and 3 (b) 1 and 4
(c) All of the above (d) 1, 2 and 3
Ans. – (c) Solar cell is used for lighting
purpose. Cesium is used in solar cell.
Silver iodide is used for artificial rain.
Polonium (PO) has highest number of isotopes. It has 27 isotopes.
Isotopes have
same number but different atomic mass.
Nitrogen can be used in place of inert gas. Q24. Match the following
List-1 and List-2:
List-1 List-2
(A) Bauxite (1) Uranium
(B) Hematite (2) Aluminium
(C) Machelite (3) Iron
(D) Pich Blend (4) Copper
Codes:
AB C D
(a) 1 2
(b) 2 3
(c) 2 3
(d) 3 2
Ans. – (b)
Bauxite
Hematite
Machelite
Pich Blend 3 4 4 1 1 4 4 1

– Aluminium
– Iron
– Copper
– Uranium

Q25. Consider the following statements:

(1) The extraction of metal from their ores is called calcination.


(2) Sodium, Potassium and Calcium are most reactive elements.
(3) Gold is more reactive than silver.
(4) Aqua regia can dissolve gold.
Choose correct options:
(a) 2 and 4
(b) 1 and 4
(c) None of these
(d) All of the above
Ans. – (a) The extraction of metal from their ores is called Metallurgy.
The reactivity series is a list of metals arranged in the order of their
decreasing activities.
Activity Series

K > Na > Ca > Mg > Al > Zn > Fe > Pb > H > Cu > Hg > Ag > Au
Aqua regia (Royal water) is a freshly prepared mixture of
concentrated HCl and the HNO3 in the ratio of 3 : 1. It can dissolve
gold, even though neither of these acid can do so alone.

Acid, Base and Salt

Q1. Which one of the following statements regarding producer


gas is incorrect: (a) It is used in the manufacturing of coal

gas and operates gas engine’s. (b) It is a mixture of Carbon


monoxide
and Nitrogen.
(c) The reaction for the production of gas
is endothermic.
(d) All of the above.
Ans. –(c) The producer gas is made by blowing air over a red hot
coke. Some of the coke burns to carbon monoxides (CO) at the
bottom of the furnace. This gets reduced to CO as it passes through
the upper layer of red hot coke. The mixture of Nitrogen and Carbon
monoxides come out from top of the furnace. This reaction is
exothermic not endothermic. Heat produced maintain the temperature
of the coke.
Q2. When fractional distillation of liquid petroleum is done,
which one of the following is the first fraction to escape: (a)
Diesel oil (b) Kerosene
(c) Naptha (d) Petroleum ether Ans. – (d) Petroleum ether is used as
a solvent. On gradually, raising of the temperature the following ether
fractions are obtained one by one. (1) Naptha, which on further
distillation yield petrol, (2) Kerosene, (3) Fuel oil (Diesel oil, (4)
Lubricating oil different viscosity, (5) Petroleum jelly, (6) Paraffin wax,
(7) Asphalt.
Q3. Which one of the following manufacturing process is
incorrectly matched: (a) Haber’s process – Ammonia (b) Bosch
process – Hydrogen (c) Nelson process – Hydrogen chloride (d)
Ostwald process – Nitric acid Ans. – (c) Commercially, chlorine is
either obtained by the electrolysis of concentrated hydrochloric acid
or of brine i.e. that by Nelson process salt solution used. In this
process, besides chlorine hydrogen and sodium hydroxides are also
obtained. So option (c) is not correctly matched. Q4. Which one of
the following statements regarding Ozone is untrue:
(a) It can be liquefied to a blue liquid. (b) It is only an oxidizing agent.
(c) It is used for sterilizing drinking
water.
(d) None of these.
Ans. – (b) For some cases ozone act as a reducing agents also. It
reduce hydrogen peroxides to water.
H2O2 + O3 = ZO2 + H2O
Ozone can be liquefied to a blue liquid, boiling at about 112°C under
the ordinary pressure. It destroys the mobility of mercury and mercury
is said to be field. Q5. Consider the following statements related
to silver plating a brass spoon and select the correct code given
below: (1) The spoon is made the anode and
cathode is a plate of pure clean silver. (2) It is suspended in a solution
of silver
nitrate.
(3) Before the plating surface/process the
spoon is washed by caustic soda and
dellite HCA (hydrochloric acid). Code:
(a) All 1, 2 and 3 are correct
(b) Only 3 is correct
(c) 1 and 3 are correct
(d) 1 and 2 are correct
Ans. – (b) The spoon is suspended in the bath of sodium silver
cyanide, made by adding sodium cyanide to a solution of silver nitrate
until perceived form just dissolved. If a silver nitrate solution is used
directly instead of sodium and silver, the deposition of silver will be
very quickly and hence not very uniform and smooth. Q6. In the
given below are the result of burning certain substances with
oxygen (O2). Which of the following is not correctly matched:
(a) Phosphorus – White fumes (b) Sodium – Green flame (c) Wood
charcoal – Yellow spark (d) Sulphur – Bright blue flame Ans. – (b)
When phosphorus, wood charcoal and sulphur burn with oxygen then
they gives white fumes, yellow spark and bright blue flame
respectively. But sodium when burn with oxygen (O2) then it gives
golden yellow flame not green flame. So except option (b) all are
correctly matched. Q7. Which of the following elements literally
means ‘acid former’:
(a) Chlorine (b) Methane
(c) Hexane (d) Oxygen
Ans. – (d) Oxygen was first discovered by Sir Joseph Priest when he
heated some mercury (II) oxides in test tube by focusing sun’s rays
with a sort focus convex lens and got a gas, which rekindled a
glowing split. He knews it dephlogisticated air. He communicated his
discovery to the Lavoiser who repeated priestly experiments and
showed that the gas was an element. He named it oxygen meaning
acid former because through that oxygen was constituents of all
acids.
Q8. Coal gas is a mixture of several gases. Which of the
following constitutes the largest bulk of its composition:
(a) Carbon dioxide (CO2)
(b) Hydrogen (H2)
(c) Methane (CH4)
(d) Carbon monoxides (CU)
Ans. – (b) The coal gas is manufactured by the distillation of coal in
iron retorts, which are heated externally by gaseous fuels, as
like/such as producer gas. Coal gas is a mixture of (Methane – 25 to
30%, Carbon monoxides 1 to 10%, Hydrogen 45 to 55%, Ethylene,
CO2 and Ether gases – 5 to 8%) to create an inert atmosphere in a
certain chemical process.
Q9. Which one of the following is the instruments than can
determine the relative amount of various isotopes in a sample of
the substances being analyzed: (a) Mass spectrometer
(b) Pyrometer
(c) Bolometer
(d) All of the above
Ans. – (a) To determine the relative amount of various isotopes in the
sample of the substance being analyzed by mass spectrometer and
scintillation counter is an instrument that can be used to confirm
whether drinking water contain a gamma emitting isotopes or not.
Q10. Consider the following statements and select the correct
code:
(1) Isobar have different atomic number. (2) Isobar are atom of the
same elements. (3) Isobar differ in electronic
configurtion and chemical properties. Code:
(a) Only 1 is correct
(b) 1, 2 and 3 are correct
(c) Only 3 is correct
(d) 1 and 3 are correct
Ans. – (d) The atoms of different element, which have same atomic
number but different atomic weight is called isobars and isobars are
differ in electronic configuration and also chemical properties of the
elements.
Q11. Match the following list-1 with list-2 and select correct
code:
List-1 List-2
(1) White Vitroil (A) K3Fe(CN)6 (2) Red Vitroil (B) ZnSO4 . 7H2O (3)
Blue Vitroil (C) CuSO4 . 5H2O (4) Green Vitroil (D) FeSO4 . 7H2O
Codes:
AB C D
(a) 1 2 3 4
(b) 2 1 3 4
(c) 2 3 4 1
(d) 1 4 3 2
Ans. – (b)
White Vitroil – ZnSO4 . 7H2O Red Vitroil – K3Fe(CN)6
Blue Vitroil – CuSO4 . 5H2O Green Vitroil – FeSO4 . 7H2O Q12.
Match the following list-1 with list-2 and select the correct code:
List-1 List-2
(A) Brimstone (1) Sulphur (S) (B) Jeweller’s rouge (2) CH4 (C) Fire
damp (3) Fe2O3 (D) Led stone (4) Fe3O4 Codes:
AB C D
(a) 1 2 3 4
(b) 1 3 2 4
(c) 2 3 1 4
(d) 1 4 3 2
Ans. – (b)
Brim stone – Sulphur (S) Jeweller’s rouge – Fe2O3
Fire damp – CH4
Led stone – Fe3O4
Q13. Isotopes of the same elements have identical electronic
configuration and same chemical properties. They differ from
each other due to the difference in the number of which one of
the following: (a) Electron
(b) Neutrons
(c) Proton and Neutron
(d) None of these
Ans. – (d) The atoms of the same elements, which have same
atomic number but differ. Atomic weight is called isotopes. The
differences in atomic weight because of the different no. of proton in
their nuclei. So none of these option is correct.
Q14. Which one of the given following option is an example of
thioacids:
(a) HBr (b) HCl
(c) H2SO4 (d) H2S2O3
Ans. – (d) H2S2O3 is an example of thioacids. Thioacids are those
oxyacids which one or more oxygen (2) atom are replaced by sulphur
(S) atom. In the given example H2SO4 is the oxyacids and H2S2O3 is
the Thiosulfuric acid or Thioacids. Q15. Match the list-1 with list-2
and select the correct code:
List-1
(1) Rancid butter (2) Eyewash
(3) Sem milk
(4) Vinegar
(5) Grape

List-2
(A) Boric acid (B) Lactic acid (C) Tartaric acid (D) Acetic acid (E)
Bytric acid

Codes: AB C D E
(a) 1234 5
(b) 2354 1
(c) 3214 5
(d) 1453 2
Ans. – (b)
Boric acid –
Lactic acid –
Tartaric acid –
Vinegar –
Bytric acid – Eyewash
Sem milk
Grape
Acetic acid Rancid butter Q16. Consider the following statements
and select the correct code/answer. Which of the following is/are
an amphoteric oxides:

(1) SO2
(2) PbO
(3) CO2
(4) K2O
Code:
(a) 1 and 2
(b) 2 and 3
(c) Only K2O (4)
(d) 1, 2, 3 and 4
Ans. – (c) The amphotric oxides are those oxides, which react with
acid as well as bases to form salt and water or neutralization reaction.
Oxides of zinc, lead and aluminium are amphoteric.
Q17. Which of the following is an example of non-electrolyte:
(a) Ether
(b) Hydrochloric acid
(c) Nitric acid
(d) Acetic acid
Ans. – (a) Ether, benzene, chloroform, alcohol or some oil or an
aques solution of cane sugar or glucose do not allow current to
passing through them such substances as know as non-electrolytes.
Non-electrolytes stay infact. So ether is a non-electrolytes example of
given question.
Q18. Match the following list-1 with list-2 and select the correct
when given element burn with oxygen then gives answer/code:
List-1
(A) Sulphur

(B) Wood charcoal (C) Phosphorus (D) Sodium

List-2
(1) Golden yellow flame

(2) Bright blue flame (3) Yellow sparks (4) White fumes

Codes: AB C D
(a) 1234 (b) 2 3 (c) 2 3 (d) 3 2 Ans. – (c)
Sulphur – Wood charcoal – Phosphorus – Sodium – 4 1 4 1 4 1

Bright blue flame Yellow sparks


White fumes
Golden yellow flame

Q19. Consider the following statements and select the correct


code:
(1) pH of milk is slightly alkaline. (2) pH of human blood is slightly
acidic. (3) pH6 means the solution contain one

part by weight of H+ ions in 1000,000 parts.

Code:
(a) All 1, 2 and 3 are correct
(b) Only 2 is correct
(c) Only 3 is correct
(d) None of these
Ans. – (c) The pH value of human body is 7.3 so it is slightly alkaline.
Milk has pH value is 6.6 is slightly acidic and pure water has pH value
7, so water is neutral not acidic and not basidity.

Q20. Match the list-1 with list-2 and select the correct code:
(1) Bosch process (2) Nelson process (3) Ostwald process (4)
Haber’s process (A) Ammonia (B) Nitric acid (C) Salt solution (D)
Hydrogen

Codes: AB C D
(a) 1 2 3 4
(b) 4 3 2 1
(c) 3 2 4 1
(d) 1 4 3 2
Ans. – (b)
Bosch process – Hydrogen Nelson process – Ostwald process –
Haber’s process – Salt solution Nitric acid Ammonia
Q21. Given below are two lists of solute and solvents
respectively. Match list-1 with list-2 and select correct code:
Solute Solvents
(A) Nitre (1) Alcohol (B) Phosphorus (2) Water
(C) Wax (3) Carbon disulfide (D) Chlorophyll (4) Turpentine oil
Codes:
AB C D
(a) 2 3 4 1
(b) 1 2 3 4
(c) 3 2 1 4
(d) 4 3 2 1
Ans. – (a)
Nitre – Water
Phosphorus – Carbon disulfide
Wax – Turpentine oil
Chlorophyll – Alcohol
Q22. The atoms of the same elements, which
have same atomic weight but different
atomic number is known as:
(a) Isotopes
(b) Isobars
(c) Both Isotopes and Isobars
(d) None of these
Ans. – (b) The Atoms of the same elements
which have same atomic weight but different atomic number is called
isobars and
which elements have same atomic number
but different atomic weight/mass is known
as Isotopes. So option (b) is the answer of
the above question.
Q23. Which one of the following elements
used to galvanizing iron:
(a) Chromium (b) Magnesium
(c) Zinc (d) Nickel
Ans. – (c) Zn (Zinc) is used to coat iron sheets to prevents them from
rusting. This process of coating iron sheet with molten zinc is known
as galvanizing. Galvanized iron sheet are used for roofing and for
making tubes, buckets etc. Zinc also used for plating other elements.
Q24. Which one of the following is correct descending order of
elements with respect to their average percentage by mass in
human body:
(a) Oxygen, carbon, calcium, hydrogen, nitrogen.
(b) Oxygen, carbon, hydrogen, nitrogen, calcium.
(c) Carbon, oxygen, calcium, nitrogen, hydrogen.
(d) None of these.
Ans. – (b) The composition of the human body varies slightly from
person to person but the average percentage by mass of the element
in the human body is oxygen (65%), carbon (18%), hydrogen (10%),
nitrogen (3%) and calcium (2%), phosphorus (1%), potassium
(0.35%), sulphur (0.25%), sodium (0.15%), chlorine (0.15%),
magnesium (0.05%) and iron (0.004%).
Q25. Petrol is the mixture of three hydrocarbons. Which one of
the following correct set of its ingredients:
(a) Hexane, Heptane and Octane. (b) Propane, Heptane and
Pentane. (c) Butane, Hexane and Isobutane. (d) None of these.
Ans. – (a) In petroleum ether is used as a solvent. On gradually
raising, the temperature the following other fraction are obtained one
by one (1) Naptha, which one further distillation yield petrol, (2)
Kerosene, (3) Fuel oil (Diesel oil), (4) Lubricating oil different
viscosity, (5) petroleum jelly, (6) paraffin wax, (7) Asphalt.

Carbon and Other Substances

Q1. In the given following some alloy and their constituents and
match the list-1 with list-2:
(A) Bismath, Lead, Tin (1) Dutch metal (B) Copper, Zinc, Tin (C)
Copper and Zinc (D) Copper and Tin (2) Rose metal (3) Gun metal
(4) Bell metal

Codes: AB C D
(a) 1234
(b) 2314
(c) 3241
(d) 1342
Ans. – (b)
Bismath, tin, lead
Copper, zinc, tin
Copper, zinc
Copper, tin
– Rose metal
– Gun metal
– Dutch metal
– Bell metal Q2. Which one of the following is not an allotropes of
carbon:
(a) Graphines (b) Graphites (c) Diamond (d) Fullerines Ans. – (a) The
graphine is basic structural elements of fullerines. There are three
allotropes of carbon – diamond, graphite and fullerines. Fullerines is
the family of newly discovered, the allotropic form of carbon. The first
member of this form of carbon is also called fellerines. It was
discovered in 1985 by a team of scientist Harled Kroto, Richard
Smalley and Robert Curl. All three are awarded by Noble Prize. Q3.
Consider the following statements and select the correct
answer/code:
(1) Oxygen is more soluble in water in comparison to Nitrogen.
(2) Every time we drink water, we drink a little bit of glass too.
(3) Pure water does not exist in natural states.
Code:
(a) 1, 2 and 3 are correct
(b) Only 3 is correct
(c) Only 1 is correct
(d) All are incorrect
Ans. – (a)
Water is known as universal solvent. Every time we drink water, we
drink a little bit of glass too. But quantity is negligible. Because of
solvent power, pure water does not exist in natural states. Even rain
water, the purest form of natural water, contain 0.005 of solid
impurities. So all of the given option is correct about water. Q4.
Consider the following statements and select the correct
code/answer:
(1) To remove impurities of water which collected from river or lake, it
is mixed with required quantities of Alum (Al) soda and limestone.
(2) Dissolved impurities in small quantities are not objectivable to
drinking. Code:
(a) 1 and 2 are correct
(b) Only 2 is correct
(c) Only 1 is correct
(d) Neither 1 nor 2 are correct
Ans. – (a) Water collected from river of lake is treated with required
quantities limestone, soda, aluminium (Al) which react with one
another to form aluminium hydroxide, a Jelly like a stick solid. The
dissolved impurities, unless they are present in large quantities are
not objectionable for drinking. So all of the option is correct. Q5.
Which one of the following statements is untrue about Diamond:
(a) It alloy light and x-ray to passing through it.
(b) It is a very good conductor of heat and electricity.
(c) It has high refractive index.
(d) None of these.
Ans. – (b) The Diamond is a very bad conductor of electricity. In a
crystal like diamond, there is no unit, which can be called a
molecules. It has high refractive index of about 2.4. It does not melt
even at a very high temperature. It is an allotropes of carbon. It is
very solid substance of the world. Q6. Which of the following
statements is incorrect:
(a) Carbogen is a mixture of 60% oxygen and 40% carbon dioxides.
(b) Carbon monoxide prevents red cell from absorbing oxygen.
(c) Carboxy haemoglobin is a more stable than Oxyhemoglobin.
(d) None of the above.
Ans. – (a) If a object/person inhaled carbon monoxides and his
condition deteriorities, life can be saved by blood transfusion, by
injecting methylene blue or by making the person inhale carbogen.
Carbon is a mixture of 95% oxygen (O2) and 5% carbon dioxides
(CO2).
Q7. Which one of the following statements is incorrect about
graphite:
(a) It is a good conductor of electricity. (b) The crystal lattice of
graphite is pentagonal.
(c) In a soft pencil, the percentage of clay is low.
(d) None of these.
Ans. – (b) The crystal lattice of graphite is hexagonal not pentagonal.
Graphite is a conductor of electricity and it is used to making
electrodes and lining electric furnace. In soft pencil, the percentage of
clay is low and percentage of graphite is high. In hard pencil the
graphite low and clay high.
Q8. Which one of the following statements regarding hydrogen
peroxide is untrue: (a) It is used for washing wounds and also used
in mouthwash.
(b) It is used for bleaching silk, wool, leather, paper etc. and bleaches
hair to a golden colour.
(c) It is a stable compound and it does not readily get decomposed.
(d) All of the above are untrue.
Ans. – (c) In case of hydrogen peroxides, it is very unstable and
decomposed readily into oxygen and water. It is this oxygen, which
gives hydrogen oxides its oxidizing, germicidal and bleaching
property. Its chemical reaction is
2H2O2 = 2H2O + O2
Q9. Which one of the following is responsible/reasonable for
temporary hardness of water (H2O):
(a) Sulphate of calcium and magnesium. (b) Chloride of calcium and
magnesium. (c) Hydrogen carbonate of calcium and magnesium.
(d) All of the above.
Ans. – (c) Water containing only sulphate and chloride of Mg and Ca
is known as permanent hard water. Water containing only hydrogen
carbonates of magnesium and calcium is known as temporary
hardness of water. The temporary hardness of water is/can be
removed if we boil/heat water.
Q10. Consider the following statements and select the correct
code/answer:
(1) The vapour pressure of efflorescent substances is higher than the
partial pressure of vapour in dry.
(2) The vapour pressure of deliquescent substances is lower than the
vapour pressure of water in moist air. Code:
(a) 1 and 2 are correct
(b) Only 2 is correct
(c) Only 1 is correct
(d) Neither one (1) nor two (2) are correct Ans. – (a) The
deliquescent substances are hygroscopic substances, which absorbs
so much water (H2O) that they loose their form and ultimately get
dissolved in the water and so absorbed. Deliquescent substances like
magnesium chloride, ferric chloride, solid NOH and KOH etc.
Washing soda (Na2CO3 . 10 H2O) and glauber salt (Na2SO4 . 10H2O)
are some example of efflorescent substances. Q11. Match the
following list-1 with list-2 and select the correct code:
List-1 List-2 (A) Aircraft manufacture (1) Graphite (B) Explosive (2)
HNO3 (C) Steel manufacture (3) Oxygen (D) Paints (4) ZnO
Codes:
AB C D
(a) 1342
(b) 2314
(c) 4321
(d) 1234
Ans. – (d)
Aircraft manufacture – Graphite
Explosive – HNO3
Steel manufacture – Oxygen
Paints – ZnO Q12. Match the following list-1 with list-2 and
select correct code:
list-1
(A) Graphite and Diamond
(B) C2H5OH and CH3OCH3
(C)20Ca40, 10K40
(D)8O16, 8O13
list-2
(1) Allotropes
(2) Isomers
(3) Isobars
(4) Isotopes
Codes:
AB C D
(a) 1 2 3 4
(b) 4 3 2 1
(c) 3 4 2 1
(d) 2 4 3 1
Ans. – (a)
Graphite and Diamond – Allotropes
C2H5OH and CH3OCH3 – Isomers
40 40
20Ca , 10K – Isobars
8O16, 8O13 – Isotopes Q13. Match the following list-1 with list-2 and
select correct code:
List-1
(A) Na2S2O3
(B) Liquid NH3
(C) Graphite
(D) Rocket fuel
List-2
(1) Photographic processing
(2) Refrigerants
(3) Conductor of electricity
(4) Liquid oxygen
Codes:
AB C D
(a) 3
(b) 1
(c) 1
(d) 3
Ans. – (c)
Na2S2O3
Liquid NH3
Graphite
Rocket fuel 4 2 1 4 3 2 2 3 4 4 1 2

– Photographic processing
– Refrigerants
– Conductor of electricity
– Liquid oxygen

Q14. Match the following list-1 with list-2 and select the correct
code:
List-1 List-2
(A) Fertilizer (1) Ni
(B) Catalyst (2) H2O2 (C) Solvent (3) Carbon (c) (D) Reducing agent
(4) (NH4)2SO2 (E) Bleaching agent (5) CHCl3 Codes:

AB C D E (a) 4 1 5 3 2
(b) 5 4 3 2 1
(c) 1 2 3 4 5
(d) 1 4 3 2 5
Ans. – (a)
Fertilizer – (NH4)2SO2
Catalyst – Ni
Solvent – CHCl3
Reducing agent – Carbon
Bleaching agent – H2O2
Q15. Match the following list-1 with list-2 and select the correct
code:
List-1 List-2
(Substances) (Function)
(1) C2H4 (A) Sedative
(2) KBr (B) Anaesthetic (3) SO2 (C) Refrigerants (4) Mg(OH)2 (D)
Purgative
Codes:
AB C D
(a) 1 2 3 4
(b) 2 3 1 4
(c) 1 3 4 2
(d) 3 2 4 1
Ans. – (b)
C2H4 – Anaesthetic
KBr – Sedative
SO2 – Refrigerants
Mg(OH)2 – Purgative
Q16. Consider the following statements and select the correct
answer/code:
(1) Phosphorus, potassium and carbon exhibit allotropy.
(2) The differences in the physical property of allotropes of an
element due to the differences in their atomic arrangements.
Code:
(a) 1 and 2 are correct
(b) Only 1 is correct
(c) Only 2 is correct
(d) Both 1 and 2 are incorrect
Ans. – (c) When an element exists in two or more form without
changing its states and possesses differences physical properties,
this phenomenon is known as allotropism or allotropy. Carbon,
sulphur and phosphorus exhibit by allotropy. The differences in their
properties perhaps due to differences in their electronic/atomic
arrangements.
Q17. Which of the following is used as a catalyst in the
hydrogenation of the vegetable oils:
(a) Platinum (b) Silver
(c) Nickel (d) All of the above Ans. – (c) For the hydrogenation of
vegetable oil which are liquid under ordinary temperature and limited
used in our diet the oil is first thoroughly purified and made free from
smelling components. Then it heated to about 200°C and hydrogen is
passed at 5 atmospheric pressure in the presence of finely divide
nickel which acts as a catalyst. So option (c) is correct. Q18. Which
one of the following is called/known as a treacherous gas: (a)
Chlorine (b) Methane (c) Carbon monoxide (d) Nitrogen Ans. – (c)
Carbonmonoxides (CO) is treacherous gas, being colourless and
odorless it does not give any notice of its presence. A person first
feels drowsy on inhaling a little bit of gas. He is unable to shout for
help and goes on inhaling more and more of the gas until it is too late.
Carbon monoxides and nitrogen are a producer gas.

Q19. Which one of the following statements regarding carbon


dioxides is incorrect: (a) Soft drink are charged with

carbondioxides under pressure. (b) If a piece of burning magnesium


is
introduced in a jar of CO2 it is
extinguished.
(c) Solid carbon dioxide is called a dry
solid ice because when it changes into
gas liquid is left behind.
(d) None of these.
Ans. – (b) A piece of burning magnesium ribbon is introduced in a jar
of CO2, it continues to burn with cracking sound. This is not happen
because CO2 is a supporter of combustion, but the temperature of
burning is so high that it splits the carbon dioxides into carbon and
oxygen and it is the O2 supporter of the combustion.
Q20. Consider the following statements and select the correct
code:
(1) Solid carbon dioxides provided an
atmosphere of gas which slow up
bacteria growth.
(2) Solid carbon dioxides lasts much
longer than ordinary ice.
(3) It freezes food safely and rapidly. Code:
(a) All 1, 2 and 3 are correct
(b) Only 2 is correct
(c) 2 and 3 are correct
(d) 1 and 2 are correct
Ans. – (a) Solid state of CO2 is used as a refrigerants has many
advantages, which are states in the statement mentioned. It is,
therefore, extensively used for shipping perishable fruits, vegetables
and meats over the long distances.
Q21. Which one of the following statements is correct about
atmolysis:
(a) This is a condition when the flow of
fluid from region of higher concentration to region of lower
concentration ceases as both reason/region now
have an equal concentration. (b) It is a principle according to which
the
process of diffusion is used for the
separation of two gases when their
rates of diffusion are different. (c) Both a and b.
(d) None of these.
Ans. – (b) Atmolysis is a principle according to which the process of
diffusion is used for the separation of two gases when their rates of
diffusion are different. This principle is employed in the large scale
separation of the isotopes of U (Uranium). Q22. Consider the
following statements and select the correct code:
(1) Carbogens is a mixture of 60%
oxygen and 40% carbon dioxides. (2) Carbogens has stimulating
effect on
breathing and is used in case of
shocks, pneumonia, gas poisoning,
drowning etc.
Code:
(a) Only 1 is correct
(b) Only 2 is correct
(c) Both 1 and 2 are correct
(d) Both 1 and 2 are incorrect
Ans. – (b) Carbogen has stimulating effect on breathing and is used
care of shocks, gas poisoning, pneumonia, drowning etc. Carbogen is
a mixture of 95% oxygen and 5% carbon dioxides not 60% and 40%
respectively.
Q23. Match the following list-1 with list-2 and select correct
code:
List-1
(A) Fire Extinguisher
(B) Fertilizer
(C) Fungicide
(D) Amphoteric
List-2
(1) Blue Vitroil
(2) Carbon dioxides
(3) Ammonium hydroxides (4) Ammonium sulphate Codes:
AB C D
(a) 1 2 3 4
(b) 4 3 1 2
(c) 1 4 3 2
(d) 4 2 3 1
Ans. – (c)
Fire extinguisher – Blue vitroil
Fertilizer – Ammonium sulphate Amphoteric – Carbon dioxides
Fungicide – Ammonium hydroxides Q24. Which one of the
following statements is related to water is incorrect:
(a) On Mount Everest, water boil at 70°C. (b) At 4°C the density of
water is 1g/cm3. (c) It has low specific heat.
(d) All of the above.
Ans. – (c) The specific heat of water is highest i.e. water requires
more heat to raise the temperature by 1°C than an equal mass of any
other liquid or solid. At 4°C water has maximum density of 1g/cm3.
The boiling point of water much effected by pressure. If pressure is
greater then boiling point highest on hills water boil at temperature
lower than 100°C and then food does not cook properly.
Q25. Consider the following statements and select the correct
answer/code:
(a) Detergent do not form/lather with
hard/tuff water.
(b) When soap combines/mixed with
hard water it form soap curd, which is
magnesium or calcium steatrate. Code:
(a) Only 1 is correct
(b) Only 2 is correct
(c) 1 and 2 are correct
(d) Neither 1 nor 2 are correct
Ans. – (b) If water is hard, the magnesium and calcium ions of the
water combine with negative steatrate ions of soap to form a slimy
precipitate of insoluble Ca and Mg or even iron soap, usually given
name of the soap curd.
2 NaH (Soap) + Ca(HCO3)2 = Ca (St)2 (Soapcurd) + 2NaH . CO3

Laws and Applications

Q1. Consider the following statements and select the correct


code:
(1) The process in which the migration of

coloidal particle takes place under the influence of electrical field


know as electrophoresis.

(2) A colloidal solution which has solid as the dispersed phase and
liquid as the dispersion medium is known as gel.

(3) ‘Tyndall effect’ is the scattering of visible light through the


dispersing medium due to large side of colloidal particle.

Code:
(a) 1, 2, 3 are correct
(b) Only 3 is correct
(c) 1 and 2 are correct
(d) 1 and 3 are correct
Ans. – (d) In a colloidal system which has liquid as the dispresed
phase and solid as the dispersion medium is called gel. Mixture in
dispresed phase is distributed throughout a dispersing medium or a
continuous phase is called colloid. Electrophorus helps in determining
the sign of the electric charge in the colloidal particle.

Q2. Consider the following statements and select correct code:


(1) Van der wal’s force is strong in

comparison to chemical bond. (2) Van der wal’s force exist in


compounds which does not contain
hydrogen atoms which is not
positively charged.
Code:
(a) Only 2 is correct
(b) Only 1 is correct
(c) Both 1 and 2 are correct
(d) Both 1 and 2 are incorrect
Ans. – (a) Van der wal’s force is a weak attractive forces between
atom or molecules. These forces is much weaker than chemical
bond. If a compound does not contain hydrogen or it contain
hydrogen atom, which is not partially positive charged, hydrogen
bond is not formed. Such a compound exist in a form of liquid state or
solid state due to presence of van der wal forces attraction between a
molecules.
Q3. Which one of the following is not take part/involved in
reduction:
(a) Decreasing in the proportion of an
electronegative element or group in a
compound.
(b) Removal of oxygen from compound. (c) Combination of an
element with
electropositive element.
(d) None of these.
Ans. – (d) Option (d) is correct. Reduction is a chemical process
which involve combination of an element with hydrogen or with any
electropositive particle/element. Removal of oxygen or any other
electronegative element from compound. Decrease the valency of
atom in a compound. Decrease the oxidation number of an element.
Q4. Which one of the following is not involved in oxidation:
(a) Removal of oxygen from a
compound.
(b) Increase the oxidation number of an
element.
(c) Loose of electron.
(d) None of these.
Ans. – (a) Oxidation is a chemical process which involves (i) removal
of hydrogen or any other electropositive element from a compound,
(ii) combination of an element with oxygen or other electronegative
particle, (iii) loose of electron, (iv) increase in proportion of an
electronegative element or group and decrease in the proportion of
electropositive element in a compound. Q5. Consider the following
statements related to radioactive and select the correct code:
(1) Radium is most powerful radioactive element.
(2) Element having atomic number greater than 83 exhibit
radioactive. Code:
(a) Only 1 is correct
(b) Only 2 is correct
(c) Both 1 and 2 are correct
(d) Neither 1 nor 2 are correct
Ans. – (b) Radium is the powerful radioactive element but
polonomium is the most powerful element of radioactivity. Gamma
rays are electromagnetic radiation of higher energy than alpha and β-
rays. It is composed of photon of high energy. Q6. Consider the
following statements and select correct code:
(1) The rate of distintegration of a radioactive element increases with
increase in time.
(2) The rate of distintegration of a radioactive element at given instant
of time is inversely proportional to the number of atoms of the
radioactive element present in it.
(a) 1 and 2 are correct
(b) Only 1 is correct
(c) Only 2 is correct
(d) Both are incorrect
Ans. – (d) The rate of distintegration of a radioactive element
decreases with increase in time. The rate of distingeration of
radioactive element at a given instant of time is proportional to the
number of atoms of the radioactive element present at the moment.

Q7. Consider the following statements and select the correct


code:
(1) Thorium dioxides used in gas

montales.
(2) Thorium occurs in monazites sand in
India.
(3) The most stable isotopes of thorium
(232Th) has the longest half-life period. Code:
(a) Only 2 and 3 are correct
(b) 1, 2 and 3 are correct
(c) 3 and 1 are correct
(d) None of these are correct
Ans. – (b) Thorium is used in gas mantales and in special heat
resisting material. It has 12 Isotopes, which have mass number from
223 to 234. The most stable Isotopes is (232Th) which has half period
of 1.39 × 1011 yrs. the longest. As it captured slow neutron to form 233U.
Q8. Match the following list-1 with list-2 and select correct code:
In the below pair of elements and their ores, match correct with
each other. (A) Nitrogen (B) Antimony (1) Chile salt paper (2) Rubby
silver (C) Phosphorus (3) Chlor apatite (D) Arsenic (4) Realgar
Codes:
AB C D (a) 1234 (b) 4321 (c) 3214 (d) 2134 Ans. – (a) Nitrogen
Antimony Phosphorus Arsenic
– Chile salt paper
– Rubby silver
– Chlor apatite
– Realgar
Q9. Basic slag of steel plants contain which of the following
fertilizers:

(a) Potash fertilizers


(b) Phosphorus fertilizer
(c) Nitrogenous fertilizers
(d) All of the above
Ans. – (b) The newest slag of steel plants is the ammonium
phosphate and ammonium phosphate has both phosphatic and
nitrogenous fertilizer.
For the basic slag of steel plants is phosphatic/phosphorus fertilizers.
So option (b) is correct.

Q10. In the given below are pairs of element and their ores
match correctly with each other and select correct code:
(A) Antimony (B) Arsenic (C) Nitrogen (1) Phosphorite (2) Stibnite
(3) Arsenolite

(D) Phosphorus (4) Nitre Codes:

AB C D
(a) 1234
(b) 2341
(c) 4231
(d) 2413
Ans. – (b)
Antimony – Stibnite
Nitrogen – Nitre
Arsenic – Arsenolite
Phosphorus – Phosphorite Q11. Which one of the following
statements is untrue:
(a) Nitrogen combined with other element under the normal
conditions. (b) Nitrogen is colourless, odorless gas and slightly lighter
than air.
(c) During respiration, Nitrogen remain unchanged as it enter and
leave the lungs.
(d) All of the above.
Ans. – (a) Under the normal condition Nitrogen does not combined
with other elements. It however does not combined hydrogen, oxygen
and active metal under special condition of pressure and
temperature. Nitrogen does not burn and does not supports in
burning. Nitrogen slightly higher in comparison to air.
Q12. Consider the following statements and select the correct
answer/code:
(1) Putrefying bacteria convert these nitrogen compound into free
nitrogen, which escape into the atmosphere and keep its balance.
(2) Symbiotic bacteria present on the nodules of the roots of
legiminous plant can fix atmospheric pressure of Nitrogen and make
it available to plants.
Code:
(a) 1 and 2 are correct
(b) Only 2 correct
(c) Only 1 correct
(d) Both 1 and 2 are incorrect
Ans. – (b) The purifying bacteria is mention on statement (1) and
statement two is correct answer/code.
Q13. Which one of the given option oxides react with both NaOH
and HCl? (a) ZnO (b) CO2
(c) NaO (d) N2O3
Ans. – (a) ZnO (Zinc oxides) react with both base NaOH (Sodium
hydroxides) and HCl (Hydrochloric acid) and other oxides of
(Aluminium) Al, Zinc (Zn), Sn, Pb are react very successfully with
sodium hydroxides and hydrochloric acid. Q14. Match the following
and select the correct code:
(A) The number of gram equivalent of solute present in one liter of the
solution.
(B) The number of moles of solute dissolved per 100g of the solvent.
(C) The ratio of moles of one components to the total number of
moles present in solution.
(D) The number of moles of solute dissolved per litre of the solution.
(1) Normality
(2) Molality
(3) Mole fraction
(4) Molarity
Codes:
AB C D
(a) 1 2 3 4
(b) 1 4 3 2
(c) 2 4 3 1
(d) 1 3 4 2
Ans. – (a)
A– 1
B– 2
C– 3
D– 4
Q15. In the given following are pairs of elements and their
respective ores. Which one of the following is not correctly
matched:
(a) Antimony – Ruby silver (b) Nitrogen – Chile salt peter (c) Arsenic
– Realgar
(d) Phosphorus – Stibnite Ans. – (d) Stibnite and rubby silver are
ores of Antimony. Arsenolite and Realgar are the ores of Arsenic.
Nitre and Chile salt peter are ores of Nitrogen. Phosphorite ore and
Chlore apatiles are ores of phosphorus. So option (d) is wrong about
ores of phosphorus.
Q16. It is impossible to determine the exact position and exact
momentum of a running micro particle in an atom. This law is
given by which one of the following: (a) Dalton (b) d-Borgelie (c)
Heisenberg (d) Aufbau
Ans. – (c) This rate is known as Heisenberg principle. It states that
the position and momentum of subatomic particle is an atom cannot
be determine correctly. If ΔP be the error in measuring momentum
and ΔX be the error in measuring position of subatomic particle then
according to Heisenberg principle’s ..
which is constant.
Q17. Which one of the following is the correct order of
increasing activity of halogens: (a) Cl, Br, I, f
(b) Br, I, Cl, f
(c) I, Br, Cl, f
(d) f, I, Cl, Br
Ans. – (c) The increasing order of activity of halogen is given below:
I > Br > Cl > f and decreasing order of halogens are
f > Cl > Br > I. f > O > Cl > Br > I = S > C > P = H > N is the order of
decreasing reactivity of non-metal.
Q18. Consider the following statements and select the correct
code:
(1) Solid carbondioxides keeps everything dry on which it is placed.
(2) Solid carbondioxides is called dry ice because it is ice like in
appearance. Code:
(a) 1 is correct
(b) 2 is correct
(c) 1 and 2 are correct
(d) 1 and 2 are incorrect
Ans. – (c) Solid carbon dioxides keeps everything dry on which it is
placed as van der wal’s forces is the weakest bond and hence it is
totally broken by the thermal energy available at R.T.P. This solid
carbondioxides directly evaporates at R.T.P. Q19. Which one of the
following is generally used as control red in a nuclear reactor:
(a) Carbon (b) Aluminium
(c) Cadmium (d) Indium
Ans. – (c) Control material is a nuclear reactor. In a order to achieve
control in reactor a neutron absorbing material is used. Cadmium red
are generally used for this purpose because its high capacity to
absorb neutron. Some other material are also used an alloy of 80%
Ag, 15% Indium and Cadmium.
Q20. Consider the following statements and select the correct
code:
(1) Beryllium oxide is the best moderator of neutron.
(2) Moderator is used in nuclear reactor to slow down the movement
of a neutron emitted in a nuclear fission reaction.
Code:
(a) 1 and 2 are correct
(b) Only 2 is correct
(c) Only 1 is correct
(d) Both 1 and 2 are correct
Ans. – (b) Heavy water, Graphite, Berillium and Berillium oxides act
as a moderator but heavy heater is the best moderator not berillium
oxides. Moderator is used to neutron emitted in a nuclear fission
reaction which is source of energy in a nuclear reactor.
Q21. Match the following list-1 and list-2: List-1 List-2
(A) Qusec (1) Pressure
(B) Byle (2) Earthquake intensity (C) Richter (3) Rate of Flow of
Liquid (D) Bar (4) Computer
Codes:
AB C D (a) 3421
(b) 3412
(c) 1234
(d) 1243
Ans. – (a)
Qusec – Rate of flow of Liquid Byle – Computer (1 Byte = 2 bit)
Richter – Earthquake intensity
Bar – Atmospheric pressure 1 atm = 101.325 K Pa
Q22. Which one of the following gases from the atmosphere
causes the setting or hardening of plaster and mortar: (a) Argon
(b) Carbondioxide (c) Chlorine (d) Nitrogen
Ans. – (b) The atmospheric carbondioxides (CO2) causes the
hardening or setting of plaster and mortar by reacting with the
calcium hydro oxides in them and converting it into a hard mass of
calcium carbonates. Carbondioxides is also supporter of combustion
with the oxygen.
Q23. Which of the following statements regarding the effect of
temperature and pressure on the solubility of a solid and of a
gas in water is incorrect:
(a) The solubility of a gas in water is directly proportional to pressure
over it.
(b) The solubility of most solids in water usually increases with rise
temperature but not the same extents. (c) The solubility of gas in
water
invariably increases with rise temperature.
(d) None of these.
Ans. – (c) According to Henry’s law the solubility of gas in water is
directly proportional to pressure over it. The solubility of gas in water
is invariably increases with the rise of temperature. In some soft/cold
drink such as soda water, lemonade carbondioxides is dissolved in
water pressure of 5 to 10 atmosphere.
Q24. Which of the following is true about one mole of gas:
(a) It always occupies fixed volume at N.T.P.
(b) It always occupies any volume at N.T.P.
(c) It always occupies 3.5 liters.
(d) It always occupies 24.5 liters. Ans. – (a) The gram molecular
weight of a substance is known as mole. The one mole of any
substances contain 6.023 × 1023 molecules of given substances. One
mole of gas at N.T.P. occupies a volume of 22.4 liters.
Q25. Which one of the following Scientist discovered neutron:

(a) Heisenberg (b) Avagdro


(c) Chadwick (d) Bohr
Ans. – (c) In 1932, Sir James Chadwick bombarded metallic lithium
with α-particle (H++ ion is known as α-particle which has mass 4 and
atomic number is 2) and he found that an intense of neutral beam
came out from lithium elements. This neutral beam is compared of
neutral particle as known as neutron which has no charge.

Chemistry : Different Dimensions

Q1. Match the following list-1 and list-2: Pollutants Effect


list-1
(A) Carbon monoxide (CO)
(B) Nitrogen oxide
(C) Dust particle
(D) Lead

list-2
1. Liver and Kidney
2. Cancer
3. Respiration problem
4. Nervous system
Codes:
AB C D
(a) 3412
(b) 1234
(c) 3421
(d) 1243
Ans. – (a)
Carbon monoxide (CO)
– Respiratory problems Nitrogen oxide
– Central Nervous System Dust particle
– Liver and Kidney problem Lead – Cancer
Q2. Match the following List-1 and List-2: List-1
(A) Dry ice
(B) Gene therapy
(C) Cryogenics
(D) Cobalt 60
List-2
1. Treatment of cancer
2. Works on low temperature
3. Solid CO2
4. Treatment of Blood Disease Codes:
AB C D
(a) 3 4 2 1
(b) 1 2 3 4
(c) 1 2 4 3
(d) 3 4 1 2
Ans. – (a)
Dry ice – Solid CO2
Gene therapy – Treatment of Blood Disease
Cobalt 60 – Treatment of Blood Cancer
Cryogenics – Works on low
temperature
Q3. Which one of the following disease is not caused by virus:
(a) Polio (b) Jaundice
(c) Small Pox (d) Diptheria
Ans. – (d) Diptheria is caused by bacteria not virus. Virus expanded
disease is as follows:
Disease
AidsImmune system Polio

Jaundice Dengue
Small Pox Influenza

Affected Part (WBC)


Throat,
Nervous system Liver
Head, Joint, Eye Whole body
Whole body

Q4. Which one of the following is basic principle of Atomic


Bomb:
(a) Nuclear Fission
(b) Nuclear Fusion
(c) Radioactivity
(d) None of the above
Ans. – (a) Generally Atomic Bomb is also called Nuclear Bomb. It is
based on Nuclear Fission principle Uranium (U235) or Plutonium is
used for making Atomic bomb. It is uncontrolled reaction which
produced excess amount of energy. Atomic bomb is destructive form
of nuclear energy. In Nuclear Fission process heavy nuclear is
divided into lighter nucleus through this process huge amount of
energy is produced.

Q5. Nessler’s Reagents is solution of which of the following:


(a) KOH and K2SO3
(b) K2SO4 and KOH
(c) KNO3 and KOH
(d) None of these
Ans. – (a) When NH3 gas is passed in Nessler’s reagent solution, a
brown precipitate is formed. It is very sensitive to NH3 and even
traces of NH3 produce brown colour. The Nessler’s Reagent used for
confirmatory test of Ammonia.

Q6. Match the following list-1 with list-2 and select the correct
code/answer:
list-1
(A) Discovery of Hydrogen
(B) Discovery of Oxygen
(C) Synthesis of Hydrocarbon
(D) X-rays
list-2
(1) Cavendish
(2) Priestly
(3) Kolbe
(4) Rongton
Codes:
AB C D
(a) 1 2 3 4
(b) 4 3 2 1
(c) 3 4 2 1
(d) 4 3 1 2
Ans. – (a)
Discovery of Hydrogen – Cavendish Discovery of Oxygen – Priestly
Synthesis of Hydrocarbon – Kolbe X-ray – Rongton Q7. Match the
following list and select the correct code:
Discovery
(A) Manufacture of Steel (B) Discovery of Oxygen (C) Synthesis of
Hydrocarbon
(D) X-rays

Discoverer (1) Priestly (2) Bossemer (3) Rongton

(4) Kolbe Codes:

AB C D (a) 1234
(b) 2341
(c) 2143
(d) 4231
Ans. – (c)
Manufacture of Steel Discovery of Oxygen
– Bossemer
– Priestly Synthesis of Hydrocarbon – Kolbe
X-rays – Rongton Q8. Which one of the following pairs of
scientist and their contribution is incorrectly matched:
(a) Rangton – X-rays
(b) Kolbe – Synthesis of hydrocarbons
(c) Cavendish – Discovery of oxygen gas
(d) Bossemer – Manufacture of steel

Ans. – (c) Rangton is discoverer of x-ray particle, Kolbe is discover of


synthesis of hydrocarbon, Bossemer is related to manufacturing of
steel but Cavendish is not discoverer of oxygen but he was discover
of hydrogen gas.
Q9. Atoms of different element having differ atomic numbers and
different atomic weight but they have same number of neutrons
is known as:

(a) Isotones (b) Allotropes (c) Isodipheres (d) Isomers Ans. – (a)
Which atom have different atomic number and differ atomic weight
but having same number of neutron is known as Isotones. Some
examples of isotones are given below:
14S30, 15P31 etc.

Q10. When a moist coloured flower is placed in a jar containing


chlorine (Cl) gas, the colour of the flower is/are discharged
permanently. Which of the following chemical reaction is
responsible for this: (a) Decomposition (b) Oxidation (c) Hydrolysis
(d) All of the above Ans. – (b) When a moist colour flower is placed in
jar of chlorine gas, the colour of flower discharged due to its oxidation
to colourless flower H2O + Cl2→ HCl + HOCl. Thus the chlorine
flower + HOCl → Colourless flower + HCl does not/never regain its
original colour in air. Hence bleaching by C/2 is also permanents.

Q11. Bleaching action of SO2 is by:


(a) Oxidation and is temporary
(b) Oxidation and is permanent
(c) Reduction and is temporary
(d) Reduction and permanent
Ans. – (c) SO2 has property of bleaching of coloured substances. In
presence of water it forms H2SO3.
H2O + SO2→ H2SO3
This H2SO3 reduced the coloured substances to colourless
substances. SO2 bleaches colour by the process of reduction. The
substance bleaches by SO2 regain its colours due to oxidation by
atmospheric oxygen.

Q12. Which one of the following is correct about Monoclinic


sulphur:
(a) It is also known as α sulphur (b) It is also known as β sulphur (c)
(a) and (b) both are correct
(d) None of these
Ans. – (b)Monoclinic sulphur is also called/known as β sulphur. β
sulphur is allotropes of sulphur which exist between 96°C to 119°C.
White sulphur, plastic sulphur, milk of sulphur and colloidal sulphur
are another/other allotropes of carbon. α sulphur is also known as
Rhombic sulphur. So option (b) is correct about monoclinic sulphur.

Q13. Which of the following statements are incorrect about


Rhombic sulphur: (a) It is also called β sulphur.
(b) All others form of sulphur get slowly

converted into this allotropes at room temperature.


(c) It is most stable allotropes of sulphur.
(d) None of these.
Ans. – (a) Rhombic sulphur is not known
as β sulphur but it is known as α sulphur.
α (Sulphur) ⇔β sulphur (Monoclinic
sulphur)
Room temperature
β sulphur is called Monoclinic sulphur is
the allotropes of sulphur exist between
96°C to 119°C. Plastic sulphur, milk of
sulphur, white sulphur and colloidal sulphur are another allotropes of
carbon. Q14. Galena and sinabar are ores of which of
the following element:
(a) Phosphorous (b) Tin
(c) Zinc (d) Sulphur
Ans. – (d) Galena and sinabar are ores of
sulphur element. Zinc blende (ZnS), galena
(PbS), iron pyrites (FES2), cinabar (HgS),
copper glance (Cu2S), copper pyrites
(CUfeS2) are ores of sulphur.
Q15. Which one of the following statement is
not correct:
(a) Denterium is an allotropes of hydrogen.
(b) D2O is poisonous in high
concentration but in low
concentration is beneficial for health.
(c) D2O or heavy water stops the growth of bacteria in it and protozoa
and tadpole die in almost puremater
(d) It is used as a moderator in atomic reactor.
Ans. – (a) In the given statements (b), (c),
(d) are correct but (a) is not correct about
dentrium, denterium is an isotopes not
allotropes of hydrogen having atomic mass
= 2 units and atomic number = 1. Q16. Match the following list 1
with list 2 and
select correct code:
(A) Conductor of
electricity (i) Hydrogen
(B) Bad conductor of
electricity (ii) Helium
(C) Noble gas (iii) Graphite
(D) Atomic number = 1 (iv) Diamond
Codes:
AB C D
(a) 1 2 3 4
(b) 3 4 2 1
(c) 4 3 2 1
(d) 3 2 4 1
Ans. – (b)
Conductor of electricity – Graphite
Bad conductor of electricity – Diamond
Noble gas – Helium
Atomic number = 1 – Hydrogen Q17. Which one of the following
statement is
not correct about Ozone:
(a) CFCs are inert themselves but liberates carbon dioxides when
exposed to ultraviolet radiation in the star to sphere.
(b) Ozone is an allotropes of oxygen.
(c) The air near sea contain small quantity of ozone.
(d) Atmospheric ozone is formed by the action of ultraviolet light on
oxygen of the air.
Ans. – (a) (a) is incorrect and other three
are correct about ozone. CFCs inert but liberate chlorine (Cl) atom
when exposed to
ultraviolet radiation in statrosphere. Q18. In several cases, water
molecules present
in crystal are not expelled even about
110°C. They are not free molecules but
they are bonded to the metals ions by
mean of covalent bonds. These water
molecules are known by which one of the
following:
(a) Water of hydration
(b) Structure water
(c) Water of crystallization
(d) Water of constitution
Ans. – (d) In ZnSO4 . 7H2O, 4 water molecules are present as water.
When salt crystallized out from solution, they remain
associated certain molecules of water.
Those molecules of water are called of
crystallization and they are expelled when
heated about 110°C.
Q19. Which one of the following allotropes of
phosphorus exhibit the phenomenon
called phosphorance:
(a) Yellow phosphorus
(b) Red phosphorus
(c) Black phosphorus
(d) White phosphorus
Ans. – (d) White phosphorus produced
light yellowish green light when kept in
dark. This phenomenon is known as phosphorence. Red phosphorus,
white or yellow phosphorus, scarlet phosphorus, black
phosphorus and violet phosphorus are
allotropes of phosphorus element. Q20. Consider the following
statements and
select correct code:
(1) Phosphorus is present in the blood and urine of animals.
(2) Phosphorus catch fire in the air.
(3) Phosphorus occurs in nature in them form of phosphate.
Code:
(a) Only 3 is correct
(b) Only 2 is correct
(c) 2 and 3 are correct
(d) 1, 2, 3 are correct
Ans. – (d) In the given statements all are
true about phosphorus. Phosphorus does
not occur in the nature in free state because
it is a very reactive non-metal. Animals
bones contain about 58% calcium phosphate.
Q21. Which of the following statements is not
correct about ammonia:
(a) It cannot be liquefied by compression at the ordinary temperature.
(b) It is manufactured on a commercial scale by Haber’s process.
(c) It is known as most soluble gas.
(d) All of the above.
Ans. – (a) Ammonia known as most soluble gas known one volume
of water at
room temperature dissolved roughly 800
volume of the gas and 1000 volume at 0°C.
It can be liquefied by compression at ordinary temperature. In
slaber’s process nitrogen and hydrogen in the ratio of 1 : 3. So
option (a) is incorrect about Ammonia. Q22. Which one of the
following is not correct
about estwald process:
(a) Ammonia is oxidized to form Nitrogen oxides in the process.
(b) The end product of process is Nitric acid.
(c) Catalyst used in the above reaction is Molybdenum.
(d) The above reaction is exothermic.
Ans. – (c) In this process ammonia is oxidized to nitric oxides by
mean of oxygen
with air. An optimum temperature 700° to
800°C is maintained. And in order to increase the rate of reaction a
platinum wire-gauge catalyst is used. The NO so obtained is oxidized
to form NO2 by mean of air. NO2 form is absorbed in water when
nitric acid is obtained.
Q23. Match the following list-1 with list-2 and select correct
code:
Compound pH Value (A) Alcohol (1) 6.4
(B) Blood (2) 2.8
(C) Milk (3) 8.4
(D) Sea water (4) 7.4
Codes:
AB C D
(a) 1 2 3 4
(b) 2 4 1 3
(c) 4 3 2 1
(d) 1 2 4 3
Ans. – (b)
Alcohol – 2.8
Blood – 7.4
Milk – 6.4
Sea water – 8.4
Q24. Match the following list-1 and list-2 and choose correct
codes:
List-1 List-2
(A) Vitamin B12 (1) Magnesium (B) Hemoglobin (2) Cobalt
(C) Chlorophyll (3) Tin
(D) Bronze (4) Iron
Codes:
AB C D (a) 2 4 1 3 (b) 1 2 3 4 (c) 1 2 4 3 (d) 2 3 1 4 Ans. – (a)
Vitamin B12 – Cobalt Hemoglobin – Iron
Chlorophyll – Magnesium Bronze – Tin
Q25. Match the following list-1 and list-2: Types of vegetables (1)
Root vegetable (2) Stem vegetable (3) Herbage vegetable (4) Fruit
vegetable Codes:
Examples
(A) Cauliflower (B) Ginger
(C) Turnip
(D) Brinjal
AB C D (a) 3214 (b) 3241 (c) 1234 (d) 1243 Ans. – (a) Cauliflower
Ginger
Turnip
Brinjal
– Herbage vegetable
– Stem vegetable
– Root vegetable
– Fruit vegetable

General Chemistry

Q1. Match the List-I with List-II and select the correct answer
using the codes given below the list:
List-I
(Characteristics) (A) Integral spin
(B) Fractional charge (C) Zero mass
(D) Fractional spin

List-II
(Particle)
(1) Photon (2) Quark (3) Positron (4) Neutrino

Codes: AB C D
(a) 1243
(b) 1234
(c) 3241
(d) 1342
Ans. – (a)
Integral spin
Fractional charge
Zero mass
Fractional spin
– Photon
– Quark
– Neutrino
– Positron Q2. In a radioactive substance has a half-life of four
months. Three-forth of the substances would decay in:
(a) 3 months (b) 9 months (c) 8 months (d) 12 months Ans. – (c)
After 4 months substance remains = 1/2 After 8 months substance
remain = 1/4 After 12 months substance remain= 1/6 So 3/4 of
substance would decay in 8 months.
Q3. Consider the following statements: Hard water is not
suitable for:
(1) Drinking
(2) Washing cloth with soap
(3) Use in boilers
(4) Irrigating crops
Which of these statements are correct? (a) 2 and 3 (b) 1, 2 and 4 (c)
1, 2, 3 and 4 (d) 1 and 3
Ans. – (a) Hardness of the water is due to chloride and sulphate of
calcium and magnesium. In the presence of chloride and sulphate of
calcium and magnesium the cloth cannot be washed and not be use
in boiler.
Q. Which of the following is a mixed fertilizer:
(a) Urea (b) CAN (c) Ammonium sulphate (d) NPK Ans. – (d) NPK –
contain Nitrogen, phosphorus and potassium in different percentage.
It is an ideal mixture for fertilizers. Nitrogen devoted to N, phosphorus
P and potassium K so that mixture we can say NPK.
Q4. Physico – chemical characteristics of water in water sources
undergo change due to:
(a) aquatic fungi
(b) aquatic microcytes
(c) evapotranspiration
(d) Effluents
Ans. – (d) Aquatic fungi, Aquatic macrophytes are natural organism
and evapotranspiration is a natural process by which plants loses
water. Aquafluent from various industrial units and sewage are the
culprits which changed the physico-chemical characteristics of water.
Q5. Match the list-I with list-II and select the correct answer by
using the codes given below the list: Type-I
(A) Khaira
(B) Anaemia (C) Goitre

Type-II
(1) Vitamin C deficiency (2) Iodine deficiency (3) Iron deficiency
(D) Scurvy (4) Zinc deficiency Codes:

AB C D
(a) 1 2 3 4
(b) 2 1 4 3
(c) 3 4 1 2
(d) 4 3 2 1
Ans. – (d)
Khaira – Zinc deficiency Anemia – Iron deficiency Goitre – Iodine
deficiency Scurvy – Vitamin C deficiency Q6. Match the list-I with
list-II and select correct answer using the codes given below the
lists: List-I
(A) Surrogacy (B) Transgenics

(C) Thalassemia
(D) Anthrax

List-II
(1) Womb renting (2) A toxin used

biowafare
(3) Science of
Altering
genomes
(4) A disease due to
a defective gene Codes: AB C D (a) 1
(b) 1
(c) 2
(d) 1
Ans. – (b) Surrogacy Transgenics

Thalassemia

Anthrax
234
342
431
432

– Womb renting
– Science of Altering genomes
– A disease due to a defective gene
– A toxin used biowafare

Q7. Match the list-I with list-II and select the correct answer by
using the codes below: List-I List-II

(A) Myoglobin (B) Carcinoma (C) Haemoglobin (D) Sarpgandha (1)


Radiotherapy (2) Muscle cells
(3) Oxygen transport (4) Tranquillizer

Codes: AB C D
(a) 2134
(b) 3241
(c) 2431
(d) 1234
Ans. – (a)
Myoglobin
Carcinoma
Haemoglobin
Sarpgandha
– Muscle cells
– Radiotherapy
– Oxygen transport
– Transquillizer Q8. Consider the following:
(1) Carbon dioxide
(2) Oxides of Nitrogen
(3) Oxides of Sulphur
Which of the above is/are the emussions/emission from the coal
combustion at the thermal plants?
(a) 1 only (b) 2 and 3 only (c) 1 and 3 (d) 1, 2 and 3
Ans. – (d) Combustion of coal or for that matter any fossil fuel
releases carbondioxide and oxides of Nitrogen and Sulphur. Q9.
‘Yellow cake’ an item of smuggling border is:
(a) a crude form of heroine
(b) a crude form of cocaine
(c) unrefined gold
(d) uranium oxide
Ans. – (d) A uranium oxides smuggling across the border is known
as yellow cake. Enriched uranium can be used in making illegal
nuclear explosive that is dangerous to humanity.
Q10. Which one of the following is used as an explosive:
(a) Phosphorus trichloride
(b) Mercuric oxide
(c) Graphite
(d) Nitroglycerine
Ans. – (d) Nitroglycerine, trinitroglycerin 1, 2, 3-trinitroxypropane and
glyceryl trinitrate is a heavy colourless, explosive, oily liquid obtained
by nitrating glycerol. Nitroglycerine is also used medically as a
vasodilator to treat heart conditions, such as angina and chronic heart
failure. Q11. Which of the following phenomea occur during the
adiabatic heating of gas: (1) Change in the entropy of the gas. (2)
Compression of the gas.
(3) Heat input from the surrounding. (a) 1 and 2 (b) 1 and 3
(c) Only 2 (d) Only 3
Ans. – (d) Adiabatic process is one in which there is not heat
exchange with surrounding. There is an increase in entropy of the
gas during a irreversible process of adiabatic. Adiabatic heating
involves comparison of the gas while adiabatic cooling involves
expansion of the gas.
Q12. Which of the following polymers is widely used for making
bullet proof material: (a) Polyvinyl chloride
(b) Polyamides
(c) Polycarbonates
(d) Polyethylene
Ans. – (c) Poly carbonates are the polymer widely used in making
bullet proof material as like jacket and equipments. Bullet proof jacket
forms with different layer’s of substances, which provided resistance
in penetrating from bullet. Q13. Which of the following statements
is not correct:
(a) The presence of NaCl increase the rate of setting of plaster of
Paris. (b) Barium and stowtrium are found free in the nature.
(c) Gypsum is added to the cement to slow down it rates of setting.
(d) All alkaline earth metal form hydrated salts.
Ans. – (b) Strontium and Barium are alkaline earth metals in group 2
of the modern periodic table. They do not occur in the free states.
Barium occurs as barytes or heavy spar and barium carbonate also
called whiterite. Strontenium is found in combined form as celetine
and storn hanite.
Q14. Assertion (A): Coal based thermal power stations contributes
to acid rain.
Reason (R): Oxides of carbon are emitted when coal burns.
(a) A is true but R is false.
(b) A is false but R is true.
(c) Both A and R are true and R is the correct explanation of A.
(d) Both A and R are true but R not correct explanation of A.
Ans. – (d) Coal based thermal power plants contributes to acid rain
because NO2 and SO2 are emitted from these plants which forms
HNO3 and H2SO4 in atmosphere that cause acid rain. Oxides of
carbon are emitted when coal burns but it does not contributes to acid
rain.
Q15. Which of these following statements is/are correct:
(a) Calcium carbonates is an ingredients of tooth paste.
(b) Bordeaux mixture consists of sodium sulphate and lime.
(c) Liquid sodium is employed as a coolant in nuclear reactors.
(d) Zinc amalgams are used as a dental filling.
Code:
(1) a and b (2) b and c
(3) Only c (4) Only a
Ans. – (3) Liquid Sodium is employed as a coolant in nuclear
reactors. Liquid Sodium is an effective coolants, which absorbs heat
because of its high specific heat.
Q16. With references to ionic compounds consider the following
statements:
(1) Ionic compound are insoluble in alcohol.
(2) Ionic compound is solid states are good conductor of electricity.
Which of the following statements is/are true:
(a) Only 1 (b) Only 2 (c) Neither 1 nor 2 (d) Both 1 and 2 Ans. – (a)
Ionic compounds are not soluble in organic solvent like ether, alcohol,
acetene but soluble in water. Ionic compound is solid states bad
conductor of electricity because there are no free ions to carry charge
but in solution state good conductor of electricity.
Q17. Match the list-I with list-II and select the correct answer
using the code given below the lists: List-I
(A) Bulmia
(B) Cholestranl (C) Atropine (D) Insulin

List-II
(1) Alkaloid
(2) Egg-yolk
(3) Pancreas
(4) Eating disorder

Codes: AB C D
(a) 4
(b) 3
(c) 1
(d) 2
Ans. – (a)
Bulmia
Cholestranl
Atropine
Insulin
321142234413

– Eating disorder
– Pancreas
– Egg-yolk
– Alkaloid

Q18. Match list-I with list-II and select correct answer using the
codes given below the lists:
List-I
(1) Sem milk
(2) Cold drink and

Soda water
(3) Apple
(4) Pickle
List-II
(A) Lactic Acid

(B) Malic Acid (C) Carbonic Acid (D) Acetic Acid

Codes: AB C D
(a) 1234
(b) 1324
(c) 2314
(d) 4231
Ans. – (b)
Sem Milk
Cold drink and
Soda Water
Apple
Pickle
– Lactic Acid

– Carbonic Acid
– Malic Acid
– Acetic Acid

Q19. Match list-I with list-II and select the correct answer using
the codes given below the lists:
List-I
(A) Lactobacillus
(B) Rhizobium
(C) Yeast
(D) Aspergillus
List-II
(1) Alcohol production
(2) Nitrogen fixation
(3) Citric acid formation
(4) Preparation of curd
Codes:

AB C D (a) 1234
(b) 2341
(c) 3421
(d) 4213
Ans. – (d)

Lactobacillus Rhizobium Yeast


Aspergillus
– Preparation of curd
– Nitrogen fixation
– Alcohol production
– Citric acid formation

Q20. Which of the following is not Radioactive:


(a) Astatine (b) Francium
(c) Tritium (d) Zirconium
Ans. – (d) Astatine, Francium and Tritium are Radioactive element
but Zirconium is not Radioactive. Nucleus of Radioactive elements is
unstable. It emits Alpha (α), Beta (β) and Gamma (γ) rays
spontaneously.

Q21. Which one of the following does not contain silver:


(a) Horn silver (c) German silver (b) Lunar caustic (d) Ruby silver

Ans. – (c) Except German silver, all alloy given in question contain
silver. There are some important alloys and their components are
given below–
(i) German silver : Cu – 50%, Zn – 35%,

Ni – 15%
(ii) Lunar caustic – It also known as silver
nitrate (AgNO3)
(iii) Horn silver : It is also called silver
chloride (AgCl). Its component are
Ag and Cl.
Q22. Assertion (A): Chemical reaction becomes faster at higher
temperature.
Reason (R): At higher temperature, molecular motion becomes more
rapid. Code:
(a) A true but R false.
(b) R true but A false.
(c) A and R both true but R is not correct
explanation of A.
(d) Both A and R are true and R is the cor
rect explanation of A.

Ans. – (d) At high temperature chemical reaction becomes faster


because molecules have high kinetic energy and molecular motion is
higher than normal temperature. Also due to molecular motion
change of combining molecules is also high.

Q23. Consider the following statements: The purpose of adding


sodium sulphate and sodium silicate to the detergent in washing
powder is:
(1) To keep the washing powder dry. (2) To maintain the alkalinity of
the

powder.
Which of these statements is/are correct? (a) Only 2 (b) Both 1 and 2
(c) Only 1 (d) Neither 1 nor 2

Ans. – (b) The purpose of adding sodium sulphate and sodium


silicate to the detergent in washing powder is to keep the washing
powder dry and to maintain the basicity of the powder.

Q24. Which one of the following is another name of RDX:


(a) Dextran (b) Cyclohexane (c) Cyanhydrin (d) None of these Ans. –
(d) Another names of RDX is none of these to given option. RDX
another name is cyclonite, hexagon and T4. Full name of RDX is
Research and Development Explosive. RDX chemical name is Cyclo
Try Methylene Tynittraminme. It also called plastic explosive. RDX is
known as cyclonite in USA, Hexagon in Germany and T4 in Italy.

Carbon Compounds and Metals

Q1. The difference between a nuclear reactor and an atomic


bomb is that: (a) The chain reaction in nuclear reactor is controlled.
(b) The chain reaction in nuclear reactor is not controlled.
(c) No chain reaction takes place in nuclear reactor while in the
atomic bomb there is a chain reaction.
(d) No chain reaction takes place in atomic bomb while it takes place
in a nuclear reactor.
Ans. – (a) The main differences between a
nuclear and an atomic bomb is that the
chain reaction in nuclear reactor can be
controlled whereas in nuclear bomb chain
reaction goes uncontrolled.
Q2. Consider the following question/statements:
Coke is one of the materials of charged
added to blast furnace for the production of
steel/iron.
Its function is to:
(1) Act as an oxidant agent
(2) Function as fuel, to supply heat
(3) Act as the reducing agent
(4) Remove silica associated with the iron ore of these statements
Code:
(a) 1 and 3 correct
(b) 1 and 2 correct
(c) 3 and 4 correct
(d) 1 and 4 correct
Ans. – (a) Coke is added to blast furnace
for the production of iron/steel. It function
as fuel to supply heat. Besides supplying
the heat it also acts as a reducing agents. Q3. Which one of the
following is used an
anti freeze (cooled) for the Automobile
engines:

(a) Ethanol (b) Methanol (c) Propyl alcohol (d) Ethylene glycol Ans. –
(d) Ethylene glycol is used as an anti-freeze agent for the automobile
engine. In cold countries where temperature is below zero degree
centigrade automobile engine cannot be used efficiently due to
freezing of fuel.
Q4. Assertion (A): Sodium metal is stored under Kerosene.
Reason (R): Metallic sodium (Na) melts when exposed to air.
Code:
(a) A is false but R is true.
(b) A is true but R is false.
(c) Both A and R are true but R is not

correct explanation of A.
(d) Both A and R are true and R is correct
explanation of A.
Ans. – (b) Sodium is generally kept inside kerosene oil because in
open environment

it react very fast with oxygen and burns to form oxide.

Q5. Assertion (A): Phenyl is used as a household germicides.


Reason (R): Phenyl is phenol derivative and phenol is an effective
germicides. Code:
(a) Both A and R are true but R is the not

correct explanation of A.
(b) A true but R false.
(c) A false but R true.
(d) Both A and R are true and R is the

correct explanation of A.
Ans. – (d) Phenol is an effective germicides. Phenol is derivative of
phenol that’s why it also has germicidal property.

Q6. Which one of the following was as a chemical weapon in


First World War: (a) Water gas
(b) Carbon Monoxides
(c) Hydrogen cyanide
(d) Mustard gas
Ans. – (d) Mustard gas was widely used as a chemical weapon in the
First World War (1914-19). Chemical weapon effect sense system of
pesson leading to asphyxiation and ultimately death.
Q7. Match List-I with List-II and select the correct answer using
the codes given below the lists:
List-I List-II
(A) Potassium nitrate (1) Gun powder (B) Potassium sulphate (2)
Bakery (C) Monopotassium

tartrate (3) Fertilizer (D) Potassium bromide (4) Photography Codes:

AB C D
(a) 1324
(b) 1234
(c) 2314
(d) 3421
Ans. – (a)
Potassium nitrate – Gun powder Potassium sulphate – Fertilizer
Monopotassium tartrate – Bakery Potassium bromide – Photography
Q8. Match List-I and List-II and select the correct answer:
List-I
(A) Pepsin (B) Ptyalin
(C) Renin (D) Oxytocin
List-II (Physiological role)
(1) Converts angiotensinogen in blood into angiotensin
(2) Digest proteins
(3) Hydrolyses fats
(4) Induces contraction of smooth muscles
(5) Digest starch

Codes: AB C D
(a) 2 5 1 4
(b) 1 2 3 5
(c) 2 1 3 4
(d) 1 2 3 4
Ans. – (a)
Ptyalin – Digest starch
Pepsin – Digest proteins
Renin – Converts angiotensino gen into angiotensin
Oxytocin – Induces contraction of smooth muscles. Q9. Assertion
(A): To dilute sulphuric acid
(H2SO4) is added to water (H2O) and not
water to acid.
Reaction (R): The specific heat of water is
quite large.
Choose the correct option from given
above:
(a) Both A and R is correct but R is the not correct explanation of A.
(b) Both A and R true and R is the correct explanation of A.
(c) A is true but R false.
(d) R is true but A false.
Ans. – (b) To dilute sulphuric acid
(H2SO4), acid is added to water, not water
to acid because specific heat of water is
quite large and it can absorb large quantity
of heat produced by H2SO4.
Q10. Which of the following statements
regarding cellulose and starch is not correct:
(a) Both of them are polymers.
(b) Both of them are of plant origin.
(c) Both of them give colour with iodine.
(d) Both of them are made up of glucose molecules.
Ans. – (c) Starch is the reserve substance
in plant cells whereas cellulose is the most important structural
component of the cell wall of plants. Starch are cellulose and are
polymers of glucose. It is branched polymer of α-D glucose units
which are linked by α-1,4 glycosidic bonds. Cellulose is insoluble in
ordinary solvents and is not hydrolysed by boiling dilute acids. Q11.
Match List-I with List-II:
List-I List-II (A) Solder (1) Tin (B) Hypo (2) Chlorine (C) Bleaching
powder (3) Sodium (D) German silver (4) Nickel
Codes:
AB C D
(a) 1 3 2 4
(b) 1 2 3 4
(c) 4 3 2 1
(d) 2 3 4 1
Ans. – (a) Tin used in formation of solder.
Nickel used in formation of German silver.
Bleaching powder contain chlorine and
hypo contain sodium (Na).
Q12. Match the list-1 with list-2 and select the
correct answer by using the codes given
below the list:
List-1 List-2
(A) Blue vitroil (1) Copper sulphate
(B) Caustic soda (2) Magnesium sulphate
(C) Baking soda (3) Sodium hydroxide
(D) Epsorm salt (4) Sodium bicarbonate
Codes:
AB C D
(a) 1 2 3 4
(b) 1 3 4 2
(c) 2 3 1 4
(d) 4 3 2 1
Ans. – (a)

Sodium bicarbonate – Blue vitroil – Magnesium sulphate – Sodium


hydroxide – Baking soda
Copper sulphate Epsom salt
Caustic soda

Q13. An aqueous solution of copper sulphate (CuSO4) is acidic


in nature because the salt undergoes:
(a) Photolysis (b) Dialysis (c) Hydrolysis (d) None of these Ans. – (c)
The aqueous solution in/of copper sulphate (CuSO4) is acidic in
nature because the copper sulphate (CuSO4) undergoes to
hydrolysis.
CuSO4 + H2O → Cu(OH)2 + H2SO4

Q14. Assertion (A): Synthetic detergents can lather well in hard


water.

Reason (R): Synthetic detergent form soluble calcium and


magnesium salt with hard water.
(a) Both A and R are true and R is the

correct explanation of A.
(b) Both A and R are true but R is the not
correct explanation of A.
(c) A true but R false.
(d) Both A and R are false.
Ans. – (a) Hard water contain magnesium and calcium, chloride and
sulphate respec

tively. Synthetic detergents form soluble calcium and magnesium


(Mg) with hard water.

Q15. Which of the following metals does not form amalgam:

(a) Magnesium (Mg) (c) Copper (Cu)


(b) Iron (Fe) (d) Zinc (Zn)

Ans. – (b) Iron does not form amalgam. Amalgam is the compound
formed by combination of metal and mercury. Except iron. All the
given in options other three example forms amalgam with mercury.

Q16. Surface of Aluminium are obtain ‘anodized’. This means


deposition of a layer of:
(a) Zinc oxides
(b) Nickel oxides
(c) Magnesium oxides
(d) None of these
Ans. – (a) Nickel, Chromium and aluminium metal have positive
charge but zinc is less positive compared to these metal. Due to this
less positive charge Aluminium (Al) rod attracts zinc from zinc
solution and zinc deposits on aluminium rod.

Q17. Match the list-I with list-II:


List-I List-II
(A) Hydrogenation (1) Rubber (B) Vulcanization (2) Copper (C)
Cracking (3) Petroleum (D) Smelting (4) Edible fats Codes:
AB C D (a) 1234
(b) 4132
(c) 3124
(d) 4321
Ans. – (b)
Hydrogenation Vulcanization Cracking
Smelting
– Edible fats
– Rubber
– Petroleum
– Copper
(2) Nickel and Cobalt
(3) Zirconium only
(4) None of these
Which of the above option is correct? (a) 1 (b) 4
(c) 3 (d) 2
Ans. – (c) Zirconium rods are used a moderator in nuclear reactor
that absorbs to slow down chain reaction.
Q20. Match the following list:
(A) Normal salt (1) Alum
(B) Acidic salt

(C) Basic salt (D) Bi-salt

(2) Barium chlo


ride/Sodium chloride
(3) Sodium bicarbonate
(4) Ferric hydroxide Choose the correct options: Codes:
AB C D (a) 3
(b) 2
(c) 2
(d) 1
Ans. – (b) Normal salt Acidic salt

Q18. The pollution of water in river is measured by the dissolved


amount of: (a) Hydrogen (b) Calcium (c) Oxygen (d) None of these
Ans. – (c) Water pollution in river is measured by dissolved amount
of oxygen. Water is measured as Biological Oxygen Demand (BOD).
Polluted water has high biological oxygen demand. It mean there is
less oxygen dissolved in it.

Q19. Which of the following elements is/are essential for the


construction of nuclear reactors:
(1) Magnese and Nickel
Basic salt Bi-salt
241341314

234

– Sodium chloride (NaCl)


– Sodium bicarbonate (Na2CO3)
– Ferric hydroxide
– Alum

* Formation of salt is carried out by neutralization process.


Acid + Base → Salt

Q21. Which of the following are used as Electrolytes in a dry cell


(battery): (a) NaCl and CaCl2
(b) MgCl2 and ZnCl2
(c) Ammonium chloride and zinc

chloride
(d) Ammonium chloride and calcium
chloride
Ans. – (c) Zinc-carbon cell also called the Leclanche cell or dry cell.
Dry cell has a zinc negative electrode and maganese dioxide positive
electrode, and the electrolyte is a solution of ammonium chloride. The
carbon rod is in contact with the positive electrode and is called the
current collector.
Q22. Arrange the following Hydrocarbon according to Molecular
Weight: (a) Methane, ethane, propane and butane (b) Propane,
butane, ethane and methane (c) Butane, ethane, propane and
methane (d) Butane, propane, ethane and methane Ans. – (a)
Methane (CH4) = 1 × 12 + 4 × 1 = 16 Ethane (C2H6) = 2 × 12 + 6 × 1
= 30 Propane (C3H8) = 3 × 12 + 8 × 1 = 44 Butane (C4H10) = 4 × 12 +
10 × 1 = 58 Q23. Which one of the following chemical is used as
a ‘fixer’ in photography is: (a) Borex
(b) Ammonium Persulphate
(c) Magnesium sulphate
(d) Sodium thiosulphate
Ans. – (d)
Sodium thiosulphate used as a fixer in a photography. It removed Ag
(silver) from negative by forming complex with it. Q24. Match the
following List-1 and List-2: List-1 List-2
(A) German silver (1) Tin
(B) Soldering (2) Nickel (C) Bleaching powder (3) Sodium (D) Hypo
(4) Chlorine

Choose the correct option: Codes:

AB C D (a) 1243
(b) 2134
(c) 1234
(d) 2143
Ans. – (d)
German silver – Nickel Soldering – Tin Bleaching powder – Chlorine
(CaOCl2) Hypo – Sodium (Na2S2O4) Q25. Match the following
Lists: List-1
(A) Cracking
(B) Melting
(C) Hydrogenation (D) Vulcanization

List-2
(1) Rubber
(2) Petroleum (3) Copper
(4) Food fats

Codes: AB C D
(a) 3214
(b) 2341
(c) 2314
(d) 3241
Ans. – (b)
Cracking
Melting
Hydrogenation
Vulcanization
– Petroleum
– Petroleum
– Food fats
– Rubber

Science and Technology

Q1. The Centre for Development of Advanced Computing


(CDAC) has developed a new software ‘NAYAN’. Consider
correct function of NAYAN:

(a) The software is used as a solution for steganography.

(b) It is uses as a tool for cyber forensics. (c) It controls the


computers access to different network services, thus, protecting the
internal network from threats and misuse.
Ans. – (c) NAYAN – It means Network
Abhigam Niyantaran, control the computers access to different
network services
protecting the internal network from fureats and misuse. NAYAN also
ensures that
only authorised personnel have access to
configurations, personal information,
source and destination IPS and roles
assigned to users. It is also used to control
cyber attack.
Q2. Which among the following
primary/secondary ary air pollution is
also known as Brown air:
(a) Photochemical smog
(b) Sulphur dioxide
(c) Nitrogen oxides
(d) Carbon monoxides
Ans. – (a) Photochemical smog is often
called brown air when solar radiation is
intense. In areas or sesson of lesser solar
radiation, smog formation is incomplete
and air is often referred to as a grey air.
Photochemical smog is formed in metropolitan cities.
Q3. Assertion (A): In the periodic table of
chemical elements, electron affinity is
always found to increase from top to bottom in a group.
Reason (R): In a group, the atomic radii
generally increase from top to bottom.

Code:
(a) A and R are true and R is correct explanation.

(b) A and R are true and R is not correct explanation.


(c) A is true but R is false.

(d) A is false but R is true.


Ans. – (b)

Electron Affinity – The amount of energy released/required to add


an electron to an atom. If energy required to add an atom then
electron affinity is negative or vice versa.

The atomic radius decreases when we go left to right in a period but


atomic radius increases when we move top to bottom in a group due
to extra addition of shells.

Q4. Assertion (A): The weight of a body decreases with the increase
of latitude on the Earth.
Reason (R): The Earth is not a perfect sphere.

Codes:

(a) Both A and R are true and R is correct explanation of A.


(b) Both A and R are true but R is not correct explanation.
(c) A is true but R is false.
(d) R is true but A is false.
Ans. – (b)
As weight of body = mg
where g = acceleration due to gravity Here g varies with height/depth

g1 =(1 − 2h) or g = GM
R R2

If distance between body and earth centre increases then the value of
g will decreases i.e. the weight of a body decreases with the increase
of latitude on earth.

Q5. Consider the following statements regarding ‘Gypsum’:

(a) It is commonly called Plaster of Paris. (b) Nasa’s rover ‘spirit’ has
recently found evidence of water one the red planet in the form of a
vein of gypsum protruding from an ancient rock.
(c) Nasa’s rover opportunity and its twin spirit arrived on the opposite
sides of Mars.
Choose the correct option:
(1) a, b and c (2) a and c
(3) All of the above (4) None of these

Ans. – (2) Gypsum is also known as Plaster of Paris and its chemical
name is CaSO4 . 2H2O.

It forms from water flowing through rock.

Evidence of water, in the form of Gypsum, protruding from an ancient


rock was found recently by Nasa’s Mars exploration rover
‘opportunity’. Gypsum is used in formation of cement.

Q6. Consider the following statement: (1) Ionic compounds are


insoluble in alcohol.

(2) Ionic compounds in the solid state are good conductor of


electricity.
Choose correct option:

(a) Only 1 (b) Only 2


(c) Both 1 and 2 (d) Neither 1 nor 2 Ans. – (a)

Ionic compound will be formed more easily between elements with


comparatively low ionisation enthalpies and elements with
comparatively high negative value of electron gain enthalpy.

* Ionic compound are insoluble in alcohol.


* Ionic compounds in the solid state are not good conductor of
electricity. It behaves good conductor of electricity in aqueous
solution.

Q7. Consider the following statements and choose correct


option:
(1) Velocity of sound in air is independent

of temperature.

(2) The velocity of sound in air decreases as the humidity increases.


(3) The velocity of sound in air is not affected by the change in
amplitude and frequency.

(4) Velocity of sound in air is independent of pressure.

Choose the correct option:


(a) 1 and 2 (b) 3 and 4
(c) 1, 2 and 3 (d) All of the above Ans. – (b) Sound is a mechanical
wave i.e.

it requires medium to propagate from one point to another point. It is


longitudinal in nature. It’s speed is maximum in solid medium. Velocity
of sound depends on temperature of medium and it is independent of
pressure.

V=

YRT
M

where Y = elasticity of medium R = gas constant


T = Temperature
M = Molecular weight

Q8. Match the following and select correct code:


Invention

(A) Time taken by earth to orbit of the sun (B) Calculation of value of
‘pi’

(C) Invention of the digit zero


(D) The game of snakes and ladder

Indian Scholar
(1) Bhaskarcharya (2) Budhyana (3) Gyandev (4) Aryabhatta Codes:

AB C D (a) 1234 (b) 4321 (c) 2341 (d) 1243 Ans. – (d)

Time taken by the earth to orbit of the sun

Calculation of value of ‘pi’


Invention of zero

The game of snakes and


– Bhaskarcharya
– Budhyana
– Aryabhatta
ladder – Gyandev Q9. Which of the following greenhouse gas is
emitted from tundra and swaps:

(a) Nitrogen oxide (Nitrous oxide) (b) HFC-23


(c) Carbon dioxide
(d) Methane

Ans. – (d) The source of Methane enteric fermentation is cattle and


insect biomass burning, coal, mines and natural gas leaks, paddies
and swaps. So Methane is a greenhouse gas is emitted from swaps
and tundra.

Q10. Which of the following chemical is used as an insecticide,


to treat a wood in bats, that was recently banned by European
Union directive on the ground that is damaged on the ozone
layer:

(a) Methyl iodide


(b) Methyl chloride
(c) Methyl sulphide
(d) Methyl bromide
Ans. – (d) The European Union has taken a decision recently outlaw
before export Methyl bromide; which is an insecticide to treat raw
blades, made of willow, known as

cleft are exported from England, India, Pakistan where they are
converted into the finished product.

Q11. Match the following lists and select correct code:

Person Known as
(A) Michael Griffin (1) Space-walker

(B) Paul G. Allen (2) Administrator of NASA


(C) Piers Seller (3) Co-founder of Microsoft (D) Mother Teresa (4)
Nobel for peace Codes:

AB C D
(a) 2 3 1 4
(b) 1 2 3 4
(c) 4 3 2 1
(d) 1 4 3 2
Ans. – (a)

Michael Griffin – Administrator of NASA


Paul G. Allen – Co-founder of Microsoft
Piers Seller – Space walker Mother Teresa – Nobel for peace Q12.
Which of the following statements is/are correct about
‘amniocentesis:
(1) It is usually done in the 16th week of pregnancy.
(2) The amount of amniotic fluid withdrawn is only about 20 ml.

(3) It is a diagnostic technique to check for genetic abnormalities in


unborn babies.

Code:
(a) 1 and 2 (b) 2 and 3
(c) All (d) None of these

Ans. – (c) Amniocentesis involves taking a sample of the amniotic


fluid, which surrounds the growing body. It is usually done in 16th
week of pregnancy as by this stage there is enough amniotic fluid to
allow the test to be carried out and enough time to terminate the
pregnancy in case abnormalities are detected. For obtaining a
sample, a hollow needle is carefully inserted under the local
anaesthesia through the expectant mother’s abdomen and about 20
ml of fluid is withdrawn.

Q13. Consider the following statements related to barcode and


select the correct code:
(1) There is a standard barcode.

(2) A barcode symbol typically consists of two parts: a start character


and a data character.

(3) It can be read by optical scanners. Code:


(a) 1 and 3 are correct
(b) 1, 2 and 3 are correct
(c) Only 1 is correct
(d) 1 and 3 are correct
Ans. – (c) A barcode is a optical machine readable representation of
data on certain products. It is the small image of lines (bar) and
spaces, affixed to retail store items, identification cards and postal
mail to identify a particular product number, person or location. The
code uses this sequence to represent numbers and other symbol. A
barcode symbol consists of fine parts: a quiet zone, a start character,
data character, a stop character and another quiet zone. There is no
standard barcode.

Q14. Aspirin is a chemically which of the following:

(a) Sodium Azide


(b) Sodium Salicylic Acid
(c) Ethyl Salicylate
(d) None of these
Ans. – (d) Aspirin is a chemical of Acetyl salicylic acid. Aspirin is a
white crytalline powder widely used as drug for relieving pain,
reducing fever and inflammation. Excessive use can cause effect like
ulcer is the stomach of the human body.

Q15. Tartazine, Erythrosine, Carmoisine and Amaranth are types


of which of the following:

(a) Sweetner (c) Flavour (b) Edible Colour (d) None of these

Ans. – (b) Quinoline yellow, tartazine, erythrosine, sunset yellow,


carmoisine, amaranth, brilliant blue and pancean 4R are edible colour
that are water soluble. Flavour like vanillin are used in James and
Juices. Generally ester are used in the flavour.

Q16. Which one of the following is the symptom of drinking


arsenic contaminated water:

(a) Rabies (b) Diptheria


(c) Keratosis (d) Folliculitis
Ans. – (c) Signs of diffuse and spotted

Keratosis (Pores on the skin) is a symptom of drinking arsenic


contaminated water. The WHO has prescribed the limit for arsenic
contamination in water as 10 PPb (Parts per billion). In many parts of
India, particularly in West Bengal and Bihar, arsenic contamination
has reached alarming proportion. In certain parts of Bhojpur district of
Bihar, arsenic contamination has reached as high as 1861 part per
billion.
Q17. What is the pH value of pure water: (a) 7 (b) 8 (c) 6 (d) 7.5

Ans. – (a) pH value of pure water is 7; it means it will have no acid


and base, it is neutral which substances have value upper than 7
then given substance is the character of base and which have pH
value lower than 7 then given substance is the character of acid.
Which substance has pH value 7 then given substance is neutral.

Q18. Consider the following statements related to Transgenic


Chicken:
(1) This chicken has enhanced green

fluorescent protein gene of Jelly Fish. (2) India has become the first
country in
the world to develop transgenic
chicken.
(3) This chicken is not edible but is for
therapeutic using in viral disease. Code:
(a) 1, 2 and 3 are correct
(b) 1 and 3 are correct
(c) Only 2 is correct
(d) 2 and 3 are correct
Ans. – (b) Transgenic chicken was developed in the US, the UK,
Canada and China. This is the first time this has been done in India.
This chicken has green fluorescent protein, it traditionally refers to
protein first isolated from the Jelly Fish. In cell and molecular biology,
the GPF is frequently used as a reporter of expression.
Q19. Match the following and select correct code:
Institute
(A) Indian Institute of Geomagnetism (B) International Advanced
Research
Centre for Power Metallurgy and New
Material
(C) Salim Ali Centre for Ornithology and
Natural History
(D) Tropic Forestry Research Institutes Located at
(1) Hyderabad
(2) Jabalpur
(3) Mumbai
(4) Coimbatore
Codes:
AB C D
(a) 3142
(b) 1432
(c) 4231
(d) 1234
Ans. – (a)
Indian Institute of Geomagnetism – Mumbai
International Advanced Research Centre for Power Metallurgy and
New Material – Hyderabad
Salim Ali Centre for Ornithology and Natural History – Coimbatore
Tropic Forestry Research Institute – Jabalpur
Q20. Match the following and select correct code:
Oxidation number
(A) 2
(B) 3
(C) 4
(D) 6
The element
(1) Oxidation number of Mn in MnO2 (2) Oxidation number of Ca in
CaO2 (3) Oxidation number of S in H2S2O7 (4) Oxidation number of
Al in NaAlH4 Codes:
AB C D
(a) 1 2 3 4
(b) 2 4 1 3
(c) 4 3 2 1
(d) 2 3 4 1
Ans. – (b)
2 – Oxidation number of Ca in CaO2 3 – Oxidation number of Al in
NaAlH4 4 – Oxidation number of Mn in MnO2 6 – Oxidation number of
S in H2S2O7 Q21. Match the following and select correct code:
Scientists
(A) Arber and Smith
(B) Feldman
(C) Mullis
(D) Temin and Baltimore

Achievement
(1) Discovered endonucleases
(2) Discovered polymerase chain reaction (3) Discovered reverse
transcriptase

(4) Developed Transgenic plant with Agrobacterium T-DNA


Codes:

AB C D
(a) 4 3 1 2
(b) 1 4 2 3
(c) 4 3 2 1
(d) 1 2 3 4
Ans. – (b)
Arber and Smith
– Discovered endonucleases

Feldman – Discovered reverse transcritase


Mullis – Discovered polymerase chain reaction
Temin and
Baltimore – Developed transgenic Agrobacterium T-DNA Q22. Match
the following and select correct code:

Biosphere Reserve
(A) Simlipal (B) Dehong Deband (C) Nokrek (D) Kanchanjunga
States
(1) Odisha (2) Arunachal Pradesh (3) Meghalaya (4) Sikkim
Codes:

AB C D (a) 1 3 4 2
(b) 4 2 1 3
(c) 3 2 4 1
(d) 1 2 3 4
Ans. – (d)
Simlipal – Odisha Dohong Deband – Nokrek – Kanchanjunga –
Arunachal Pradesh Meghalaya
Sikkim

Q23. Match the following and select correct code:


List-I Lis-II

(A) CNG (1) Butane, propane (B) Coal gas (2) Carbon monoxide,
Hydrogen
(C) LPG (3) H2, Methane, Carbon monoxide
(D) Water gas (4) Methane, ethane Codes:

AB C D
(a) 1234
(b) 4321
(c) 4312
(d) 2431
Ans. – (c)
CNG – Methane, ethane
Coal gas – H2, Methane and CO LPG – Butane, propane
Water gas – CO and H2

Q24. Match the following and select correct code:

Scientist
(A) C.N.R. Raw
(B) Jagdish Bhagwati
(C) G.N. Ramchandran
(D) Ashok Jhunjhunuwala
Area of work
(1) Physics
(2) Telecommunication
(3) Economics

(4) Solid state chemistry and material science


Codes:
AB C D G.N.R. Raw – Solid state chemistry (a) 4 3 2 1 and material
science
Jagdish Bhagwati – Economics (c) 2 4 1 3 G.N. Ramchandran–
(b) 4 3 1 2
Physics (d) 1 2 4 3 Ashok Jhunjhunwala – Telecommunication

Ans. – (b)

You might also like